Sei sulla pagina 1di 220

2007

Oral and Maxillofacial Surgery


Self Assessment Tool (OMSSAT)

Administration Dates: April 14 - 28, 2007


The American Board of Oral and Maxillofacial Surgery
2006 - 2007 Oral and Maxillofacial Surgery
Self Assessment Tool (OMSSAT) Committee

Chair - Patrick J. Louis

Anesthesia Pathology/Reconstruction

Section Editor – Jeffery D. Bennett Section Editor – Eric Carlson

Vincent Williams Nabil Abaza


Patrick J. Vezeau Tara Aghaloo
Trevor Treasure Jon Holmes
David Todd Brian Schmidt
Tony Petito Brent Ward
Vasiliki Karlis Dale Baur
Joseph Frielich Rui Fernandes
Vince DiFabio
Kevin Butterfield Orthognathic/Cleft/Craniofacial

TMD/Pain Section Editor – Ramon Ruiz

Section Editor – Samuel McKenna Sean Edwards


Bernard J. Costello
Leon Assael John Caccamese
John H. Campbell Paul S. Tiwana
Louis Mercuri Patrick Ricalde
Stephen Milam Tinerfe J. Tejera
H. Clifton Simmons III Paul S. Tiwana
Alireza Sodeifi Gregory Casey
John Zuniga Jonathan Bailey
L. George Upton
Michael Locke
2007 Oral and Maxillofacial Surgery Self Assessment Tool (OMSSAT)

1. What is the recommended maximum dose of 4% articaine 1:200,000 epinephrine for a 70 kg


adult?

A. 280 mgs

B. 350mgs

C. 420 mgs

D. 490 mgs

Answer: D

Rationale:
Manufacture's recommended maximum dose is 7.0 mg/kg or 3.2 mg/lb. For children
between the ages of 4 and 12 years, the manufacturer recommends a dose 5 mgs/kg or 2.27
mgs/lb.

Reference:
Malamed SF: Handbook of Local Anethesia, 4th Edition. St. Louis, Mosby, 1997 p. 63-64

The American Board of Oral and Maxillofacial Surgery 3


2007 Oral and Maxillofacial Surgery Self Assessment Tool (OMSSAT)

2. Which of the following local anesthetics has the slowest onset time?

A. Lidocaine

B. Prilocaine

C. Bupivicaine

D. Mepivicaine

Answer: B

Rationale:
The pKa of a local anesthetic determines its onset time. The closer the pKa of the
anesthetic is the pH of tissue (7.4), the more rapid the onset time. The pKa of a local
anesthetic is the pH at which equal concentrations of ionized and unionized forms exist.

Reference:
Malamed SF: Handbook of Local Anethesia, 4th Edition. St. Louis, Mosby, 1997 p. 49-73

The American Board of Oral and Maxillofacial Surgery 4


2007 Oral and Maxillofacial Surgery Self Assessment Tool (OMSSAT)

3. The lipid solubility of a local anesthetic determines its:

A. duration of anesthesia.

B. onset time.

C. potency.

D. toxicity.

Answer: C

Rationale:
The lipid solubility of a local anesthetic appears to be related to its intrinsic potency.
Increased lipid solubility permits the anesthetic to penetrate the nerve membrane (which is
90% lipid) more easily. This is reflected biologically in an increased potency of the
anesthetic. Local anesthetics with greater lipid solubility produce more effective
conduction blockade at lower concentrations (lower percentage solutions or smaller
volumes deposited) than the less lipid soluble solutions. Onset time is related to the pKa of
the anesthetic. The degree of protein binding will determine the duration of the local
anesthetic.

Reference:
Malamed SF: Handbook of Local Anethesia, 4th Edition. St. Louis, Mosby, 1997 p. 20

The American Board of Oral and Maxillofacial Surgery 5


2007 Oral and Maxillofacial Surgery Self Assessment Tool (OMSSAT)

4. A 36 year-old atopic male presents for the extraction of a tooth. He has a past history of an
anaphylactic reaction when undergoing a previous dental procedure under local anesthesia.
Which of the following solutions would be best to use in this individual for future dental
procedures?

A. Lidocaine 2% with epinephrine 1:100,000 (multidose vial)

B. Prilocaine 4% plain (dental cartridge)

C. Mepivacaine 3% (multidose vial)

D. Bupivacaine 0.5% with epinephrine 1:200,000 (dental cartridge)

Answer: B

Rationale:
With a history of atopy, one can assume that the greatest number of potential allergens
should be eliminated from any medications administered. Multi-dose vials of local
anesthetics often contain preservatives such as parabens which are allergenic; this is not
the case with single use dental cartridges. The presence of a vasoconstrictor usually is
accompanied by an antioxidant such as a bisulfite which also may be allergenic, and
should be avoided in cases of atopy. Therefore, a non-epinephrine containing solution
without preservatives would be most indicated: prilocaine 4% in a dental cartridge.

Reference:
Malamed S: Handbook of Local Anesthesia. Mosby, St. Louis, 1997

The American Board of Oral and Maxillofacial Surgery 6


2007 Oral and Maxillofacial Surgery Self Assessment Tool (OMSSAT)

5. Which of the following is the cardiovascular manifestation of lidocaine toxicity?

A. Bradycardia

B. Premature ventricular contractions (PVC’s)

C. Prolonged QT interval

D. Hypertension

Answer: A

Rationale:
Lidocaine has a depressor effect on the myocardium. Lidocaine toxicity causes sinus
bradycardia because lidocaine increases the effective refractory period relative to the
action potential duration and lowers cardiac automaticity. The bradycardia is followed by
impaired contractility, massive peripheral vasodilation, hypotension and possible cardiac
arrest. Lidocaine may be used to treat PVC's and lidocaine toxicity produces hypotension,
not hypertension.

Reference:
Malamed SF: Handbook of Local Anesthesia, 4th Edition. St. Louis, Mosby, 1997 p. 269-
270

The American Board of Oral and Maxillofacial Surgery 7


2007 Oral and Maxillofacial Surgery Self Assessment Tool (OMSSAT)

6. Prolonged muscle relaxation can result from the concomitant use of succinylcholine and
which of the following local anesthetics?

A. Bupivicaine

B. Procaine

C. Mepivicaine

D. Articaine

Answer: B

Rationale:
Procaine is an ester local anesthetic and metabolized in the blood by plasma cholinesterase.
Succinylcholine is a depolarizing muscle relaxant that also requires plasma cholinesterase
for hydrolysis. Prolonged apnea or paralysis may result form the concomitant use of these
drugs. Bupivicaine, mepivicaine and articaine are all amides and thus metabolized in the
liver.

Reference:
Malamed SF: Handbook of Local Anesthesia, 4th Edition. St. Louis, Mosby, 1997 p. 35

The American Board of Oral and Maxillofacial Surgery 8


2007 Oral and Maxillofacial Surgery Self Assessment Tool (OMSSAT)

7. Geriatric increases in anesthetic sensitivity is most closely associated with:

A. a decrease in the number of neurons is compensated for by a increased cerebral


metabolic rate.

B. an increase in cerebral metabolic rate, which is unrelated to cerebral blood flow.

C. a decrease in levels of neurotransmitters and receptors in different regions of the brain.

D. a decrease in the number of neurons, which is related to an increase in cerebral blood


flow.

Answer: C

Rationale:
In the elderly, there is a reduction in the number of neurons; this is matched by an decrease
in the cerebral metabolic rate. The cerebral metabolic rate is directly related to the cerebral
blood flow. The decrease in the neurons and neurotransmitters is related to a decrease in
the cerebral blood flow.

Reference:
Cole, D.J. and Schlunt, M.: Adult Perioperative Anesthesia 2004. Philadelphia, Mosby.
Pp.249 and 466.

Power, I. and Kam, P.: Physiology for the Anaesthetist. 2001. London, Arnold, pp42-44.

Stoelting, R.K. and Dierdorf, S. F.: Anesthesia and Co-Existing Disease. 4th Edition.
2002. Philadelphia, Churchill Livingstone, pp. 238-9.

The American Board of Oral and Maxillofacial Surgery 9


2007 Oral and Maxillofacial Surgery Self Assessment Tool (OMSSAT)

8. The efficiency of gaseous exchange in the elderly decreases as a result of:

A. a decrease in the closing volume.


B. a reduced alveolar surface area.
C. a decreased alveolar capillary membrane thickness.
D. a decreased V/Q ratio.

Answer: B

Rationale:
In the elderly there is an increase in the closing volume, an increase in the alveolar
capillary membrane thickness and an increase in the V/Q (ventilation/perfusion) ratio. The
gaseous exchange is decreased because of reduced alveolar surface area.

Reference:
Power, I. and Kam, P.: Principles of Physiology for the Anaesthetist 2001. London,
Oxford Press. pp. 367-8 and 73-92.

Cole, D.J. and Schlunt, M.: Adult Perioperative Anesthesia. 2004. Philadelphia, Mosby.
p. 467.

The American Board of Oral and Maxillofacial Surgery 10


2007 Oral and Maxillofacial Surgery Self Assessment Tool (OMSSAT)

9. When comparing a morbidly obese patient to a non-obese patient, which of the following
statements is correct?

A. Oxygen consumption is higher in the non-obese patient.

B. Functional residual capacity is the same.

C. Time to desaturation with a period of apnea is the same.

D. Positioning may diminish pulmonary reserve more in the obese patient.

Answer: D

Rationale:
Patients who are morbidly obese have changes in pulmonary, cardiovascular,
gastrointestinal, and metabolic systems. Patients who are morbidly obese have increased
minute ventilation at rest to meet the metabolic needs of the increased tissue mass.
Changes in lung volumes at rest include reduced FRC, vital capacity, and total lung
capacity. Closing volume is unchanged and reduced FRC can result in lung volumes
below closing capacity in normal tidal ventilation. Anesthesia compounds these problems
with greater reductions in FRC in obese patients compared to nonobese patients of the
same age. As a result, obese patient's ability to tolerate periods of apnea is reduced.
Patient positioning aggravates these changes in lung volumes and contributes to poor
respiratory reserve in obese patients. Reverse Trendelenberg is the most optimal position
for lung volumes whereas supine position and Trendelenberg are worst in terms of safe
apnea periods and recovery time.

Reference:
Stoelting RK, Dierdorf SF. Anesthesia and Co-Existing Disease, 2002.

Todd DW. Anesthetic Considerations for the Obese and Morbidly Obese Oral and Oral
and Maxillofacial Surgery Patient. J Oral and Maxillofacial Surgery 63:1348-1353, 2005.

The American Board of Oral and Maxillofacial Surgery 11


2007 Oral and Maxillofacial Surgery Self Assessment Tool (OMSSAT)

10. In the elderly, differences in drug response include a/an:

A. increase in MAC.

B. decreased rate of hepatic glucuronidation of morphine.

C. lowered induction dose of thiopental.

D. shorter recovery time to the normal ventilatory response with fentanyl.

Answer: C

Rationale:
The elderly require a lower dose of thiopental for the induction of anesthesia. With
increasing age, the MAC for inhalation anesthetics decreases. The recovery of normal
ventilatory drive after fentanyl is delayed. Regarding the rate of hepatic synthetic
reactions: glucuronidation of morphine is also unchanged in the elderly but the rate of
hepatic oxidative and reductive reactions are decreased with an increase in age.

Reference:
Longnecker, DE et al.:Principles and Practice of Anesthesiology. 2nd Edition, 1997. St.
Louis: Mosby. pp. 481-2.

Calvey, T.N. and Williams, N.E.: Pharmacology for Anaesthetists 4th Edition 2001,
London, Blackwell Science pp 106-112.

Cole, D.J. and Schlunt, M.: Adult Perioperative Anesthesia. 2004. Philadelphia, Mosby,
pp.468-470.

The American Board of Oral and Maxillofacial Surgery 12


2007 Oral and Maxillofacial Surgery Self Assessment Tool (OMSSAT)

11. Which of the following agents is associated with the highest incidence of nausea and
vomiting in the post-operative period?

A. Etomidate

B. Propofol

C. Ketamine

D. Clonidine

Answer: A

Rationale:
Etomidate is associated with a high incidence of nausea and vomiting. Although ketamine
can cause nausea and vomiting the incidence is much lower. Propofol has antiemetic
effects at higher dosages. Clonidine has a low incidence of post operative nausea and may
be beneficial in the treatment of cyclical vomiting syndrome.

Reference:
Miller RD (ed) Anesthesia 4th edition. New York, Churchill Livingstone, 1994; p268

Stoelting RK: Pharmacology and Physiology in Anesthetic Practice, 2nd Edition.


Philadelphia, JB Lippincott, 1991.

The American Board of Oral and Maxillofacial Surgery 13


2007 Oral and Maxillofacial Surgery Self Assessment Tool (OMSSAT)

12. Which drug should be avoided in the patient with pre-existing renal dysfunction?

A. Ketorolac (Toradol)

B. Midazolam (Versed)

C. Methohexital (Brevital)

D. Droperidol (Inapsine)

Answer: A

Rationale:
Barbiturates and benzodiazepines both reduce glomerular filtration rate but can be used in
patients with renal failure. Doses need to be reduced and the anesthetic medication titrated
to effect. Opioids, such as morphine and meperidine have active metabolites that are
renally excreted and require cautious administration. The glucoronidated morphine
compound is more potent than morphine itself. Droperidol, although, not generally used in
outpatient anesthesia currently produces an -adrenergic blocking effect that preserves
renal blood flow and hemodynamics. Ketorolac or other NSAIDs are prostaglandin
inhibitors that interfere with the prostaglandin mediated renal vasodilatation and has been
shown to produce acute renal failure.

Reference:
Altee John L. Complications in Anesthesia. WB Saunders 1999

Roizen MF, Fleisher LA. Essence of Anesthesia Practice. WB Saunders 1997

The American Board of Oral and Maxillofacial Surgery 14


2007 Oral and Maxillofacial Surgery Self Assessment Tool (OMSSAT)

13. A 22 year-old female presents for removal of her third molars under deep sedation. She has a
history of Wolff-Parkinson-White Syndrome (WPW). Midazolam, fentanyl and propofol are
administered, and she develops atrial fibrillation consistent with the re-entry phenomenon of
WPW. The most appropriate medication to treat this problem is:

A. adenosine.

B. diltiazem.

C. esmolol.

D. amiodarone.

Answer: D

Rationale:
Drugs such as adenosine, calcium channel blockers, and beta blockers can cause a
paradoxical increase in the ventricular response to the rapid atrial impulses of atrial
fibrillation. This increase in ventricular response occurs because these agents slow or block
conduction through the AV node and in some instances may facilitate conduction to the
ventricle via the accessory pathway. The treatment of choice for a atrial fibrillation
associated with Wolff-Parkinson-White syndrome is direct cardioversion, which if
inappropriate, is then followed preferentially by amiodarone.

Reference:
ACLS: Principles and Practice, Pp. 328, American Heart Association 2003

GE Morgan et al, Clinical Anesthesiology, pp. 385, Lange/McGraw-Hill, 2002

The American Board of Oral and Maxillofacial Surgery 15


2007 Oral and Maxillofacial Surgery Self Assessment Tool (OMSSAT)

14. A 25 year-old female is sedated with nitrous oxide and intravenous ketamine. In the recovery
area the patient is noted to be hallucinating. This could possibly have been prevented by
concomitant use of

A. scopolamine.

B. phenothiazenes.

C. propofol.

D. droperidol.

Answer: C

Rationale:
Ketamine delirium may be prevented by concomitant use of a benzodiazepine or propofol.
The other 3 answers may also be responsible for post operative / emergent excitement.

Reference:
Firestone, Lebowitz and Cook: Clinical Anesthesia Procedures of the Massachusetts
General Hospital , Third edition, P. 497

Barash, Cullen and Stoelting, Handbook of Clinical Anesthesia, Fourth edition, P.864

The American Board of Oral and Maxillofacial Surgery 16


2007 Oral and Maxillofacial Surgery Self Assessment Tool (OMSSAT)

15. A 65 year-old 45 kg frail cardiac transplant patient presents for extractions. Local anesthesia
with mepivicaine 3% 270 mg and sedation with midazolam 1.25 mg i.v. with a 50% mixture
of nitrous oxide and oxygen is used. The patient becomes hypotensive and bradycardic with a
narrow complex QRS complex. What drug or intervention would you use in the management
of this patient?

A. Flumazenil 4 mg IV push

B. Fluid bolus of 500 mg normal saline

C. Dopamine 200 µg/min IV infusion

D. Atropine 0.5mg IV push

Answer: C

Rationale:
The cardiac transplant patient's chronic problems may include diffuse accelerated
atherosclerosis leading to generally non-anginal myocardial ischemia and decreased
cardiac output. Additionally, the chronotropic response is markedly decreased in the
denervated transplanted heart. This patient is suffering from symptomatic bradycardia.
One contributing factor could be the local anesthetic. Although the dosage of mepivicaine
administered is within the upper acceptable recommended guidelines the practitioner must
take into consideration the patient. Cardiovascularly compromised patients may be on
several drugs that interact with intraoperatively administered medications. It would not be
uncommon for a patient with this history to be taking an additional local anesthetic type
dysrhythmic drug acting on her myocardium (such as procainamide.) Additionally, a local
anesthetic injection into or near the pterygoid plexus can cause a precipitous rise in local
anesthetic blood levels, especially with a lack of vasoconstrictor. The appropriate first
step would be external pacing if available. If this is not available, a dopamine infusion
titrated between 5-20 µg/kg/min would give both alpha and beta effects to raise heart rate
and blood pressure. Since this patient's transplanted sinus node is denervated, atropine
would not reverse her bradycardia. Flumazenil should be given in incremental doses of
0.05-0.1 mg slowly. For a frail patient, a 500 ml fluid bolus might significantly increase
cardiac preload and further decrease cardiac output.

Reference:
ACLS Provider manual, 2001 American Heart Association, pp145-155

Weinberg, GL: Basic Science Review of Anesthesiology McGraw-Hill, New York, 1997
p 25

The American Board of Oral and Maxillofacial Surgery 17


2007 Oral and Maxillofacial Surgery Self Assessment Tool (OMSSAT)

16. Which of the following agents can be used to reverse the effects of dexmedetomidine?

A. Flumazenil

B. Narcan

C. Atipamezole

D. Atropine

Answer: C

Rationale:
Flumazenil and Narcan are used to reverse effects of benzodiazepines and narcotics,
respectively. Atropine is an antimuscarinic and does not reverse deximedetomidine.
Atipamezole is an alpha2-adrenoceptor antagonist with an imidazole structure. It rapidly
reverses sedation/anesthesia induced by alpha2-adrenoceptor agonists. In humans,
atipamezole at doses up to 30 mg produces no cardiovascular or subjective side effects,
while at a high dose (100 mg) it produced subjective symptoms, such as motor
restlessness, and an increase in blood pressure.

Reference:
Jones JG, Taylor PM. Receptor specific reversible sedation: dangers of vascular effects.
Anesthesiology 1999;90: 1489-1490.

Scheinin H, Aantaa R, et al. Reversal of the sedative and sympatholytic effects of


dexmedetomidine with a specific alpha 2 adrenoceptor antagonist atipamezole: a
pharmacodynamic and kinetic study in healthy volunteers. Anesthesiology 1998;89:574-
584.

The American Board of Oral and Maxillofacial Surgery 18


2007 Oral and Maxillofacial Surgery Self Assessment Tool (OMSSAT)

17. A 40 year-old male Type II diabetic arrives at your office for the extraction of multiple teeth
under intravenous sedation. He is combative and disoriented. He has been NPO for six
hours. He took his diabetic medications (acarbose and glyburide) this morning. A finger
stick glucose is 50. What is the next appropriate step?

A. Fruit juice PO

B. Intramuscular 50% Dextrose

C. Intramuscular glucagon

D. Dextrose tablet dissolved sublingually

Answer: C

Rationale:
Hypoglycemia after fasting while on oral hypoglycemics is a concern in office anesthesia
settings, and patients must understand that they should avoid immediate preoperative
hypoglycemic medications in that setting. This patient's medications include a second
generation sulfonylurea (glyburide) which increases endogenous insulin release; and an α-
glucosidase inhibitor (acarbose) which blocks intestinal breakdown of fructose and
dextrose into absorbable glucose. In this case intake of fructose-containing juice is not
indicated. Although one could consider oral administration of glucose containing tablets or
solution, oral administration in a disoriented patient is also relatively contraindicated.
Intramuscular administration of the very hyperosmotic 50% dextrose would cause extreme
muscle irritation; its use should be limited to intravenous administration in a large vein.
Dextrose is not absorbed significantly through sublingual epithelium. Glucagon 1 mg IM
would be indicated. This would be absorbed and cause hepatic glycogenolysis and
gluconeogenesis, raising blood glucose levels in about 15 minutes. Glucagon may cause
significant nausea.

Reference:
Sarasin D: Ambulatory anesthetic management of the patient with diabetes. Oral
Maxilofac Surg Clin N Amer 11(4): 589-99, 1999

Washington Manual of Medical Therapeutics, 30th ed, Liippincott, Philadelphia, 2001


pp463-66

The American Board of Oral and Maxillofacial Surgery 19


2007 Oral and Maxillofacial Surgery Self Assessment Tool (OMSSAT)

18. What effect will ketamine have on the degree of regurgitation in a patient with mitral valve
prolapse with regurgitation?

A. Increase
B. Decrease
C. No effect
D. Variable

Answer: A

Rationale:
Ketamine is discouraged in patients with mitral valve prolapse with regurgitation, due to
its sympathomimetic actions. It will increase vascular resistance and worsen regurgitant
flow.

Reference:
Waxman K, Shoemaker WC, Lippman M: Cardiovascular effects of anesthetic induction
with ketamine. Anesth Analg 1980; 59:355-8.

White PF, Way WL, Trevor AJ: Ketamine: Its pharmacology and therapeutic uses. Anesth
1982; 56:119-36.

The American Board of Oral and Maxillofacial Surgery 20


2007 Oral and Maxillofacial Surgery Self Assessment Tool (OMSSAT)

19. Barbiturates have which of the following effects on the myocardium?

A. Directly sensitize the myocardium to arrhythmias

B. Directly increase myocardial contractility

C. Indirectly increase heart rate by inducing venodilation

D. Indirectly increase myocardial contractility

Answer: C

Rationale:
Barbiturates have no effect on myocardial sensitization. They decrease myocardial
contractility. Reflex tachycardia is common after an induction dose of barbiturate to
compensate for the vasodilatation.

Reference:
Miller RD (ed) Anesthesia 4th edition. New York, Churchill Linignstone, 1994; p238

Lieblich SE. Methohexital Versus Propofol of Outpatient Anesthesia Part 1: Methohexatal


is Superior. JOMS 58:811-815, 1995

The American Board of Oral and Maxillofacial Surgery 21


2007 Oral and Maxillofacial Surgery Self Assessment Tool (OMSSAT)

20. Ketamine’s direct effect on the heart is:

A. chronotropic depression.

B. chronotropic stimulation.

C. inotropic depression.

D. inotropic stimulation.

Answer: C

Rationale:
Ketamine direct action on the myocardium is a negative inotropic effect. Its centrally
mediated sympathetic responses (indirect activation of the sympathetic nervous system)
usually override the depression. Ketamine causes an increase in circulating
catecholamines, especially norepinephrine, by inhibiting reuptake at postganglionic
sympathetic neurons.

Reference:
Reich DL, Silvay G. Ketamine: an update on the first twenty-five years of clinical
experience. Can J Anaesth 1989;36:186–97.

Hirota K, Lambert DG. Ketamine: its mechanism(s) of action and unusual clinical uses. Br
J Anaesth 1996; 77:441–4.

The American Board of Oral and Maxillofacial Surgery 22


2007 Oral and Maxillofacial Surgery Self Assessment Tool (OMSSAT)

21. Which of the following is a side effect associated with etomidate?

A. Decreased venous return and myocardial contractility

B. Intra-arterial injection causing nerve injury and gangrene

C. Adrenal suppression lasting at least 6 hours

D. Triggering of porphyria in susceptible individuals

Answer: C

Rationale:
Etomidate maintains hemodynamic stability and has little effect on the heart. Barbiturates
are known to have severe effects associated with intra-arterial injection. This group is also
responsible for porphyria in susceptible individuals. Etomindate causes adrenal
suppression and steroid administration may be necessary in patients already having adrenal
axis suppression.

Reference:
Miller RD (ed) Anesthesia 4th edition. New York, Churchill Livingstone, 1994; p268

Dembo JB. Methohexital Versus Propofol of Outpatient Anesthesia Part II: Propofol is
Superior. JOMS 58:816-820, 1995

The American Board of Oral and Maxillofacial Surgery 23


2007 Oral and Maxillofacial Surgery Self Assessment Tool (OMSSAT)

22. Which of the following neuromuscular blockers requires little if any dosage .change in the
elderly patient?

A. Pancuronium

B. Vecuronium

C. Cisatracurium

D. Mivacurium

Answer: C

Rationale:
Cisatracurium is the neuromuscular blocker that is metabolized by Hoffman degeneration
and organ-independent elimination. This process is non-enzymatic and occurs
spontaneously and thus needs little if any change in the routine dose in the elderly. Also
the clinical effect is not prolonged. Pancuronium is dependent on renal function and
excretion which decreases with age. Vecuronium is dependent on hepatic function which
also decreases with age; and mivacurium is metabolized by plasma cholinesterase which
also decreases with age. Because of the age dependent nature of metabolism for these three
neuromuscular blockers, they all require a reduction in dosage for the elderly.

Reference:
Cole, D.J and Schlunt, M: Adult Perioperative Anesthesia. 2004. Philadelphia, Mosby, p.
471.
Vickers, M.D. and Power, I.: Medicine for Anaesthetists. 4th Edition, 1999. London,
Blackwell Science, p. 114

The American Board of Oral and Maxillofacial Surgery 24


2007 Oral and Maxillofacial Surgery Self Assessment Tool (OMSSAT)

23. A 72 year-old male presents for removal of tooth #3. His written medical history is
significant for stable angina, congestive heart failure, type 2 diabetes mellitus, and coronary
stent placed after myocardial infarction 3 months ago. He is taking metoprolol and
furosemide. A history and physical examination reveals the following: no chest pain with
mild exercise, reduced carotid pulses, audible 3rd and 4th heart sound, and inspiratory basilar
rales. The most influential determinant of this patient’s perioperative cardiac risk status is:

A. stable angina.

B. coronary stent placement following MI 3 months ago.

C. congestive heart failure.

D. diabetes mellitus.

Answer: C

Rationale:
This patient has signs of acute congestive heart failure. Physical signs of left ventricular
CHF include: reduced carotid pulsations, diffuse and laterally displaced apical impulse,
palpable and audible 3rd and 4th heart sounds, accentuated pulmonic 2nd sound,
inspiratory basilar rales, and right-sided pleural effusion. There are several factors which
contribute to determining a patient's overall preoperative cardiac risk. As shown in the
ACC/AHA 2002 Guidelines Update on Perioperative Cardiovascular Evaluation for
Noncardiac Surgery and Goldman et al. (Multifactorial index of cardiac risk in non-cardiac
surgical procedures. N Engl J med 1977;297:845-50.), the preoperative factor which had
the highest predictive value for an adverse cardiac event is the patient's acute congestive
heart failure.

Reference:
ACC/AHA 2002 Guidelines Update on Perioperative Cardiovascular Evaluation for
Noncardiac Surgery . Circulation. March 5,2002
Goldman et al.: Multifactorial index of cardiac risk in non-cardiac surgical procedures. N
Engl J med 1977;297:845-50.

The American Board of Oral and Maxillofacial Surgery 25


2007 Oral and Maxillofacial Surgery Self Assessment Tool (OMSSAT)

24. In a patient with interstitial lung disease (ILD) why would one expect the FEV1/FVC ratio to
be normal?

A. Vital capacity is reduced in ILD

B. FEV1 is normal in a restrictive pattern

C. ILD does not alter PFT’s

D. ILD affects only the diffusing capacity of CO2

Answer: A

Rationale:
In ILD a known or unknown agent stimulates macrophage induced alveolitis. This results
in a loss of functioning alveolar capillary units and an accumulation of fibroblasts. The
accumulation of collagen in the alveolar interstitium produces a thickened, scarred and
eventually small lung.

The FEV/FVC ratio is typically normal in ILD due to a tendency for patients to have small
lung volumes. Small lung volumes (VC – vital capacity, TLC – total lung capacity) will be
noted on both PFT examination and Chest radiographs. While FEV1 is decreased in ILD,
the FVC is decreased at the same time resulting in a normal FEV/FVC ratio.

Reference:
Braunwald, E, et al: Interstitial Lung Disease: Harrison's Principles of Internal Medicine.
McGraw-Hill Co.
Alex, CG and Tobin, MJ: Assessment of Pulmonary Function in Critically Ill Patients, in
Textbook of Critical Care, Shoemaker, WC. 4th Edition. W B Saunders Co. 2000.

The American Board of Oral and Maxillofacial Surgery 26


2007 Oral and Maxillofacial Surgery Self Assessment Tool (OMSSAT)

25. Which effect is seen with propofol?

A. Elevation of intracranial pressure

B. Increases in intraocular pressure

C. Potentiation of neuromuscular blockade

D. Increases in bronchodilation

Answer: D

Rationale:
Propofol has a direct smooth muscle effect on the bronchi, causing bronchodilation.
Propofol decreases intracranial and intraocular pressure, and it does not potentiate
neuromuscular blockade.

Reference:
Miller RD (ed) Anesthesia 4th edition. New York, Churchill Livingstone, 1994; p270-272

Dembo JB. Methohexital versus Propofol of Outpatient Anesthesia Part II: Propofol is
Superior. JOMS 58:816-820, 1995

The American Board of Oral and Maxillofacial Surgery 27


2007 Oral and Maxillofacial Surgery Self Assessment Tool (OMSSAT)

26. Which drug must be used with caution in a patient with a history of epilepsy?

A. Propofol

B. Fentanyl

C. Dexmedetomidine

D. Methohexital

Answer: D

Rationale:
Methohexital can cause seizure activity; and therefore is contraindicated in patients with
temporal lobe seizures. The other three drugs are not associated with seizure-like activity.

Reference:
Cote, Charles J., Pediatric Anesthesia, pg. 2375 in Miller's Anesthesia, Sixth Edition,
Volume 2, by Elsevier Inc.

Stoelting, Robert K. MD, Dierdorf, Stephen F, MD, Chapter 17, Anesthesia and Coexisting
Disease, Fourth Edition, pg. 284,

The American Board of Oral and Maxillofacial Surgery 28


2007 Oral and Maxillofacial Surgery Self Assessment Tool (OMSSAT)

27. A 61 year-old patient presents for removal of tooth #2, and general anesthesia He has aortic
regurgitation. The perioperative management of this patient includes which of the following?

A. Relative hypovolemia

B. Negative inotropic agents

C. Increase afterload

D. Positive chronotropic agents

Answer: D

Rationale:
The perioperative goals of managing aortic regurgitation are to promote forward flow by
decreasing afterload and to maintain a normal to slightly increased heart rate.

Reference:
Goldman et al. (Multifactorial index of cardiac risk in non-cardiac surgical procedures. N
Engl J med 1977;297:845-50.

DE Longnecker, FL Murphy, Introduction to Anesthesia, 9th ed., pp. 291, Saunders, 1997
GE Morgan et al, Clinical Anesthesiology, pp. 414, Lange/McGraw-Hill, 2002

The American Board of Oral and Maxillofacial Surgery 29


2007 Oral and Maxillofacial Surgery Self Assessment Tool (OMSSAT)

28. A 49-year-old patient presents for removal tooth #15 under general anesthesia. His clinical
examination is significant for a IV/VI holosystolic murmur heard at the apical area and
radiates to left axilla. The perioperative management should include which of the following?

A. Decrease intravascular volume

B. Decrease myocardial contractility

C Increase peripheral vascular resistance

D Increase heart rate

Answer: D

Rationale:
A holosystolic murmur heard at the apex and radiating to the left axilla is consistent with a
mitral regurgitation. The perioperative goals of managing mitral regurgitation are to
promote forward flow by decreasing afterload and to maintain a normal to slightly
increased heart rate.

Reference:
DE Longnecker, FL Murphy, Introduction to Anesthesia, 9th ed., pp. 291, Saunders, 1997

GE Morgan et al, Clinical Anesthesiology, pp. 414, Lange/McGraw-Hill, 2002

The American Board of Oral and Maxillofacial Surgery 30


2007 Oral and Maxillofacial Surgery Self Assessment Tool (OMSSAT)

29. An 18 year-old with Cerebral Palsy (CP) requires intubation for a general anesthetic.
Succinylcholine is contraindicated in this patient secondary to

A. structural similarity to acetylcholine.

B. potential for a significant increase in circulating levels of potassium.

C. deficiency of pseudocholinesterase.

D. fasciculation and post-operative muscle pain.

Answer: B

Rationale:
Circulating potassium increases significantly in patients with CP following administration
of succinylcholine and may lead to lethal arrhythmias.

Reference:
Firestone, Lebowitz and Cook: Clinical Anesthesia Procedures of the Massachusetts
General Hospital , Third edition, P. 171

Barash, Cullen and Stoelting, Handbook of Clinical Anesthesia, Fourth edition, P.585

The American Board of Oral and Maxillofacial Surgery 31


2007 Oral and Maxillofacial Surgery Self Assessment Tool (OMSSAT)

30. A patient with which condition is least likely to be an aspiration risk during an outpatient
non-intubated general anesthetic?

A. Systemic lupus erythematosus

B. Parkinson’s disease

C. Myasthenia gravis

D. Diabetes mellitus

Answer: A

Rationale:
Parkinson's disease, myasthenia gravis, a transection of the vertebral column above T4, and
diabetes mellitus all increase the risk of aspiration. Parkinson's disease and myasthenia
gravis impair the patient's ability to control their airway. Parkinson’s' disease is a
neurodegenerative disease characterized by a loss of dopaminergic neurons in the
substantia nigra of the basal ganglia. Clinical signs and symptoms include a resting tremor,
rigidity, postural instability and bradykinesia. Myasthenia gravis is an autoimmune disease
of the neuromuscular junction. Clinical symptoms include fatigue, weakness of the striated
muscles, diplopia, inspiratory muscle weakness, and bulbar weakness with impaired ability
to handle secretions and swallow. The diabetic patient may have autonomic dysfunction
resulting in gastroparesis. Frequently gastroparesis is undiagnosed and asymptomatic. This
increases the risk of gastric aspiration. The patient with systemic lupus erythematosus may
present anesthetic airway and pulmonary concerns such as reduced TMJ range of motion,
decreased arytenoid movement, diaphragmatic dysfunction, pulmonary infiltrates and
reduced PFTs. However, the patient is not at increased risk for aspiration.

Reference:
Altee John L. Complications in Anesthesia. WB Saunders 1999

Roizen MF, Fleisher LA. Essence of Anesthesia Practice. WB Saunders 1997

The American Board of Oral and Maxillofacial Surgery 32


2007 Oral and Maxillofacial Surgery Self Assessment Tool (OMSSAT)

31. The use of succinylcholine is acceptable in which patient population?

A. Behcet’s muscular dystrophy

B. Four month old spinal cord injury

C. Multiple sclerosis

D. Myasthenia gravis

Answer: D

Rationale:
Myasthenia gravis (MG) is a neuromuscular disease that is characterized by weakness and
fatigability of skeletal muscles. The MG patient is more sensitive to the use of
nondepolarizing relaxants. However, because there are fewer functional receptors the MG
patient may demonstrate increased resistance to depolarizing muscular relaxants. While
many anesthesiologists may prefer to avoid neuromuscular blocking agents,
succinylcholine (in higher doses) can be used in the MG patient and provide satisfactory
intubating conditions. Succinylcholine may precipitate hyperkalemia in muscular
dystrophy, spinal cord injuries less than 6 to 8 months and MS.

Reference:
Altee John L. Complications in Anesthesia. WB Saunders 1999

Roizen MF, Fleisher LA. Essence of Anesthesia Practice. WB Saunders 1997

The American Board of Oral and Maxillofacial Surgery 33


2007 Oral and Maxillofacial Surgery Self Assessment Tool (OMSSAT)

32. A 17 year-old asthmatic with a preoperative FEV1/FVC of 85% requires which preoperative
treatment prior to induction of general anesthesia?

A. Ipratropium MDI

B. Nebulized Racemic Epinephrine

C. Advair MDI

D. No treatment indicated

Answer: D

Rationale:
No treatment is required for a normal FEV1/FVC.

Reference:
Firestone, Lebowitz and Cook: Clinical Anesthesia Procedures of the Massachusetts
General Hospital , Third edition, P. 35-7

Barash, Cullen and Stoelting, Handbook of Clinical Anesthesia, Fourth edition, P. 419-22

The American Board of Oral and Maxillofacial Surgery 34


2007 Oral and Maxillofacial Surgery Self Assessment Tool (OMSSAT)

33. A 52 year-old obese patient with a hiatal hernia requires induction of general anesthesia.
Which premedicant could be considered to minimize the patient’s risk of respiratory
problems?

A. Glycopyrolate

B. Amitriptyline

C. Metoclopramide

D. Meperidine

Answer: C

Rationale:
Metoclopramide would increase gastric emptying and increase esophageal sphincter tone.
This would help decrease the risk of aspiration. All the other choices decrease esophageal
sphincter tone and would increase the risk of aspiration.

Reference:
Firestone, Lebowitz and Cook: Clinical Anesthesia Procedures of the Massachusetts
General Hospital , Third edition, P. 624-5

Barash, Cullen and Stoelting, Handbook of Clinical Anesthesia, Fourth edition, P. 549

The American Board of Oral and Maxillofacial Surgery 35


2007 Oral and Maxillofacial Surgery Self Assessment Tool (OMSSAT)

34. Which of the following statements is correct regarding the LMA (laryngeal mask airway?

A. The classic LMA when in correct position protects against aspiration of stomach
contents

B. Insufflation of the stomach can occur with excess pressure exerted through the LMA.

C. Administration of emergency drugs via a LMA or endotracheal tube has equivalent


success rates.

D. A cuffless LMA is available for young children to minimize potential soft tissue
potential ischemia.

Answer: B

Rationale:
The laryngeal mask airway is an airway device that is placed in the hypopharynx above the
opening of the larynx. The LMA does not protect against aspiration of stomach contents
and insufflation of the stomach can be expected with pressures above 20 cm of water. The
black orientation line faces cephalad when in correct position. Delivery of emergency
drugs through a LMA are less successful (approximately 20% successful administration)
than via an endotracheal tube. All sizes of the LMA have a cuff that allows for seating of
the LMA in the hypopharynx.

Reference:
The LMA Manual. The Laryngeal Mask Airway Company, LTD.

Todd DW. Use of the LMA for Outpatient General Anesthetics, Pro. J Oral and
Maxillofac Surg, 61:2003.

The American Board of Oral and Maxillofacial Surgery 36


2007 Oral and Maxillofacial Surgery Self Assessment Tool (OMSSAT)

35. A 7 year-old male needs a general anesthetic and intubation for an elective surgical
procedure. He presents with malaise, a productive cough and thick nasal discharge. How
long will you wait to reschedule the procedure?

A. No delay necessary

B. 7 to 10 days

C. 2 to 3 weeks

D. 1 ½ to 2 months

Answer: D

Rationale:
It is likely that this child has an upper respiratory infection (URI). Signs of an URI include
fever, fatigue, loss of appetite, productive cough, and thick nasal discharge. Children with
URI have an irritable airway and are at increased risk for laryngospasm, bronchospasm,
postintubation croup, pneumonia, and episodes of desaturation. Bronchial hyperreactivity
may last 4 to 6 weeks after an URI; it is recommended to delay treatment for at least 6
weeks. If urgent or emergency surgery is necessary a LMA may reduce complications
associated with an irritable airway.

Reference:
Cote, Charles J., Pediatric Anesthesia, pg. 2381 - 2382 in Miller's Anesthesia, Sixth
Edition, Volume 2, by Elsevier Inc.

Dembo, Jeffrey B., Pediatric Consideration in Office Anesthesia, pg. 108, OMS
Knowledge Update, Vol. 1, Part 1, August 1994

The American Board of Oral and Maxillofacial Surgery 37


2007 Oral and Maxillofacial Surgery Self Assessment Tool (OMSSAT)

36. Which mode of mechanical ventilation is described as allowing the patient to trigger a
breath; but can cause respiratory alkalosis when the patient is tachypneic?

A. Assist control ventilation

B. Synchronized intermittent mandatory ventilation

C. Pressure controlled ventilation

D. Controlled mechanical ventilation

Answer: A

Rationale:
Mechanical ventilation may be either volume or pressure controlled. Volume controlled
ventilation consists of CMV(controlled mechanical ventilation), ACV (assist controlled
ventilation), IMV (intermittent controlled ventilation), SIMV (synchronized intermittent
controlled ventilation). Pressure controlled ventilation is either PSV (pressure support
ventilation) which is patient triggered or PCV (pressure controlled ventilation) which is
ventilator triggered. CMV is most commonly used during general anesthesia in the
neuromuscularly blocked patient. The various other settings can be used for ventilatory
support and weaning. The ACV mode requires that the patient trigger a breath. When the
patient triggers a breath the ventilator delivers a preset volume. The disadvantage of the
mode is that if the patient is tachypneic excessive volume and thus a respiratory alkalosis
will develop. Alternatively, IMV allows the patient to breath spontaneously and delivers
positive pressure ventilation at a preset volume and rate to ensure oxygenation and
ventilation. SIMV prevents PVP (positive pressure ventilation) during a spontaneous
breath.

Reference:
Miller, Stoelting, Basics in Anaesthesia

The American Board of Oral and Maxillofacial Surgery 38


2007 Oral and Maxillofacial Surgery Self Assessment Tool (OMSSAT)

37. A 64 year-old female presents for removal of tooth #30 under general anesthesia. Her past
medical history is significant for rheumatic heart disease, and subsequent mitral valve
stenosis. The preoperative management of this patient would include which of the following?

A. Decrease intravascular volume

B. Increase intravascular volume

C. Maintain a slower heart rate

D. Maintain a faster heart rate

Answer: C

Rationale:
The principal hemodynamic goals of managing patients with mitral stenosis are to
maintain a slower to normal sinus rhythm and to avoid tachycardia, large increases in
cardiac output, and both hypovolemia and fluid overload by judicious fluid therapy.

Reference:
DE Longnecker, FL Murphy, Introduction to Anesthesia, 9th ed., pp. 290, Saunders, 1997

GE Morgan et al, Clinical Anesthesiology, pp. 411, Lange/McGraw-Hill, 2002

The American Board of Oral and Maxillofacial Surgery 39


2007 Oral and Maxillofacial Surgery Self Assessment Tool (OMSSAT)

38. Twenty four hours after an ultralight general anesthesia procedure for third molar removal
using intravenous fentanyl, methohexital, and nasal nitrous oxide/oxygen, your 40 year-old
female patient calls complaining of intense abdominal pain in all quadrants and muscle
weakness. Over the next several days, confusion develops and the urine turns dark. After
successful medical management, the patient relates a family history of “severe reactions” to
sulfonamide antibiotics, including prolonged hospitalizations. Which of the following agents
would be safest to use on this patient in the future?

A. Morphine

B. Ketamine

C. Isoflurane

D. Diazepam

Answer: A

Rationale:
The symptoms are classic for acute intermittent porphyria, an autosomal dominant
condition leading to a deficiency in uroporphyrinogen synthetase activity, causing an
accumulation of uroporphobilogen, which is excreted in the urine turning it a dark color.
Classic symptoms of an acute attack include intense abdominal pain, motor weakness,
(usually starting proximally in the upper limbs) and confusion/agitation. These are due to
nervous system dysfunction and demylenization. Diagnosis is by family history, increased
levels of urinary porphobilogen, and deficient uroporphyrogen synthetase in the red blood
cells. Treatment for an acute episode is supportive, including withdrawal of precipitating
agents, carbohydrate support, hydration, and careful administration of hematin.

Many agents have been implemented as potential triggering agents for acute intermittent
porphyria. The most commonly cited are barbiturates. Morphine, fentanyl and its
conjoiners, and nitrous oxide are considered safe. Other anesthetic agents implicated as
causative or exacerbative agents for AIP include most potent inhalation volatile
anesthetics, benzodiazepines, ketamine, etomidate, meperidine, and lidocaine.

Reference:
Weinberg GL (ed;): Basic Science Review of Anesthesiology, McGraw-Hill, New York,
1997 pp64-66

Benumof JL (ed.) Anesthesia and Uncommon Diseases, WB Saunders, Philadelphia, 1998


pp 162-163

The American Board of Oral and Maxillofacial Surgery 40


2007 Oral and Maxillofacial Surgery Self Assessment Tool (OMSSAT)

39. Thirty minutes after extubating a 6 year old asthmatic male patient in your office you notice
the child to be in respiratory distress characterized by high pitched coarse sounds occurring
during inspiration. What is the most likely diagnosis and treatment?

A. Partial laryngospasm initially managed with positive pressure ventilation

B. Bronchospasm initially managed with albuterol inhaler

C. Postextubation stridor initially managed with racemic epinephrine

D. ost-obstructive pulmonary edema initially managed with supplemental oxygen

Answer: C

Rationale:
Postextubation stridor is a result of laryngeal inflammation reducing the airway
DIMENSION. In a young child it may be secondary to irritation of the endotracheal tube
infringing on the narrowest part of the child's airway – being the cricoid cartilage.
Maneuvers to avoid such include using a cuffless tube and ensuring that there is an air leak
around the tube. Movement of the patient's head during surgery can cause a displacement
of the tube superiorly (essentially extubating the patient) or inferiorly (irritating the
mucosa around the region of the cricoid cartilage). The risk of the child developing
postextubation stridor persists for up to 24 hours after the extubation. Aerosolized 2.25%
racemic epinephrine produces mucosal vasoconstriction and reduces laryngeal edema. The
patient has a history of asthma and could have developed a bronchospastic event post-
operatively. However, the clinic presentation was an inspiratory sound where
bronchospasm would more likely demonstrate an expiratory wheeze. Laryngospasm is also
a possibility but there is nothing in the history to put the patient at risk for a laryngospasm
30 minutes postextubation and it should not be highest on your differential.

Reference:
Altee John L. Complications in Anesthesia. WB Saunders 1999

Roizen MF, Fleisher LA. Essence of Anesthesia Practice. WB Saunders 1997

The American Board of Oral and Maxillofacial Surgery 41


2007 Oral and Maxillofacial Surgery Self Assessment Tool (OMSSAT)

40. Four days after discharge after a bimaxillary osteotomy, a 16 year-old female returns to your
office complaining of malaise. You note considerable pitting edema of the lower extremities
and rales on chest auscultation. Review of the anesthesia record shows approximately 1000
ml of blood loss and induced hypotension to a mean arterial pressure of 60 during the
maxillary downfracture. Surgical time was 7 hours. No blood products were administered.
Which of the following would you expect to find?

A. Serum osmolality 250 mOsm/L

B. Serum creatinine 0.9 mg/dL

C. Intense diruesis

D. Hypokalemia

Answer: A

Rationale:
This patient's clinical presentation is consistent with acute intrinsic renal failure due to
intraoperative hypoperfusion. Renal ischemia brought about by loss of red cell mass and
hypotension in this prolonged procedure caused glomerular and tubular damage. The
classic initial presentation of acute renal failure is oliguria and fluid retention, causing a
decreased serum osmolality (normal = about 275mOsm/L.) Retention of nitrogenous
waste products by lack of renal excretion would cause increases in serum creatinine
(normal = 0.5-1.2 mg//dL.) In the early phase of ARF, typically an oliguric phase occurs
for the first 1 to 2 weeks. If there is a renal recovery, intense dieresis usually follows this.
Early ARF also generally shows metabolic acidosis and subsequent hyperkalemia.

Treatment of ARF includes intense fluid and electrolyte management, avoiding


nephrotoxic medications, dialysis if necessary, and prevention of infection (which is more
common during the diuretic phase of ARF.)

Reference:
Washington Manual of Medical Therapeutics, 30th ed, Liippincott, Philadelphia, 2001
pp258-263

Fauci et al (eds.) Harrison's Principles of Internal Medicine 14th Ed, McGraw-Hill, New
York, 1998 pp1511-1513

The American Board of Oral and Maxillofacial Surgery 42


2007 Oral and Maxillofacial Surgery Self Assessment Tool (OMSSAT)

41. A patient develops STATUS asthmaticus after an anesthetic and requires emergent
endotracheal intubation and mechanical ventilation for respiratory failure. Once transferred
to the ICU, it is noted that your patient has a depressed cardiac output, hypotension and
hypercapnia. What is the likely explanation for this constellation of findings?

A. Secondary bacterial pneumonia

B. Dynamic hyperinflation by intrinsic positive end-expiratory pressure (air trapping)

C. Beta agonist toxicity

D. Inadequate fluid resuscitation

Answer: B

Rationale:
This constellation of findings is seen with intrinsic/auto PEEP due to dynamic airway
collapse (“air trapping”). The FRC is increased by recruiting more alveoli and by
increasing alveolar size. Intrinsic (auto) PEEP occurs when the inspiratory/expiratory time
ratio approaches 1:1. Hyperinflation of the thoracic cavity results in increased intra-
thoracic pressure and subsequently a decrease in cardiac output. Ideally, the I/E ventilator
ratio should be maintained at 1:3 or 1:4 for obstructive pulmonary patients.

Reference:
Ninane V, et al: Intrinsic PEEP in patients with chronic obstructive lung disease: Role of
expiratory muscles. Am Rev Respir Dis 148:1037, 1993.

Alex, CG and Tobin, MJ: Assessment of Pulmonary Function in Critically Ill Patients, in
Textbook of Critical Care, Shoemaker, WC. 4th Edition. W B Saunders Co. 2000.

The American Board of Oral and Maxillofacial Surgery 43


2007 Oral and Maxillofacial Surgery Self Assessment Tool (OMSSAT)

42. While under general anesthesia for a maxillo-mandibular advancement procedure, an


obstructive sleep apnea patient is exhibiting significant hypotension. Pre-operatively, the
patient’s baseline blood pressure was 170/90. On pre-operative physical examination, the
patient had jugular venous distention and a para-sternal PMI. In this circumstance what is the
most appropriate management for hypotension?

A. Intravenous fluid bolus

B. Trendelenberg positioning

C. Intravenous alpha-1 blockade

D. Intravenous vasopressor agent

Answer: D

Rationale:
This OSA patient has physical signs of pulmonary hypertension. The most appropriate
manner to treat anesthetic induced hypotension without exacerbating right heart failure is
an IV vasopressor agent (e.g. phenylephrine). Fluid bolus may exacerbate right heart
failure and positioning has little effect on anesthesia induced hypotension. Alpha blockade
may prevent the physiologic responses during efforts to correct the hypotension.

Reference:
Ronderos, J and Boyd, G: Anesthetic Considerations for Obstructive Sleep Apnea. Oral
and Maxillofacial Clinics of North America: Vol. 7, No. 2 1995,

Hess, ML and Sibbald, WS: Applied Cardiovascular Physiology in the Critically Ill.
Textbook of Critical Care, 4th Edition 2000, Chapter 89: 1001

The American Board of Oral and Maxillofacial Surgery 44


2007 Oral and Maxillofacial Surgery Self Assessment Tool (OMSSAT)

43. A 68 year-old male with stable atherosclerotic heart disease develops significant
hypotension, BP 75/35 with a heart rate of 110 following induction of general anesthesia.
What would be the most appropriate first line pharmacotherapy?

A. Atropine

B. Phenylephrine

C. Ephedrine

D. Epinephrine

Answer: B

Rationale:
Phenylephrine is an alpha agonist. It will increase SVR and may cause a reflex
bradycardia, which would be beneficial for cardiac function.

Ephedrine and epinephrine would both increase heart rate due to their beta stimulation and
given this patients medical history that would not be desirable.

Atropine is an anticholinergic drug, would increase heart rate and compromise the
ischemic heart.

Reference:
Firestone, Lebowitz and Cook: Clinical Anesthesia Procedures of the Massachusetts
General Hospital , Third edition, P. 634-5

Barash, Cullen and Stoelting, Handbook of Clinical Anesthesia, Fourth edition, P.871

The American Board of Oral and Maxillofacial Surgery 45


2007 Oral and Maxillofacial Surgery Self Assessment Tool (OMSSAT)

44. Initial acute management of intraoperative bronchospasm under inhalational general


anesthesia includes which of the following?

A. Decadron IV

B. Reducing the depth of anesthesia

C. Albuterol MDI through the endotrachial tube

D. Epinephrine IV

Answer: C

Rationale:
Intraoperative bronchospasm should be first treated by confirming that there is no
mechanical obstruction of the tracheal tube, tube placement is correct and adequate depth
of anesthesia is present. The initial treatment consists of the administration of a -agonist
such as albuterol. Epinephrine should be reserved for a severe bronchospasm refractory to
initial -agonist therapy because of the potential for severe adverse effects. Steroids are
not helpful in the acute management and muscle paralysis will not improve the situation.
Differential diagnosis should also include pneumothorax, pulmonary edema, pulmonary
embolus, and pulmonary aspiration.

Reference:
Stoelting RK, Dierdorf SF. Anesthesia and Co-Existing Disease, Churchill Livingstone,
2002.

AAOMS Office Emergency Manual, AAOMS, Chicago 2004.

The American Board of Oral and Maxillofacial Surgery 46


2007 Oral and Maxillofacial Surgery Self Assessment Tool (OMSSAT)

45. The patient is a 72-year old male who lives alone and has recently started to exhibit signs of
memory loss. He is otherwise healthy except for multiple medications including:
levothyroxine (thyroid replacement), vytorin (hypercholesterelemia) and hydrochlorothiazide
(diuretic). He is cleared for surgery and anesthesia for multiple extractions and mandibular
tori removal. Intraoperatively, he develops a significant tachycardia and severe hypotension.
A fluid challenge and reduction in anesthetic agents do not alter these conditions. The most
likely diagnosis is:

A. fluid overload.

B. malignant hyperthermia.

C. thyrotoxicosis factitia.

D. pheochromocytoma.

Answer: C

Rationale:
Thyroid storm is an abrupt exacerbation of hyperthyroidism caused by sudden release of
thyroid hormone (T4) into circulation by accident or an otherwise clinical presentation.
Clinical symptoms mimic malignant hyperthermia. There is tachycardia, dehydration,
hyperthermia and shock. This would be high on the list of possible causes due to the
sudden loss of memory, the use of multiple medications and the solitary situation.

Reference:
Stoelting, R.K. and Dierdorf, S.F: Anesthesia and Co-Existing Disease. 4th Edition, 2002.
Philadelphia, Churchill Livingstone. Pp. 415-7

Beers, M. and Berkow, R.: The Merck Manual. 17th Edition, 1999. Whitehouse Station,
Merck Research Laboratories. P. 89.

Cole, D.J. and Schlunt, M.: Adult Perioperative Anesthesia 2004. Philadelphia, Mosby
pp.71-73

The American Board of Oral and Maxillofacial Surgery 47


2007 Oral and Maxillofacial Surgery Self Assessment Tool (OMSSAT)

46. In children, blood pressure is determined almost entirely by which of one of the following
physiologic parameters?

A. Stoke Volume

B. Myocardial Contractility

C. Systemic Vascular Resistance

D. Heart Rate

Answer: D

Rationale:
A child's required cardiac output can be double that of an adult; it is related to an increased
metabolic rate and increased oxygen consumption. Blood pressure is dependent on stroke
volume, myocardial contractility, systemic vascular resistance and heart rate. In the child,
stroke volume, myocardial contractility, or systemic vascular resistance are relatively
invariable. Blood pressure is almost entirely rate dependent. Since children show a
tendency toward vagal response to many stimuli; monitoring of the heart rate is critical.
Bradycardia in children invariably leads to hypotension.

Reference:
Dembo, Jeffrey B., Pediatric Anesthesia pg.837 in Oral & Maxillofacial Surgery Clinics of
North America. W.B. Saunders Co., Nov. 1992.

Kelly, John P., Dembo, Jeffrey,B ., in Anesthesia Section, OMS Know ledge Update, Vol.
1 Part 1, August 1994, Pg. 42

The American Board of Oral and Maxillofacial Surgery 48


2007 Oral and Maxillofacial Surgery Self Assessment Tool (OMSSAT)

47. In the infant and young child, the narrowest portion of the larynx is located at which
anatomical position?

A. Vocal cords

B. Cricoid cartilage

C. Thyroid cartilage

D. Valecula

Answer: B

Rationale:
In infants or young children, the narrowest portion of the larynx is at the cricoid cartilage.
In a child, an endotracheal tube might pass easily through the vocal cords but not through
the subglottic region. The cricoid is the only complete ring of cartilage in the
laryngotracheobronchial tree and is therefore nonexepandable. A tight fitting endotracheal
tube that compresses the tracheal mucosa at this level may cause edema and result in
increased airway resistance at the time of extubation. For this reason, uncuffed
endotracheal tubes are usually preferred for infants or young children. As the child
matures (age 10 to 12 years of age), the cricoid and thyroid cartilages have grown,
eliminating the narrowing of the subglottic area and angulation of the vocal cords.

Reference:
Cote, Charles J., Pediatric Anesthesia, pg. 2376 in Miller's Anesthesia, Sixth Edition,
Volume 2, by Elsevier Inc.

The American Board of Oral and Maxillofacial Surgery 49


2007 Oral and Maxillofacial Surgery Self Assessment Tool (OMSSAT)

48. While under anesthesia for an iliac crest bone graft a patient is noted to become suddenly
tachycardic, hypotensive and hypoxic. The capnography reveals decreasing end-tidal
CO2.The procedure is terminated. A 12-lead EKG is obtained that reveals new findings of
right-axis deviation, peaked T-waves and an incomplete right bundle-branch block. The most
likely diagnosis for this situation is:

A. right to left cardiac shunting.

B. malignant hyperthermia.

C. pulmonary embolism.

D. pulmonary edema secondary to left ventricular failure.

Answer: C

Rationale:
Massive pulmonary embolism/ cor pulmonale are associated with right ventricular failure,
arterial hypoxemia and decreasing end-tidal CO2. The V/Q mismatch results in areas of
wasted pulmonary ventilation. Conversely, a right to left shunt is an extreme V/Q
abnormality when there is perfusion and no ventilation at all. A decrease in CO2 on intra-
operative capnography suggests decreased lung perfusion. The EKG will rule out acute
AMI and show right heart strain.

Reference:
Dehring, DJ, Arens JF: Pulmonary Thromboembolism: Disease recognition and patient
management. Anesthesiology 73:146, 1990

Divekan, VM et al. Pulmonary embolism during anesthesia: Case Report. Can Anesthes
Soc J 28:277, 1981

The American Board of Oral and Maxillofacial Surgery 50


2007 Oral and Maxillofacial Surgery Self Assessment Tool (OMSSAT)

49. Which of the following is contraindicated in management of cardiac arrhythmias during a


malignant hyperthermia episode?

A. Procainamide

B. Cardizem

C. Regular insulin and D50

D. Sodium bicarbonate

Answer: B

Rationale:
Etiologic treatment of an MH episode requires dantrolene at an initial dose of 2-3 mg/kg.
Cardiac dysrrhythmias should be treated by procainamide at an initial dose of 100mg IV.
Management of hyperkalemia to correct the arrhythmia can be achieved with regular
insulin 10 units IV together with 1 amp of D50, as well as sodium bicarbonate to help
drive potassium intracellularly. Calcium channel blockers such as cardizem are
contraindicated because they can cause severe myocardial depression in the presence of
dantrolene.

Reference:
MHAUS, Emergency Therapy for MH Crisis
Stoelting RK, Dierdorf SF. Anesthesia and Coexisting Disease, Churchill- Livingston,
2002.

The American Board of Oral and Maxillofacial Surgery 51


2007 Oral and Maxillofacial Surgery Self Assessment Tool (OMSSAT)

50. In a patient who undergoes mandibular advancement, the ratio of soft tissue changes to hard
tissue changes at pogonion would most likely be which of the following?

A. 1:1

B. 1:2

C. 2:3

D. 1:4

Answer: A

Rationale:
During mandibular orthognathic surgery, the overlying soft tissues follow osseous changes
closely. Generally, the soft tissues follow bony movements in a 1:1 ratio. The exception
is the lower lip which relies heavily on the position of the incisor teeth and can also be
affected by the position of the upper lip.

Reference:
Betts NJ, Dowd KF. Soft tissue changes associated with orthognathic surgery. Atlas Oral
Maxillofac Surg Clin North Am 8: 13-38, 2000.

The American Board of Oral and Maxillofacial Surgery 52


2007 Oral and Maxillofacial Surgery Self Assessment Tool (OMSSAT)

51. In a distraction osteogenesis procedure, the consolidation phase of treatment extends until
which of the following stages of bone healing?

A. Hematoma formation and inflammation

B. Soft callus formation

C. Hard callus formation

D. Bony maturation

Answer: C

Rationale:
Bony healing after a fracture or osteotomy consists of four histologically distinct stages: an
inflammatory phase, soft callus formation, hard callus formation, and bony
maturation/remodeling. During soft callus formation, fibrovascular structures bridge the
osteotomized bone segments and there is recruitment of fibroblasts and mesenchymal stem
cells within the fracture zone. It is this flexible soft callus which is lengthened via gradual
traction during the subsequent distraction phase. Once the desired bony lengthening has
been achieved, the distraction appliance is left in place and serves as a stabilizing device
while the regenerate bone (soft callus) matures and calcifies forming a hard callus. Most
distraction protocols recommend a consolidation/healing phase of at least 6 to 8 weeks.

Reference:
Crago C.A., Proffit W.R., and Ruiz R.L. Maxillofacial Distraction Osteogenesis. Pp. 357-
393. In: Proffit W.R., White R.P., and Sarver D.M. (Eds) Contemporary Treatment of
Dentofacial Deformity. Philadelphia. Mosby. 2003.

The American Board of Oral and Maxillofacial Surgery 53


2007 Oral and Maxillofacial Surgery Self Assessment Tool (OMSSAT)

52. A patient with an underlying diagnosis of Hemifacial Microsomia presents with severe
unilateral hypoplasia of the ascending ramus, condyle, and glenoid fossa. There is little to no
translational movement of the affected condyle. This mandibular-TMJ deformity is
consistent with which skeletal type hemifacial microsomia?

A. Type I

B. Type IIa

C. Type IIb

D. Type III

Answer: C

Rationale:
Patients with Hemifacial Microsomia (oculoauriculovertebral spectrum) will present with
varying degrees of mandibular and temporomandibular joint hypoplasia as a primary
finding in the condition. Kaban has refined a classification system useful in defining the
degree of mandibular deformity in these patients. The specific type of mandibular/TMJ
deformity present is a critical factor in deciding upon the specific reconstructive techniques
that will be employed. The following is a brief synopsis of each Kaban type:
Type I: All skeletal components (glenoid fossa, condyle, ascending ramus) are present with
a mild degree of hypoplasia. Normal function is present.
Type IIa: All of the skeletal components demonstrate a moderate degree of hypoplasia.
While present, the condyle may appear to be malpositioned so that it is anterior and medial
to the contralateral (normal) side. Function is affected, but remains satisfactory.
Type IIb: There is moderate to severe hypoplasia of the glenoid fossa and the condyle-
ramus complex. Despite an abnormal and severely hypoplastic condyle, many patients
will still have a working joint and an actual “stop” where the condylar segment seats
against the skull base. Most patients will demonstrate function that is limited to simple
rotation of the condyle without any translational movements.
Type III: This is the most severe form of mandibular hypoplasia with complete absence
of the condyle-ramus complex and the glenoid fossa. The affected side of the mandible
has no working articulation against the skull base.

Reference:
Kaban LB, Moses MH, Mulliken JB. Correction of hemifacial microsomia in the growing
child: A follow-up study. Cleft Palate J 23 (Suppl 1) 50, 1986.
Kaban LB. Congenital Abnormalities of the Temporomandibular Joint. In: Kaban LB and
Troulis MJ (Eds). Pediatric Oral and Maxillofacial Surgery. Pp. 302-339. Elsevier,
Philadelphia, 2004.

The American Board of Oral and Maxillofacial Surgery 54


2007 Oral and Maxillofacial Surgery Self Assessment Tool (OMSSAT)

53. What musculature must be re-approximated as a distinct layer during soft tissue closure
following a transoral genioplasty procedure?

A. Genioglossus

B. Mentalis

C. Digastric

D. Platysma

Answer: B

Rationale:
When a transoral approach to the bony chin is utilized, the mentalis musculature is
routinely disrupted. These are paired muscles that insert on each side of the bony midline
and are separated by a small fat pad. During closure, adequate repair of the mentalis
muscles is critical in order to avoid a “witches chin” deformity.

Reference:
Posnick JC. Craniofacial and Maxillofacial Surgery in Children and Young Adults. Ed. J.
C. Posnick. W. B Saunders, Philadelphia.

The American Board of Oral and Maxillofacial Surgery 55


2007 Oral and Maxillofacial Surgery Self Assessment Tool (OMSSAT)

54. Which of the following is a consideration when performing sagittal split osteotomy for
mandibular advancement in children compared to adults?

A. The lingula and inferior alveolar foramina are located in a more inferior and anterior

position within the ramus.

B. The sagittal bone cuts should be positioned more medial.

C. The propensity for “greenstick” fracture of the inferior border of the mandible is greater.

D. Simultaneous removal of partially developed third molar teeth is not possible.

Answer: C

Rationale:
The lingula and inferior alveolar foramina are located in a more superior and posterior
position in this age group. This has technical implications in determining the vertical
placement of the medial osteotomy of the ramus. If the medial bone cut is positioned high
on the ramus, then injury to the nerve is avoided, but the risk of unfavorable split (i.e.
buccal plate fracture) increases. In children, the sagittal component of the osteotomy
design should be placed as far laterally as possible in order to avoid injury to the
developing teeth. Children will have a higher propensity for “greenstick” fracture along
the inferior border of the mandible. Their bone is more cancellous in nature and this often
results in a longer area of fracture along the inferior border of the mandible. Developing
third molars can be removed after the mandibular ramus has been split. In cases where the
third molar teeth are only partially developed, they can still be enucleated while the
proximal and distal segments are separated

Reference:
Bell WH: Mandibular Advancement in Children Special Considerations. In Bell WH,
Modern Practice in Orthognathic and Reconstructive Surgery. Vol. 3, 1992, WB Saunders.
pp. 2516

The American Board of Oral and Maxillofacial Surgery 56


2007 Oral and Maxillofacial Surgery Self Assessment Tool (OMSSAT)

55. Which type of osteotomy procedure allows only a limited amount of mandibular
advancement?

A. Bilateral saggittal split osteotomies

B. Inverted “L” osteotomies with bone grafts

C. Distraction osteogenesis after complete corticotomies

D. Trans-oral vertical ramus osteotomies

Answer: D

Rationale:
If any significant forward movement of the mandible is undertaken using trans-oral vertical
ramus osteotomies, a large gap between the proximal and distal segments will result. The
clinical applications of vertical ramus osteotomy procedures are limited primarily to
mandibular set-back procedures and correction of relatively minor lower facial
asymmetries. All other choices are reasonable approaches to a large mandibular
advancement.

Reference:
Bell WH, Proffit WR, White RP, (eds.) Surgical correction of dentofacial deformities.
Philadelphia: WB Saunders; 1980.

The American Board of Oral and Maxillofacial Surgery 57


2007 Oral and Maxillofacial Surgery Self Assessment Tool (OMSSAT)

56. A 7 year-old patient with Hemifacial Microsomia demonstrates complete absence of the
ipsilateral condyle and the entire ascending ramus. There is a severe deficiency of posterior
facial height on the affected side and a large maxillary cant. The patient has undergone an
initial phase of interceptive orthodontic treatment and is ready for surgery. Which of the
following is the most predictable surgical treatment?

A. Distraction osteogenesis of the affected side of the mandible

B. Bilateral distraction osteogenesis of the mandible

C. Maxillary osteotomy for leveling of the occlusal plane

D. Costochondral graft for mandibular reconstruction

Answer: D

Rationale:
The patient described in this question has a Kaban Type III mandibular deformity with the
complete absence of the condyle-ramus complex and glenoid fossa. The reconstructive
approach for these patients consists of an initial phase of mandibular reconstruction using a
costchondral graft for the construction of missing skeletal components (e.g. condyle-
ascending ramus). This is typically carried out at approximately ages 6 to 12 years. A
second stage of surgery is undertaken as the patient approaches skeletal maturity and
typically consists of maxillomandibular surgery in order to normalize the occlusion and
position of the lower facial skeleton.

Often, placement of a rib graft on the affected side is combined with a contralateral sagittal
split osteotomy on the unaffected side. This allows for better three-dimensional
repositioning of the distal segment of the mandible. At the same time, a posterior open
bite on the affected side is created and maintained during the postoperative healing period
with the use of an occlusal splint. This occlusal splint is then gradually reduced in order to
facilitate dentoalveolar development of the maxilla on the affected side.

Distraction osteogenesis is a technique that allows for the lengthening of a skeletal part. In
the type III deformity, there is no role for distraction osteogenesis since the condyle-ramus
complex is congenitally missing and these anatomic parts can not be distracted into
existence.

Reference:
Posnick JC. Hemifacial Microsomia: Evaluation and Treatment. In: Posnick JC (Ed).
Craniofacial and Maxillofacial Surgery in Children and Young Adults. Pp: 419-445. W.B.
Saunders. Philadelphia, 2000.

The American Board of Oral and Maxillofacial Surgery 58


2007 Oral and Maxillofacial Surgery Self Assessment Tool (OMSSAT)

57. An 3 year-old undergoes bilateral mandibular osteotomies with application of internal


distraction appliances. Surgery is followed by a 2 day latency phase. The distraction
appliances are then activated 0.75 mm four times daily for a total of 8 days and a
consolidation phase of 8 weeks follows. Upon removal of the distraction appliances,
bilateral mandibular nonunions are noted. Which of the following is the most likely cause of
this complication?

A. Inadequate length of time for the latency phase

B. The rate and rhythm of the distraction phase

C. Inadequate rigidity of the device during consolidation phase

D. Excessive soft tissue resistance during the advancement

Answer: B

Rationale:
In the above example, the most likely cause of the nonunions is the rapid rate of
lengthening during the distraction phase. Most distraction protocols advocate a maximum
daily rate of distraction of 1 mm total. In addition, many authors advocate a “rhythm” of
distraction whereby the 1 mm total movement is divided into four incremental movements
of 0.25 mm. This allows for gradual lengthening of the bony segments while avoiding
excessive disruption of the bony regenerate (soft callus) and reduces soft tissue pain
associated with activation of the device. In the above example, the patient's rate of
distraction was approximately 3 mm per day and a large movement (24 mm) resulted in a
relatively short period of 8 days. Excessive lengthening over a short period may cause
stretching of the soft callus with a thinned, hour-glass shaped bony regenerate or a fibrous
nonunion.
Although the duration of the latency phase (2 days) described is shorter than is typical, the
likelihood that it would have a negative consequence in a young child is very low.
Average latency phase length is usually 5 to 7 days and allows for adequate formation of a
soft callus before active distraction is initiated. In young children, bone healing occurs
much faster and little or no latency phase is required.

Reference:
Crago C.A., Proffit W.R., and Ruiz R.L. Maxillofacial Distraction Osteogenesis. Pp. 357-
393. In: Proffit W.R., White R.P., and Sarver D.M. (Eds), Contemporary Treatment of
Dentofacial Deformity. Philadelphia. Mosby. 2003.

The American Board of Oral and Maxillofacial Surgery 59


2007 Oral and Maxillofacial Surgery Self Assessment Tool (OMSSAT)

58. A patient undergoes bilateral sagittal split osteotomies for mandibular advancement and the
distal segment if placed into the final surgical splint. After elimination of bony interferences,
a bony gap remains between the anterior extent of the proximal and distal segments of the
mandible. If lag-screw fixation is applied in this area, which of the following will occur?

A. There will be anterior displacement of the distal segment.

B. An anterior open bite will result

C. The mandibular condyles will be displaced medially

D. The mandibular condyles will be displaced laterally

Answer: D

Rationale:
Changes in intercondylar width may occur as a result of sagittal split osteotomy. As the
distal segment is advanced, there is often the creation of a gap at the anterior-superior
extent of the osteotomy site. If the segments are compressed together at the time of rigid
fixation placement, there is a fulcrum based on the distal segment and lateral displacement
of the condyle. Several techniques have been described in order to avoid unfavorable
condylar displacement. These include the use of bicortical position screws, placement of
bone shim between the proximal and distal segments, recontouring of the proximal/lingual
aspect of the distal segment, and addition of a midline/symphyseal osteotomy. Use of lag
screw fixation will compress the segments and actually cause lateral condylar
displacement. Alternatively, lag screws may be utilized only when at least one position
screw is placed first in order to maintain the anterior osteotomy gap.

Reference:
Tucker M.R., Frost D.E., and Terry B.C. Mandibular Surgery. In: Tucker M.R., Terry
B.C., White R.P., and VanSickels J.E. (Eds), Rigid Fixation for Maxillofacial Surgery.
J.B. Lippincott, New York, Pp. 251-295, 1991.

The American Board of Oral and Maxillofacial Surgery 60


2007 Oral and Maxillofacial Surgery Self Assessment Tool (OMSSAT)

59. A patient with unilateral failure of hardware (loss of fixation) one week following bilateral
sagittal split osteotomies of the mandible may present with which of the following physical
findings?

A. Unilateral open bite

B. Coincident maxillary and mandibular midlines

C. Deviation of the mandible on extreme opening

D. Increased lower facial height

Answer: A

Rationale:
Loss of fixation on one side of a bilateral sagittal split osteotomy typically causes a
contralateral open bite, deviation of the mandible towards the side of the hardware failure,
and lower facial asymmetry. The open bite occurs typically occurs as ramus height is lost
and an early occlusal contact occurs. Midline shift occurs with loss of sagittal projection
after fixation loss.

Reference:
Kaban L, et al. Complications in Oral and Maxillofacial Surgery. Complications of
Orthognathic Surgery. WB Saunders; Philadelphia, Pennsylvania.

The American Board of Oral and Maxillofacial Surgery 61


2007 Oral and Maxillofacial Surgery Self Assessment Tool (OMSSAT)

60. Which of the following statements about nasal changes following Le Fort I osteotomy with
maxillary advancement and impaction is true?

A. Nasolabial angle will increase

B. Alar base width will decrease

C. Dorsal hump will be less noticeable

D. Nasal tip rotation will be unaffected

Answer: C

Rationale:
Movement of the maxilla directly affects the lower nasal anatomy and the nasal septum.
When the maxilla is advanced and/or superiorly repositioned, the alar base width will
increase, nasolabial angle will decrease, and increased superior rotation of the nasal tip is
noted. As a result of these changes, the prominence of any dorsal hump will be reduced by
the surrounding changes of the lower nasal dorsum.

Reference:
O'Ryan F, Schendel S. Nasal anatomy and maxillary surgery III. Surgical techniques for
correction of nasal deformities in patients undergoing maxillary surgery. Int J Adult Ortho
Orthogn Surg 4;157, 1989.

The American Board of Oral and Maxillofacial Surgery 62


2007 Oral and Maxillofacial Surgery Self Assessment Tool (OMSSAT)

61. Which of the following statements is correct regarding the use of distraction osteogenesis
compared to conventional orthognathic surgical techniques for maxillary advancements of 5
millimeters or less?

A. Sagittal stability has been proven to be superior

B. Vertical stability has been proven to be superior

C. Perioperative morbidity is less

D. Long-term stability is similar

Answer: D

Rationale:
Distraction osteogenesis has not been proven to provide significant advantages when one
looks at post-operative stability. Similar results have been seen when compared with
studies done after traditional orthognathic surgery. There is no difference in orthodontic
finishing requirements, and additional compensation may be required when compared with
traditional techniques. During the consolidation phase, appliances (either internal or
external) are kept in place for several months.

Reference:
Turvey TA, Ruiz RL, and Costello BJ. Surgical Correction of Midface Deficiency in Cleft
Lip and Palate Malformation. Oral and Maxillofacial Surgery Clinics of North America:
Secondary Cleft Surgery. Philadelphia: W.B. Saunders, 14: 491-507, 2002

The American Board of Oral and Maxillofacial Surgery 63


2007 Oral and Maxillofacial Surgery Self Assessment Tool (OMSSAT)

62. Which statement regarding surgically assisted rapid palatal expansion (SARPE) is correct?

A. Complete downfracture of the maxilla is required in order to realize adequate maxillary


mobility.

B. The procedure creates more expansion within the anterior maxilla than the posterior
maxilla.

C. The procedure is indicated when the amount of expansion necessary is less than 6 mm
total.

D. The posterior maxilla may be adequately freed without the need for osteotomy in the
lateral buttress region.

Answer: B

Rationale:
During surgically assisted rapid palatal expansion (SARPE), there is greater widening
within the anterior maxilla than there is posteriorly. By contrast, a segmental osteotomy
will produce more expansion posteriorly than anteriorly. This is the result of each
procedure having a distinctive “hinge” pattern during the actual maxillary widening.
The SARPE procedure is reserved for cases in which greater than 8 to 10 mm of widening
is necessary. If segmental osteotomy is undertaken for widening of greater than 10 mm,
the likelihood of soft tissue dehiscence, periodontal defects, and poor long term stability
increases.
The surgical approach requires that osteotomies be created at all of the points where
potential resistance to lateral expansion will be encountered. These include the palatal
suture, lateral buttresses, and pterygomaxillary junction. Complete maxillary downfracture
is not required.

Reference:
Bailey LJ. Segmental Le Fort I osteotomy to effect palatal expansion. J Oral Maxillofac
Surg 55:728-731, 1997.

The American Board of Oral and Maxillofacial Surgery 64


2007 Oral and Maxillofacial Surgery Self Assessment Tool (OMSSAT)

63. At the time of a LeFort I osteotomy, the width of the alar base of the nose can be controlled
by which of the following?

A. Augmenting the piriform rim

B. Contouring the lateral extent of the alar cartilages

C. Performing a V-Y closure of the upper lip

D. Pexy of the transverse nasalis muscle

Answer: D

Rationale:
Widening of the nasal base occurs anytime the maxilla is repositioned anteriorly or
superiorly. In order to avoid undesirable nasal widening, a cinch-suture is passed through
the transverse nasalis muscle on each side of the alar base of the nose and tied. Some
surgeons prefer to pass this suture through the caudal edge of the nasal septum or anterior
nasal spine.

Reference:
Schendel SA, Williamson L. Muscle reorientation following superior repositioning of the
maxilla. J Oral Maxillofac Surg 41: 235-40, 1983.

The American Board of Oral and Maxillofacial Surgery 65


2007 Oral and Maxillofacial Surgery Self Assessment Tool (OMSSAT)

64. The Steiner analysis is useful in evaluating which of the following?

A. Upper and lower incisor position

B. The degree of lip incompetence

C. Lower facial height to width ratio

D. Vertical maxillary position

Answer: A

Rationale:
The Steiner analysis is a cephalometric analysis useful in evaluating the degree of
maxillary and mandibular incisor protrusion relative to the surrounding skeletal structures.
The position of the maxillary and mandibular incisors is related to Nasion-A point (N-A)
and Nasion-B point (N-B) lines using both angular and linear measurements.

Reference:
Proffit W.R., Sarver D.M. Diagnosis: Gathering and Organizing the Appropriate
Information. Pp. 127-170. In: Proffit W.R., White R.P., and Sarver D.M. (Eds)
Contemporary Treatment of Dentofacial Deformity. Philadelphia. Mosby. 2003.

The American Board of Oral and Maxillofacial Surgery 66


2007 Oral and Maxillofacial Surgery Self Assessment Tool (OMSSAT)

65. Which of the following two-jaw orthognathic surgical procedures demonstrates the greatest
long-term skeletal stability?

A. Maxillary impaction with mandibular advancement

B. Maxillary advancement with mandibular advancement

C. Maxillary downgraft with mandibular setback

D. Maxillary transverse widening with mandibular setback

Answer: A

Rationale:
Long-term data on skeletal stability following orthognathic surgery suggests that some
procedures are more predictable than others. Overall, surgical procedures aimed at the
correction of Class II problems tend to be extremely stable. These include mandibular
advancement, superior and/or anterior maxillary repositioning, and combinations of the
two. Skeletal stability following mandibular setback and combined procedures for the
correction of Class III situations are less reliable. Treatment plans that include the
transverse widening of the maxilla are the most problematic in terms of long term stability.

Reference:
Proffit WR, Turvey TA, Phillips C. Orthognathic Surgery: a hierarchy of stability. Int J
Adult Orthod Orthogn Surg 11: 191-204, 1996.

The American Board of Oral and Maxillofacial Surgery 67


2007 Oral and Maxillofacial Surgery Self Assessment Tool (OMSSAT)

66. A 48 year-old patient presents with obstructive sleep apnea and a respiratory disturbance
index (RDI) of 110. Diagnostic work-up is undertaken and reveals that the patient has signs
of pulmonary hypertension, intermittent ventricular ectopy, and moderate to severe
congestive heart failure (cor pulmonale). Which of the following is the most appropriate
initial surgical treatment?

A. Septoplasty, turbinectomy

B. Uvulopalatopharyngoplasty

C. Tracheostomy

D. Maxillo-mandibular advancement

Answer: C

Rationale:
Tracheostomy remains an extremely effective treatment modality in patients with
obstructive sleep apnea because it completely bypasses all of the levels of obstruction
within the upper airway. Although permanent tracheostomy is an unattractive alterative
for patients, its use as a temporary measure can produce marked and rapid improvements
in sleep architecture and oxygenation.

In the above clinical situation, the primary indication for tracheostomy as an initial therapy
is the patient's strikingly abnormal RDI and the presence of severe cardio-pulmonary
consequences associated with OSA. The presence of life-threatening cardiac conditions
requires that the upper airway be bypassed. In this example, tracheostomy may be utilized
as an interim treatment until other procedures (e.g. maxillomandibular advancement) may
be carried out in order to improve the patient's upper airway.

Reference:
Tiner BD, Waite PD. Surgical and Nonsurgical Management of Obstructive Sleep Apnea.
In: Miloro M, Ghali GE, Larsen PE, and Waite PD (Editors), Peterson's Principles of Oral
and Maxillofacial Surgery, Second Edition. Pp. 1297-1313. BC Decker, Hamilton, 2004.

The American Board of Oral and Maxillofacial Surgery 68


2007 Oral and Maxillofacial Surgery Self Assessment Tool (OMSSAT)

67. Which of the following statements about cleft lip and palate is correct?

A. Overall prevalence is approximately 1 in 4000 live births.

B. Orofacial clefting rates are lowest in Native-Americans.

C. Isolated cleft palate is more common than cleft lip.

D. A syndromic diagnosis is more common with isolated cleft palate.

Answer: D

Rationale:
Cleft lip with or without cleft palate is a common congenital malformation with an
incidence of approximately 1 in 700 live births, but significant variation is encountered
when different ethnic/racial populations are examined. African Americans have an
incidence which is significantly lower than the general population while Asians and
Native-Americans have the highest rates of birth prevalence. By contrast, isolated cleft
palate has a lower overall incidence of approximately 1 in 2,000 live births with similar
distribution among the different racial and ethnic populations. Of note is that children with
isolated cleft palate are almost five-times more likely to also have some other underlying
syndromic condition.

Reference:
Costello BJ, Ruiz RL. Cleft Lip and Palate: Comprehensive Treatment Planning and
Primary Repair. In: Miloro M, Ghali GE, Larsen PE, and Waite PD (Editors): Peterson's
Principles of Oral and Maxillofacial Surgery, Second Edition. BC Decker. Hamilton. 2004.
Pp: 839-858.

The American Board of Oral and Maxillofacial Surgery 69


2007 Oral and Maxillofacial Surgery Self Assessment Tool (OMSSAT)

68. The superiorly based pharyngeal myomucosal flap contains a portion of what muscle?

A. Superior constrictor

B. Palatopharyngeus

C. Palatoglossus

D. Tensor veli palatini

Answer: A

Rationale:
The pharyngeal flap is fashioned on the posterior pharyngeal wall by incising through
mucosa and bluntly dissecting through superior constrictor until prevertebral fascia is
identified. A superior pedicle is maintained for this finger-like, random pattern flap that is
then sewn into the soft palate.

Reference:
Costello BJ, Ruiz RL. Reconstruction of Cleft Lip and Palate: Secondary Procedures. In
Peterson's Principles of Oral and Maxillofacial Surgery, ed 2. Miloro M, Larsen P, Ghali
GE, and Waite P eds. Hamilton: Decker 2004:871-886.

The American Board of Oral and Maxillofacial Surgery 70


2007 Oral and Maxillofacial Surgery Self Assessment Tool (OMSSAT)

69. The diagnosis of velopharyngeal insufficiency is best established by which of the following?

A. Perceptual evaluation by a speech language pathologist

B. Fiberoptic nasopharyngoscopy

C. Videofluoroscopy

D. Nasometry

Answer: A

Rationale:
The determination of the presence or absence of velopharyngeal dysfunction is made
during the perceptual speech evaluation performed by a speech language pathologist.
During this examination, the speech pathologist relies on auditory and visual input
perceived during spontaneous speech and during the production of standard phrases/words
designed to test velopharyngeal function.

Nasopharyngoscopy and videofluoroscopy are instruments employed to further study


individuals diagnosed with velopharyngeal dysfunction. These provide information
regarding the cause and the magnitude of the dysfunction. The information gained therein
may be used to select subsequent therapy and tailor it according to each individual's
functional anatomy.

Nasometry is a physiologic assessment tool that provides indirect, quantitative


measurements of nasal, relative to oral, acoustic energy transmission. Its utility is in
helping quantify dysfunction and perhaps as a biorationale device for training proper
velopharyngeal function.

Reference:
Marsh JL. The evaluation and management of velopharyngeal dysfunction. Clin Plast Surg
2004;31:261-269.

The American Board of Oral and Maxillofacial Surgery 71


2007 Oral and Maxillofacial Surgery Self Assessment Tool (OMSSAT)

70. What physical finding should alert the surgeon to the possibility of a submucous cleft palate?

A. Alveolar notch

B. Nasolabial fistula

C. Enlarged adenoid pad

D. Bifid uvula

Answer: D

Rationale:
As described by Calnan, the classic clinical findings seen when a patient has a submucous
cleft palate are a triad of bifid uvula, hard palate bony notch, and separation along the
median raphe of the soft palate especially during elevation of the velum.

When a submucous cleft palate is present, the levator muscle is clefted and inserts
abnormally into the posterior edge of the hard palate. The primary functional concern
related to submucous cleft palate is the possibility that the patient will develop
velopharyngeal insufficiency and hypernasal speech as with other cleft palate patients.

Reference:
Calnan, J. Submucous cleft palate. Br J Plast Surg. 1954; 6:264-82.

Ruiz RL, Costello BJ. Reconstruction of Cleft Lip and Palte: Secondary Procedures. In:
Miloro M, Ghali GE, Larsen PE, and Waite PD (Editors): Peterson's Principles of Oral and
Maxillofacial Surgery, Second Edition. Pp: 871-886. BC Decker. Hamilton, Ontario,
2004.

The American Board of Oral and Maxillofacial Surgery 72


2007 Oral and Maxillofacial Surgery Self Assessment Tool (OMSSAT)

71. The repair of a submucous cleft palate is indicated:

A. after 3 or more episodes of otitis media during a 6 month period.

B. at the time of the diagnosis.

C. only in the presence of velopharyngeal dysfunction.

D. in the presence of obstructive sleep apnea.

Answer: C

Rationale:
The lone indication for repair of a submucous cleft palate is the presence of velopharyngeal
dysfunction. While middle ear disease has long been associated with clefting of the
secondary palate it's incidence in the submucous cleft palate population may only be
slightly higher compared with t he general population. Otitis media with effusion is
managed with myringotomy and tube placement

It has been estimated that only 11% of children diagnosed with a submucous cleft will go
on to develop velopharyngeal dysfunction. As such, repair at the time of diagnosis would
represent overtreatment.

There is no direct association of submucous cleft palate and obstructive sleep apnea.

Reference:
Gosain AK, Conley SF, Marks S, Larson DL. Submucous Cleft Palate: Diagnostic
Methods and Outcomes of Surgical Treatment.

The American Board of Oral and Maxillofacial Surgery 73


2007 Oral and Maxillofacial Surgery Self Assessment Tool (OMSSAT)

72. Phase I orthodontic treatment in the patient with a bilateral cleft lip and palate prior to
maxillary bone graft surgery is primarily intended to:

A. collapse the cleft segments in order to obtain a class I molar relationship.

B. correct transverse maxillary dimension and align the pre-maxilla for symmetry.

C. expedite the exfoliation of remaining anterior primary teeth.

D. expand the cleft segments until a Class I molar relationship is obtained.

Answer: B

Rationale:
Surgical reconstruction of the child with cleft lip and palate requires familiarity with
orthodontic management especially when considering sequencing of the procedures. Phase
I orthodontic care of the child with bilateral cleft lip and palate should consist of maxillary
transverse expansion to reduce cross-bites and align the pre-maxilla for position and
symmetry. This will allow proper bone graft placement and consolidation of the premaxilla
in the most appropriate position for function and aesthetics.

Reference:
Vig K, Turvey TA, Fonseca RJ: Orthodontic and Surgical Considerations in Bone Grafting
in the Cleft Maxilla and Palate. In Facial Clefts and Craniosynostosis Principles and
Management. Turvey et al. Saunders, Philadelphia, 1996. pg 396.

The American Board of Oral and Maxillofacial Surgery 74


2007 Oral and Maxillofacial Surgery Self Assessment Tool (OMSSAT)

73. Intravelar veloplasty during cleft palate repair is undertaken in order to reorient which
aberrantly positioned muscle?

A. Salpingopharyngeus

B. Palatoglossus

C. Levator veli palatini

D. Tensor veli palatini

Answer: C

Rationale:
The term “intravelar veloplasty” was first used by Kriens in 1969 to describe a muscular
reconstruction of the cleft palate. In particular, the levator veli palatini, which lie in an
abnormal sagittal orientation and are aberrantly inserted at the posterior edge of the hard
palate, are mobilized and re-oriented to lie transversely across the palate in a more normal
orientation. The goal is to restore normal palatal function through the restitution of normal
palatal functional anatomy.

The salpingopharyngeous, palatoglossus and superior constrictor, though part of the


velopharyngeal mechanism, are not directly involved in this muscular reconstruction. The
tensor veli palatini muscle passes around the hamular process and is not part of the
musculature that is re-directed during veloplasty.

Reference:
Sommerlad BC. A Technique for Cleft Palate Repair. Plast Reconstr Surg 2003;112:1542-
1548.

The American Board of Oral and Maxillofacial Surgery 75


2007 Oral and Maxillofacial Surgery Self Assessment Tool (OMSSAT)

74. When raising a superiorly based pharyngeal flap, dissection is carried to, but not through,
which anatomic structure?

A. Superior pharyngeal constrictor muscle

B. Palatopharyngeous muscle

C. Pre-vertebral fascia

D. Pre-tracheal fascial

Answer: C

Rationale:
The superiorly based pharyngeal flap recruits tissue from the posterior wall of the pharynx
and inserts it within the nasals side of the soft palate. The procedure involves the elevation
of a superiorly based myomucosal flap (includes mucosa and superior pharyngeal
constrictor muscle) off of the underlying pre-vertebral fascia. During the dissection,
vertical incisions are made and blunt dissection is utilized to reach a glistening, distinct
fascial layer. The soft palate is then split with dissection of the oral, nasal , and muscular
layers. The flap is then inset into the nasal side closure and the oral mucosa is closed over
the raw surface.

Reference:
Costello BJ, Turvey TA. Velopharyngeal insufficiency in patients with cleft palate. Oral
Maxillofacial Surg Clin N Am 14 (2002) 539-551.

The American Board of Oral and Maxillofacial Surgery 76


2007 Oral and Maxillofacial Surgery Self Assessment Tool (OMSSAT)

75. One-stage repair of a cleft of the secondary palate is generally undertaken at what age?

A. Prior to 3 months of age

B. 3-6 months of age

C. 10-18 months of age

D. 3-5 years of

Answer: C

Rationale:
Cleft palate repair is generally undertaken between the ages of 10 and 18 months, The
choice of timing is a balance between optimizing facial growth and the development of
normal speech patterns. Repair prior to this age range has little impact on speech outcomes
but does impact negatively on midfacial growth. Repair is then chosen to coincide with the
generation of early speech such that a child who is not yet speaking need not undergo
palatal repair with any sense of urgency. Delay of palatal repair beyond 18 months is
generally felt to be detrimental to speech outcomes.

Reference:
LaRossa, D. The State of the Art in Cleft Palate Surgery. Cleft Craniofac J 2000;37:225-
228.

The American Board of Oral and Maxillofacial Surgery 77


2007 Oral and Maxillofacial Surgery Self Assessment Tool (OMSSAT)

76. A 17 year-old male with a history of a repaired left unilateral cleft lip and palate presents
with a large Class III skeletal malocclusion. He is scheduled to undergo maxillary
advancement via a LeFort I osteotomy. The anticipated incidence of postsurgical
velopharyngeal insufficiency 6 months post surgery would be approximately:

A. 1%.

B. 15%.

C. 50%.

D. 75%.

Answer: B

Rationale:
Deterioration in velopharyngeal function after maxillary advancement surgery in patients
with cleft palates is a common occurrence. With the passage of time, a spontaneous
recovery is observed such that, at 6 months, only approximately 15% will exhibit a
persistent deterioration in their velopharyngeal function.

Reference:
Janulewicz JJ, Costello BJ, Buckley MJ, et al. The Effects of LeFort I Osteotomies on
Velopharyngeal and Speech Functions in Cleft Patients. J Oral Maxillofac Surg
2004;62:308-314.

The American Board of Oral and Maxillofacial Surgery 78


2007 Oral and Maxillofacial Surgery Self Assessment Tool (OMSSAT)

77. During the primary repair of a cleft palate utilizing a two-flap technique, the greater palatine
neurovascular bundle is transected during elevation of the left palatal flap. What is the next
step in the surgical management of this patient?

A. Abort the procedure and reposition the flap.

B. Proceed with the palatoplasty procedure.

C. Proceed but without any further elevation of the flap.

D. Attempt re-anastamosis of the vessel.

Answer: B

Rationale:
Two-flap palatoplasty techniques involve the elevation of full-thickness mucoperiosteal
flaps on each side of the cleft defect for oral side closure. After the nasal mucosa is closed,
these soft tissue flaps are sutured together in the midline for closure of the cleft defect.
During the initial dissection and elevation of the flaps, the greater palatine neurovascular
bundles are identified and protected. The result is that the axial soft tissue flaps are raised
based upon the blood supply of the greater palatine arteries bilaterally. If the greater
palatine artery is injured or cauterized, surgery may proceed without any modifications.
This is because of the abundant blood supply provided by the palatal soft tissue pedicle.
Injury to the greater palatine artery simply converts the palatal flap's blood supply from an
“axial” pattern to a “random” pattern (i.e. not based on one specific arterial supply).

Reference:
Bardach J: Two-flap palatoplasty: Bardach's technique. Operative Techniques in Plastic
and Reconstructive Surgery, Vol 2. No 4(November), 1995: pp 211-214

The American Board of Oral and Maxillofacial Surgery 79


2007 Oral and Maxillofacial Surgery Self Assessment Tool (OMSSAT)

78. Initial management of the patient who develops velopharyngeal incompetence after LeFort I
advancement surgery should be which of the following?

A. Double opposing Z-plasty (Furlow) palatoplasty

B. V-Y palatoplasty pushback surgery (Veau-Wardhill-Kilner Procedure)

C. Superiorly based pharyngeal flap

D. Observation for six months to one year

Answer: D

Rationale:
While a deterioration in velopharyngeal function is a very common manifestation after
maxillary advancement surgery in the cleft palate population, the long term incidence is
only approximately 18%. Because of the demonstrated recovery potential, it is most
prudent to delay any interventions until sufficient time has passed to allow for spontaneous
return of function.

A Furlow conversion, VY pushback and superiorly based pharyngeal flap are all surgical
modalities employed in the surgical management of velopharyngeal insufficiency. Their
use may be indicated in those where the velopharyngeal function fails to improve but
again, only after waiting 6 months to 1 year to allow for spontaneous recovery.

Reference:
Janulewicz JJ, Costello BJ, Buckley MJ, et al. The Effects of LeFort I Osteotomies on
Velopharyngeal and Speech Functions in Cleft Patients. J Oral Maxillofac Surg
2004;62:308-314.

The American Board of Oral and Maxillofacial Surgery 80


2007 Oral and Maxillofacial Surgery Self Assessment Tool (OMSSAT)

79. In a patient with cleft lip and palate, which of the following treatment modalities may cause
growth restriction of the maxilla?

A. Lip adhesion procedure

B. Lip taping

C. Naso-alveolar molding devices

D. Pin retained orthopedic devices

Answer: D

Rationale:
All of the above listed treatment modalities are frequently utilized prior to definitive cleft
lip repair in order to idealize the position of the maxillary dentoalveolar segments, improve
the position of the alveolus and nasal base, and reduce the width of the cleft defect. The lip
adhesion procedure involves the creation of a small “repair” of the cleft lip defect by
attaching only the mucosa and lip skin without undertaking the repair of the orbicularis
oris muscle. Lip adhesion is performed in order to convert a large complete cleft defect
into a narrower, incomplete cleft defect. After several months, the patient is returned to the
operating room and the definitive lip repair procedure (including the orbicularis muscle) is
undertaken. Routine application of elastic bands and adhesive strips across the cleft defect
may also help narrow the defect as well without the need for surgical intervention. Naso-
alveolar molding devices are composed of a passive plate that fits over the maxillary
segments and have attached wire and acrylic components for use in the molding of the
nasal complex and lip taping. Unlike the other techniques listed, the use of pin-retained
orthopedic devices (Latham appliance) involves more aggressive repositioning of the
maxillary segments. Unfortunately, the use of these pin-retained devices with active
orthopedic movement of the skeletal components is associated with maxillary growth
restriction and a significant increase in the likelihood that orthognathic surgery will be
necessary in the teenage years. The most popular theory to explain this association is that
the aggressive skeletal movements produce disruption with hemorrhage at nasopalatine
and maxillary suture regions.

Reference:
Berkowitz S. The comparison of treatment results in complete cleft lip/palate using
conservative approach vs Millard-Latham PSOT procedure. Semin Orthod 2: 169, 1996.

The American Board of Oral and Maxillofacial Surgery 81


2007 Oral and Maxillofacial Surgery Self Assessment Tool (OMSSAT)

80. In a case of single-suture craniosynostosis, in what direction is cranial growth arrested?

A. Parallel to the affected suture

B. Perpendicular to the affected suture

C. Parallel to the open sutures

D. Perpendicular to the open sutures

Answer: B

Rationale:
The term “craniosynostosis” refers to the premature fusion of one or more of the cranial
vault sutures. The result is fusion of the bones adjacent to the suture and arrested sutural
growth of the adjacent bones. The classic theory, known as Virchow's law, states that
premature fusion of a cranial suture results in limited development of the skull
perpendicular to the fused suture and a compensatory overgrowth at the remaining open
sutures.

Reference:
Nonsyndromic craniosynostosis: diagnosis and management. Ruiz, et.al. Oral and
Maxillofacial Clinics of North America: Craniofacial Update.

The American Board of Oral and Maxillofacial Surgery 82


2007 Oral and Maxillofacial Surgery Self Assessment Tool (OMSSAT)

81. Most forms of craniofacial dysostosis syndromes (e.g. Crouzon, Apert, Saethre-Chotzen)
demonstrate which of the following inheritance patterns?

A. X-linked recessive

B. X-linked dominant

C. Autosomal dominant

D. Autosomal recessive

Answer: C

Rationale:
The craniofacial dysostosis syndromes are inherited forms of craniosynostosis. Patients
are born with more extensive sutural involvement which includes premature fusion of
cranial vault sutures, but also affects the sutures of the mid-facial structures. This results
in a typical morphological pattern that includes a total mid-face deficiency, proptosis, and
a severe Angle Class III malocclusion.

Reference:
Chapter 6, Genetic etiologies of craniosynostosis by Ethylin Wang Jabs, MD.

Understanding craniofacial anomalies, the etiopathogenesis of craniosynostosis and facial


clefting. Mooney and Siegel, Willey-Liss, New York, 2002.

The American Board of Oral and Maxillofacial Surgery 83


2007 Oral and Maxillofacial Surgery Self Assessment Tool (OMSSAT)

82. This CT image of a 3 month old male demonstrates which of the following conditions?

A. Metopic synostosis

B. Coronal synostosis

C. Sagittal synostosis

D. Lambdoid synostosis

Answer: C

Rationale:
The CT scan image shown above reveals sagittal suture craniosynostosis. The sagittal
suture is absent and there is a classic scaphocephalic skeletal deformity. This is
characterized by a narrow cranial vault in the bi-temporal and bi-parietal dimensions and
elongation of the cranial vault in the antero-posterior dimension. This craniofacial
deformity is the result of both arrested development at the site of the affected suture (e.g.
sagittal) and an abnormal compensatory overgrowth at the site of the remaining open
sutures (e.g. coronal and lambdoid).

Reference:
Posnick JC, Lin KY, Chen P, et al. Sagittal Synostosis: Quantitative assessment of
presenting deformity and surgical results based on CT scans. Plast Reconstr Surg 92:1015,
1993.

The American Board of Oral and Maxillofacial Surgery 84


2007 Oral and Maxillofacial Surgery Self Assessment Tool (OMSSAT)

83. In diagnosing a child with a posterior cranial vault asymmetry characterized by unilateral
flattening, benign positional skull molding must be differentiated from which type of
craniosynostosis?

A. Lambdoid synostosis

B. Metopic synostosis

C. Sagittal synostosis

D. Bicoronal synostosis

Answer: A

Rationale:
When a lambdoid suture is absent, there is arrested skeletal growth across the affected
occipital and parietal bones, which also results in significant flattening or posterior
plagiocephaly, but that is where the similarities to positional plagiocephaly end.

Nonsynostotic Positional versus Lambdoid Synostosis


Plagiocephaly

Occipitoparietal flattening Occipitoparietal flattening


Ipsilateral frontal bossing Ipsilateral frontal flattening
Forward displacement of Posterior displacement of
the ipsilateral ear the ipsilateral ear
No bony ridging Bony ridging along involved
suture region

Reference:
From Positional plagiocephaly: evaluation and management. John Caccamese, DMD, MD,
et.al. Oral and Maxillofacial Clinics of North America: Craniofacial Update.

The American Board of Oral and Maxillofacial Surgery 85


2007 Oral and Maxillofacial Surgery Self Assessment Tool (OMSSAT)

84. A 4 year-old child is seen for extraction of a several carious teeth. Her medical history
reveals a repaired ventricular septal defect. On exam, you note a bifid uvula and hypernasal,
poorly intelligible speech. She has normal vision and the remainder of her exam is otherwise
unremarkable. Which of the following is the most likely diagnosis?

A. Van der Woude syndrome

B. Stickler syndrome

C. Nager syndrome

D. Velocardiofacial syndrome

Answer: D

Rationale:
Velocardiofacial syndrome was first described by Shprintzen and colleagues in 1978 as a
syndrome comprising clefts of the palate either overt or submucous, congenital heart
defects, typical facies and learning difficulties. The hypernasal speech and bifid uvula are
suggestive of a submucous cleft palate, which is a feature of the syndrome.

Van der Woude syndrome displays an autosomal dominant pattern of inheritance and
consists of lower lip pits and clefts of the lip and/or palate.

Stickler syndrome, also known as Hereditary Arthro-Ophthalmopathy, consists of a flat


facies with a short nose, epicanthal folds and anteverted nares, a cleft palate,
spondyloepiphyseal dysplasia and high grade myopia.

Nager syndrome is characterized by radial limb hypoplasia, malar hypoplasia with down-
slanting palpebral fissures, ear defects and occasionally a cleft palate

Reference:
Smith's Patterns of Recognizable Human Malformation, ed. 6. Jones ed. Philadelphia:
Saunders 2005:976pp.

The American Board of Oral and Maxillofacial Surgery 86


2007 Oral and Maxillofacial Surgery Self Assessment Tool (OMSSAT)

85. Which cranial vault suture(s) is most commonly involved in Crouzon Syndrome?

A. The lambdoid suture(s)

B. The sagittal suture

C. The coronal suture(s)

D. The metopic suture

Answer: C

Rationale:
The craniosynostosis of Crouzon, Apert, and Pfeiffer syndromes involve sites of
pathogenesis either unilaterally or bilaterally along the calvarial articulations of the coronal
ring. Most commonly, patients affected with one of these craniofacial dysostosis
syndromes will present with bilateral involvement of the coronal sutures.

Reference:
Chapter 16, Facial dysmorphology in the craniosynostoses: clinical implications by
Katherine W.L. Vig, BDS, MS, Dorth.

Understanding craniofacial anomalies, the etiopathogenesis of craniosynostosis and facial


clefting. Mooney and Siegel, Willey-Liss, New York, 2002.

The American Board of Oral and Maxillofacial Surgery 87


2007 Oral and Maxillofacial Surgery Self Assessment Tool (OMSSAT)

86. In which disorder is absence of the frontal sinus most commonly noted?

A. Apert Syndrome

B. Carpenter Syndrome

C. Cystic Fibrosis

D. Neurofibromatosis

Answer: C

Rationale:
Knowledge of how certain genetic disorders modify the craniofacial skeleton has an impact
on directed surgical intervention. In cystic fibrosis, congenital absence of the frontal sinus
is a noted feature of the disease. Patients with a diagnosis of cystic fibrosis are no more or
less likely to sustain maxillofacial trauma than those of their age matched peers and are
more susceptible to severe maxillofacial infections than that of the general population
because of chemotherapeutic regimens and disorders of mucous regulation.

Reference:
Boat TF: Cystic Fibrosis. In Nelson Textbook of Pediatrics. Behrman, Kliegman, Jensen
eds. Saunders, Philadelphia, 2000. pg 1315.

The American Board of Oral and Maxillofacial Surgery 88


2007 Oral and Maxillofacial Surgery Self Assessment Tool (OMSSAT)

87. The clinical term trigonocephaly is used to describe which form of craniosynostosis?

A. Metopic

B. Sagittal

C. Lambdoid

D. Coronal

Answer: A

Rationale:
The term trigonocephaly (e.g. triangular-head) refers to a specific morphological pattern
that may be the result of metopic suture craniosynostosis. In patients with absence of the
metopic suture, there is a lack of skeletal growth perpendicular to the metopic suture
which runs down the center of the forehead. At the same time, there is an increased,
“compensatory,” growth at the cranial vault sutures that remain open. This results in the
characteristic trigonocephalic deformity which includes a triangular-shaped forehead,
hypotelorism, and horizontally retrusive lateral orbital rims.

Reference:
From Chapter 2, Terminology and classification of craniosynostosis by Marilyn Jones,
MD. Understanding craniofacial anomalies, the etiopathogenesis of craniosynostosis and
facial clefting. Mooney and Siegel, Willey-Liss, New York, 2002.

The American Board of Oral and Maxillofacial Surgery 89


2007 Oral and Maxillofacial Surgery Self Assessment Tool (OMSSAT)

88. In a patient with Treacher Collins Syndrome, the reconstruction of zygomatic and orbital
deformities should be delayed until what age?

A. 1 to 2 years

B. 3 to 4 years

C. 8 to 10 years

D. 4 to 17 years

Answer: C

Rationale:
Children with Treacher Collins Syndrome require multiple stages of skeletal reconstruction
which include repair of the orbitozygomatic deformities and definitive orthognathic
surgery. Reconstruction of the orbital hypoplasia and missing zygomatic arches is
typically undertaken with the use of cranial bone grafts.

Whenever possible, orbitozygomatic reconstruction should be delayed until the orbital


structures themselves are at or approaching skeletal maturity. By seven years of age, the
cranio-orbitozygomatic skeleton is almost completely done growing. This makes late
childhood the ideal time for construction of orbital and zygomatic components. If surgical
correction of the orbital-zygomatic deformity is undertaken during infancy or early
childhood, then subsequent growth may adversely effect the outcome.

Reference:
Waitzmann AA, Posnick JC, Armstrong D, et al. Craniofacial skeletal measurements
based on computed tomography: II. Normal values and growth trends. Cleft Palate
Craniofacial Journal 29:118, 1992.

The American Board of Oral and Maxillofacial Surgery 90


2007 Oral and Maxillofacial Surgery Self Assessment Tool (OMSSAT)

89. This CT scan image is of a 6 month old infant. What is the most appropriate course of
treatment for the patient?

A. Observation with no intervention

B. Reshaping of the posterior two-thirds of the cranial vault

C. Reshaping of the anterior cranial vault and fronto-orbital advancement

D. Non-surgical treatment with a custom cranial orthotic band

Answer: C

Rationale:
The CT Scan demonstrates that the right coronal suture is congenitally missing confirming
the diagnosis of unilateral coronal craniosynostosis. Absence of growth perpendicular to
this suture causes a characteristic craniofacial deformity that is characterized by orbital
dystopia, shortening of the anterior cranial base on the affected side, and flattening of the
forehead.
The treatment for this patient is designed to release the affected suture so that normal brain
growth can occur and increased intracranial pressure is avoided. At the same time, the
dysmorphic skeletal components are reshaped into a more anatomically correct position
and contour. Reconstruction typically requires, bifrontal craniotomy for access, fronto-
orbital osteotomies with reshaping, and reshaping of the anterior cranial vault.

Reference:
Turvey TA, Gudeman SK. Nonsyndromic Craniosynostosis. In: Turvey TA, Vig KWL,
and Fonseca RJ (Editors): Facial Clefts and Craniosynostosis: Principles and Management.
Pp: 596-629, W.B. Saunders. Philadelphia, 1996.

The American Board of Oral and Maxillofacial Surgery 91


2007 Oral and Maxillofacial Surgery Self Assessment Tool (OMSSAT)

90. During craniofacial surgery, a venous air embolism is suspected by the surgeon. Which of
the following maneuvers would be most helpful in controlling the entry of air into the venous
circulation?

A. Raising the patient’s head above the level of the heart

B. Limiting or avoiding irrigation of the involved surgical field

C. Application of bone wax to cut bone edges

D. Administration of a nitrous oxide – oxygen mixture

Answer: C

Rationale:
The cranial vault has a large number of diploic and emissary venous channels. These
channels within the bone can be opened during a variety of surgical procedures including
split-thickness cranial bone harvest, skull base procedures, and any reconstructive
craniofacial operation that includes a craniotomy for access. As these venous channels are
opened, air may travel to the right atrium. Because 35% of the pediatric population has a
patent foramen ovale, the paradoxic passage of air from the right to left heart is possible
and this air may then enter the cerebral and/or coronary circulation.

When air embolus is suspected during a craniofacial procedure, rapid control of the air
entry portal is critical. Several technical maneuvers are useful in eliminating the entry of
air into the cranial venous circulation. These include cessation of surgery, rapid lowering
of the head below the level of the heart, copious irrigation of the involved field, manual
compression of the venous channel, and the use of bone wax on cut bone surfaces. Nitrous
oxide should be avoided and 100% oxygen administered.

Reference:
Greensmith AL, Meara JG, and Holmes AD. Complications related to cranial vault
surgery. Oral Maxillofacial Surg Clin N Am 16 (2004) 465-473.

The American Board of Oral and Maxillofacial Surgery 92


2007 Oral and Maxillofacial Surgery Self Assessment Tool (OMSSAT)

91. When assessing the efficacy of myofascial trigger points injections what substance should be
used as a control?

A. Xylocaine

B. Sterile water

C. Dry needling of the trigger point

D. Normal saline

Answer: D

Rationale:
Normal saline is used as the control in studies of efficacy of trigger point injection
solutions.

Reference:
Byrn C, Olsson J, Falkheden L, Lindh M, Hosterey U, Fogelberg MN, Linder LE,
Bundertorp O. Subcutaneous sterile water injections for chronic neck and shoulder pain
following whiplash injuries. Lancet, 1993;341:449-52.

The American Board of Oral and Maxillofacial Surgery 93


2007 Oral and Maxillofacial Surgery Self Assessment Tool (OMSSAT)

92. Which of the following classes of drugs may provide benefits of both analgesia and reduction
of bruxism in the myofascial pain patient?

A. Monoamine oxidase inhibitors

B. Tricyclic antidepressants

C. Nonsteroidal anti-inflammatory medications

D. Selective serotonin reuptake inhibitors

Answer: B

Rationale:
Tricyclic antidepressants, including specifically amitriptyline, have been suggested to be
effective in alleviating pain in chronic pain syndromes and in reducing bruxism by an
unknown mechanism. None of the distractors have been shown to decrease bruxism.
SSRI's may increase bruxism.

Reference:
Karlis, V and Glickman, R. Nonsurgical management of temporomandibular disorders. In
Miloro, M (ed). Peterson's Principles of Oral and Maxillofacial Surgery, 2nd ed. 2004
Hamilton: BC Decker p. 953.

The American Board of Oral and Maxillofacial Surgery 94


2007 Oral and Maxillofacial Surgery Self Assessment Tool (OMSSAT)

93. Comparison of outcomes one year after arthroscopy, discectomy, modified condylotomy, and
disc repositioning surgery for TMJ internal derangement is most correctly summarized by
which of the following statements?

A. Superior pain relief and diet improvement after arthroscopy

B. Statistically indistinguishable pain relief and diet improvement

C. Superior pain relief and improved diet after disc repositioning surgery

D. Statistically indistinguishable range of motion

Answer: B

Rationale:
A prospective investigation comparing outcomes one year after arthroscopy, discectomy,
condylotomy, and disc repositioning for internal derangement showed no statistically
significant differences in pain reduction or diet improvement. Greater improvement in
contralateral range of motion occurred with condylotomy and arthroscopy, presumably due
to intracapsular scarring following discectomy and disc repositioning.

Reference:
Hall HD, Indresano TA, Kirk WS, et al. Prospective multicenter comparison of 4
temporomandibular joint operations. J Oral Maxillofac Surg 2005;63:1174-1179

The American Board of Oral and Maxillofacial Surgery 95


2007 Oral and Maxillofacial Surgery Self Assessment Tool (OMSSAT)

94. Which statement most accurately describes the TMJ disc?

A. Convex superior surface, concave inferior surface

B. Composed primarily of Type-2 collagen

C. Biconcave cross-sectional morphology

D. Composed of hyaline cartilage

Answer: C

Rationale:
In cross section the disc has a biconcave shape. This shape enhances load distribution
between the surfaces of the condyle and temporal bone. The disc is composed of fibrous
connective tissue. The disc has little or no innervation.

Reference:
Mohamed,S:Developmental and Clinical Anatomy and Physiology of the
Temporomandibular Joint. In Fonseca RJ, Bays RA, Quinn PD (eds):Oral and
Maxillofacial Surgery Vol 4. 2000, WB Saunders, Philadelphia, p 8.

The American Board of Oral and Maxillofacial Surgery 96


2007 Oral and Maxillofacial Surgery Self Assessment Tool (OMSSAT)

95. Which of the following best describes the correct sequence of structures encountered
superior to the zygomatic arch with a preauricular approach to the temporomandibular
superior joint space?

A. Skin, superficial deep temporal fascia, temporoparietal fascia, subgaleal fascia, capsule

B. Skin, subgaleal fascia, temporoparietal fascia, temporalis fascia, capsule

C. Skin, temporoparietal fascia, superficial deep temporalis fascia, subgaleal fascia,


capsule

D. Skin, temporoparietal fascia, subgaleal fascia, temporalis fascia, capsule

Answer: D

Rationale:
Temporoparietal fascia is continuous superiorly with galea. Subgaleal fascia is a discrete
fascial layer deep to temporoparietal fascia. The layer immediately deep to subgaleal
fascia is temporalis fascia. Temporalis fascia is immediately superficial to joint capsule.

Reference:
Politi M, Toro C, Cian R, Costa, F, et al. The deep subfascial approach to the
temporomandibular joint. J oral Maxillofac Surg 2003;62:1097-1102.

Ellis EE, Fide MF. Surgical Approaches to the facial skeleton. Williams and Wilkins,
Baltimore, 1995 p.167-911.

The American Board of Oral and Maxillofacial Surgery 97


2007 Oral and Maxillofacial Surgery Self Assessment Tool (OMSSAT)

96. The auriculotemporal nerve:

A. originates from 2 roots surrounding the anterior deep temporal artery.

B. innervates the stapedius muscle.

C. innervates the lateral pterygoid muscle.

D. has communications with the otic ganglion.

Answer: D

Rationale:
The auriculotemporal nerve is one of several nerves provides sensation to the TMJ and a
portion of the ear. Its origin is by two roots that surround the middle meningeal artery. The
stapedius muscle is innervated by the facial nerve. The lateral pterygoid is innervated by
pterygoid branches form the 3rd division of the trigeminal nerve. Auriculotemporal
contributions to the otic ganglion supply afferent innervation to the parotid gland.

Reference:
Mohamed,S: Developmental and Clinical Anatomy and Physiology of the
Temporomandibular Joint. In Fonseca RJ, Bays RA, Quinn PD (eds): Oral and
Maxillofacial Surgery Vol 4, 2000, WB Saunders, Philadelphia, p. 14.

The American Board of Oral and Maxillofacial Surgery 98


2007 Oral and Maxillofacial Surgery Self Assessment Tool (OMSSAT)

97. What statement is true about internal derangement of the TMJ?

A. Disc morphology is more important than disc position.

B. Disc mobility is more important than disc position.

C. Disc position is more important than disc mobility.

D. Disc morphology is more important than disc mobility.

Answer: B

Rationale:
Disc displacement may follow or precede alteration in the surface characteristics of the
joint, although contemporary literature has discounted the importance of both disc
displacement and the value of surgical disc repositioning. Impaired disc mobility is more
closely related to alterations in the internal milieu of the joint and altered joint mechanics
than is disc position. Disc displacement, though a marker of internal derangement, is
probably not as important as altered disc mobility

Reference:
Dolwick, MF: Temporomandibular Joint Disc Displacement: A Clinical Perspective. In,
Sessle, BJ; Bryant, PS; Dionne, RA (Eds): Temporomandibular Disorders and Related
Pain Conditions. Progress in Pain Research and Management. Vol 4 1995 IASP Press.
Seattle. pp.79 – 87.
Mercuri, LG and Laskin, DM: Indications for Surgical Treatment of Internal
Derangements of the TMJ. Oral Maxillofac Surg Clin N Am 1994;6:223.

The American Board of Oral and Maxillofacial Surgery 99


2007 Oral and Maxillofacial Surgery Self Assessment Tool (OMSSAT)

98. The temporal branch of the seventh cranial nerve is located in which soft tissue plane when
approaching the TMJ via a preauricular approach?

A. Within the superficial temporal fat pad

B. Within the subcutaneous fat

C. Deep surface of the temporalis fascia

D. Deep surface of the temporoparietal fascia

Answer: D

Rationale:
The temporal branch of the facial nerve is most commonly found on the deep surface of the
temporoparietal fascia and superficial to the temporalis fascia.

Reference:
Ellis, E, Zide, M: Surgical Approaches to the Facial Skeleton, Baltimore, 1995, Williams
and Wilkins, pp. 167-168

The American Board of Oral and Maxillofacial Surgery 100


2007 Oral and Maxillofacial Surgery Self Assessment Tool (OMSSAT)

99. In the preauricular approach to the TM joint what best describes the location of temporal
branch of the facial nerve?

A. Deep to temporalis fascia, 8-35 mm anterior to the external auditory canal

B. Deep to temporoparietal fascia, 8-35 mm anterior to the external auditory canal

C. Deep to temporoparietal fascia, mean 3.5 cm anterior to the external auditory canal

D. Deep to temporalis fascia, mean 3.5 cm anterior to the external auditory canal

Answer: B

Rationale:
The main trunk of facial nerve exits from the skull at the stylomastoid foramen and
subsequently enters the parotid gland. The temporal branch of the facial nerve emerges
from the parotid gland and crosses the zygoma deep to or within the temporoparietal fascia
to innervate the frontalis muscle in the forehead. The temporal branch crosses the
zygomatic arch 8-35 mm (mean 2.0cm) anterior to the most anterior portion of the external
auditory canal.

Reference:
Quinn P: Color atlas of temporomandibular joint surgery. 1988 Mosby St. Louis pps. 30-
31.

Ellis E, Zide: Surgical approach to the facial skeleton. 1995 Lippincott Williams &
Wilkins, Baltimore pps. 163-185.

Al-Kayat A, Bramley P: Modified pre-auricular approach to the temporomandibular joint


and malar arch. Br J Oral Surg 1980;17:91.

The American Board of Oral and Maxillofacial Surgery 101


2007 Oral and Maxillofacial Surgery Self Assessment Tool (OMSSAT)

100. MR image generation is a function of which mechanism?

A. Detection of tissue positron emission

B. Detection of tissue features as a radiofrequency signal

C. Detection of tissue gamma irradiation

D. Detection of ionizing radiation which has traversed the tissue

KEY: B

Answer: B

Rationale:
Choice A describes Positron emission tomography (PET). It is a nuclear medicine
imaging technique which produces a three dimensional image or map of functional
processes in the body. PET is a valuable technique for some diseases and disorders,
because it is possible to target the radio-chemicals used for particular bodily functions.
Choice C Describes Bone Scan. A bone scan is a nuclear medicine study to detect bone
abnormalities. The patient is injected with a small amount of radioactive material and
then scanned with a Gamma camera, a device sensitive to the radiation emitted by the
injected material. Several gamma-emitting radioisotopes are used, one of which is
technetium-99m. When administered to a patient, a gamma camera can be used to form
an image of the radioisotope's distribution by detecting the gamma radiation emitted.
Such a technique can be employed to diagnose a wide range of conditions (e.g. spread
of cancer to the bones).
Choice D Describes X-ray. X-rays are primarily used for diagnostic medical imaging
and crystallography. X-rays are a form of ionizing radiation and as such can be
dangerous. Ionizing radiation is a type of particle radiation in which an individual
particle (for example, a photon, electron, or helium nucleus) carries enough energy to
ionize an atom or molecule (that is, to completely remove an electron from its orbit). If
the individual particles do not carry this amount of energy, it is essentially impossible
for even a large flood of particles to cause ionization. These ionizations, if enough
occur, can be very destructive to living tissue. By far, the most significant source of
man-made radiation exposure to the general public is from medical procedures, such as
diagnostic X-rays, nuclear medicine, and radiation therapy. Some of the major
radionuclides used are I-131, Tc-99, Co-60, Ir-192, Cs-137. These are rarely released
into the environment.

The American Board of Oral and Maxillofacial Surgery 102


2007 Oral and Maxillofacial Surgery Self Assessment Tool (OMSSAT)

Reference:
Grainger & Allison's Diagnostic Radiology: A Textbook of Medical Imaging, 4th ed.,
Copyright © 2001 Churchill Livingstone, Inc.

The American Board of Oral and Maxillofacial Surgery 103


2007 Oral and Maxillofacial Surgery Self Assessment Tool (OMSSAT)

101. Low signal intensity on a T-1 weighted TMJ MRI is characteristic of which substance?

A. Joint effusion

B. Marrow

C. Cortical bone

D. Fat

Answer: C

Rationale:
Magnetic resonance imaging (MRI) is a noninvasive method of mapping the internal
structure of the body which completely avoids the use of ionizing radiation and appears
to be without hazard. It employs radiofrequency (rf) radiation in the presence of
carefully controlled magnetic fields in order to produce high quality cross-sectional
images of the body in any plane. It portrays the distribution of hydrogen nuclei and
parameters relating to their motion in water and lipids.

The exposure of the tissue to a Radiofrequency (RF) radiation causes the nuclei in the
lower energy state to jump to the higher energy state. MR Imaging is based on the
observation of the relaxation that takes place after the RF pulse has stopped and
subsequent return of the excited nuclei from the high energy to the low. In pure water,
the T2 and T1 times are approximately identical. For biological material, the T2 time is
considerably shorter than the T1 time. By varying imaging it is possible to weight the
signal to produce T1-, T2- or PD-weighted (proton density) images. From a medical
perspective, it means that MR Imaging can provide multiple channels to observe the
same anatomy. For instance in a brain image, white matter appears in a light grey in T1
and a dark grey in T2. Grey matter appears grey in both images. The Cerebro-Spinal
Fluid (CSF) appears black in T1 and white in T2. The background of the image (air)
appears black in both images. Fluid matters such as edema and water appear white (or
whiter) on the T-2 image compared to T-1, while bone will appear darker.

Reference:
Grainger & Allison's Diagnostic Radiology: A Textbook of Medical Imaging, 4th ed.,
Copyright © 2001 Churchill Livingstone, Inc. .

The American Board of Oral and Maxillofacial Surgery 104


2007 Oral and Maxillofacial Surgery Self Assessment Tool (OMSSAT)

102. Which of the following structures lies in closest proximity to the medial aspect of the
TMJ?

A. Carotid artery

B. Middle meningeal artery

C. Internal jugular vein

D. 3rd division of Trigeminal nerve

Answer: B

Rationale:
The mean distance from the outer aspect of the zygomatic arch to the middle meningeal
artery has been reported as 31 mm (range: 21-43 mm). This vessel is located slightly
forward of the center (i.e., depth) of the glenoid fossa (mean: 2.4 mm). The mean
distances from the outer aspect of the zygomatic arch to the carotid artery (37.5 mm;
range: 29-48 mm), internal jugular vein (38.3 mm; range, 31-49 mm), and the third
division of the trigeminal nerve (35 mm; 24-46 mm) were greater than that of the
middle meningeal artery, but are nevertheless at risk for injury during TMJ surgery.

Reference:
Talebzadeh, N. Rosenstein, TP. Pogrel, MA. Anatomy of the structures medial to the
temporomandibular joint. Oral Surg Oral Med Oral Pathol Oral Radiol Endod, 1999;
88): 674-78.

The American Board of Oral and Maxillofacial Surgery 105


2007 Oral and Maxillofacial Surgery Self Assessment Tool (OMSSAT)

103. Patients who undergo unilateral segmental mandibular resection with preservation of a
short condylar segment can be expected to have what findings prior to delayed mandibular
reconstruction:

A. painful disc displacement with the condyle out of the fossa.

B. painless normal disc relationship with the condyle in the fossa.

C. painless disc displacement with the condyle out of the fossa.

D. painless normal disc relationship with the condyle out of the fossa.

Answer: D

Rationale:
Unilateral segmental mandibular resection does not affect the relationship between the
disc and the condyle. Most condyles with a short condyle-ramus remnant will displace
out of the fossa while retaining a normal relationship to the disc. Clinical symptoms of
TMJ pain, TMJ noise, or muscular pain are not associated with partial mandibulectomy
defects.

Reference:
Yoshiki H, Toshirou K, Kazutoshi N, Kanichi S: Magnetic resonance imaging findings
and clinical symptoms in the temporomandibular joint in patients with mandibular
continuity defects. J Oral and Maxillofac Surg 2000;50: 487-493.

The American Board of Oral and Maxillofacial Surgery 106


2007 Oral and Maxillofacial Surgery Self Assessment Tool (OMSSAT)

104. A TMJ MRI has been obtained on a symptomatic 19-year-old. Which clinical scenario is
most compatible with this imaging study?

A. Bilateral clicking

B. Recent cessation of clicking with deviation on opening

C. Unilateral reciprocal clicking

D. Bilateral crepitace

The American Board of Oral and Maxillofacial Surgery 107


2007 Oral and Maxillofacial Surgery Self Assessment Tool (OMSSAT)

Answer: B

Rationale:
This MRI demonstrates unilateral disc displacement without reduction. Preservation of
near normal disc morphology is consistent with recent progression to disc displacement
without reduction. The contralateral joint disc position is normal. No clicking would be
expected and, in fact, this patient stopped clicking and experienced a closed lock with
deviation to the locked side 3 weeks prior to the MRI. The MRI demonstrates normal
condylar morphology without evidence of arthrosis.

Reference:
Katzberg RW, et al. Magnetic resonance imaging of the temporomandibular joint
meniscus. Ora Surg Oral Med Oral Pathol 1985;59:332.

Anderson QN. Temporomandibular joint imaging: treatment planning. In: Fonseca RJ,
Bays RA, Quinn PD (eds):Oral and Maxillofacial Surgery Vol 4. 2000, WB Saunders,
Philadelphia, pp129-142.

The American Board of Oral and Maxillofacial Surgery 108


2007 Oral and Maxillofacial Surgery Self Assessment Tool (OMSSAT)

105. A 12 year-old male is noted to have his mandibular dental and symphysis midlines 3 mm to
the right of his facial midline. Centric occlusion position approximates centric relation
position. These findings are most consistent with:

A. myofascial pain dysfunction.

B. internal derangement of the right TMJ.

C. left lateral pterygoid spasm.

D. internal derangement of the left TMJ.

Answer: B

Rationale:
Mandibular asymmetry in a growing child is often associated with an internal
derangement on the short side. This association is associated with a small or deformed
condyle and decreased vertical ramus height on the affected side.

Reference:
Schellhas KP, Pollei SR, Wilkes CH: Pediatric Internal Derangements of the
Temporomandibular Joint: Effect on facial development. Am J Orthodont Dentofac
Orthoped, 1993;104:51-59

The American Board of Oral and Maxillofacial Surgery 109


2007 Oral and Maxillofacial Surgery Self Assessment Tool (OMSSAT)

106. What is the most common configuration of TMJ disc displacement?

A. Medial

B. Anteromedial

C. Anterior

D. Anterolateral

Answer: C

Rationale:
Of 58 consecutive TMJ patients imaged by 3-DIMENSIONal MRI 44/116 (38%) had
anterior displacement of the TMJ disk. 23/116 (20%) had anterolateral displacement,
10/116 (9%) had medial displacement, and 10/116 (9%) had anteromedial displacement.

Reference:
Simmons HC, Gibbs SJ: Initial TMJ disk recapture with anterior repositioning
appliances and relation to dental history. J Craniomandib Prac 1997;15:281-295.

The American Board of Oral and Maxillofacial Surgery 110


2007 Oral and Maxillofacial Surgery Self Assessment Tool (OMSSAT)

107. Which of the following skeletal jaw deformities is most likely associated with long-
standing bilateral TMJ disc displacement without reduction?

A. High mandibular plane angle Class II malocclusion

B. Asymmetric Class II malocclusion

C. Low mandibular plane angle Class II malocclusion

D. Class III malocclusion

Answer: A

Rationale:
TMJ internal derangement should be suspected in patients with mandibular deficiency,
symmetrical and asymmetric Class II malocclusion, and vertical ramus deficiency with
anterior open bite. Of the skeletal deformity options, high mandibular plane angle Class
II patient is most likely to be associated with bilateral non-reducing disc displacements.
Nickerson and Link reported that 11/11 open bite patients and 29/33 Class II patients
had bilateral internal derangements.

Reference:
Link JL, Nickerson JW. Temporomandibular joint internal derangements in an
orthognathic surgery population. Int J Adult Orthod Orthognath Surg 1992;7:161-9
Gidarakou IK, Tallents RH, Kyrkanides S, et al. Comparison of skeletal and dental
morphology in asymptomatic volunteers and symptomatic patients with bilateral disc
displacement with reduction. Angle Orthod 2002;72:541-46.

The American Board of Oral and Maxillofacial Surgery 111


2007 Oral and Maxillofacial Surgery Self Assessment Tool (OMSSAT)

108. Which of the following is the most common complaint in TMD patients?

A. Pain with opening

B. Open lock

C. Headache

D. Tinnitus

Answer: C

Rationale:
Forty-eight consecutive TMD patients were asked 86 different symptoms on initial
diagnostic visit. The occurrence of the symptoms was: Cephalalgia 94%, Painful to
chew food 94%, Pain upon opening or closing mouth 90%, Open lock 13%.

Reference:
Simmons HC, Gibbs SJ: Anterior Repositioning Appliance Therapy for TMJ Disorders:
Specific symptoms relieved and relationship to disk status on MRI. J Craniomandib
Prac 2005;23:89-99

The American Board of Oral and Maxillofacial Surgery 112


2007 Oral and Maxillofacial Surgery Self Assessment Tool (OMSSAT)

109. Which of the following devices is an absolute contraindication to performing an MRI of


the temporomandibular joint?

A. Cardiac pacemaker

B. Titanium dental implants

C. Orthodontic appliances

D. Prosthetic knee joint

Answer: A

Rationale:
A cardiac pacemaker can malfunction when subjected to a strong magnetic field. The
other choices are typically not affected by the magnetic field.

Reference:
Quinn, P: Color Atlas of Temporomandibular Joint Surgery, St. Louis, 1998, Mosby, p.
23.

The American Board of Oral and Maxillofacial Surgery 113


2007 Oral and Maxillofacial Surgery Self Assessment Tool (OMSSAT)

110. What tissue type demonstrates a bright signal on T1-weighted MR images?

A. Fat

B. Muscle

C. Bone

D. Cartilage

Answer: A

Rationale:
Fat appears as a bright (white) signal on conventional T1 weighted MRI.

Reference:
Cunningham, L et al. Magnetic resonance imaging of the head and neck. Atlas of the
Oral and Maxillofacial Surgery Clinics of North America 2003;11:87-88.

The American Board of Oral and Maxillofacial Surgery 114


2007 Oral and Maxillofacial Surgery Self Assessment Tool (OMSSAT)

111. A 21 year-old patient with a two week history of painful limited opening (20 mm
interincisal) presents with the following MRI images. What is the most likely diagnosis?

A. Acute TM joint effusion

B. Disc displacement without reduction

C. Disc displacement with reduction

D. Lateral pterygoid fibrosis

Answer: B

The American Board of Oral and Maxillofacial Surgery 115


2007 Oral and Maxillofacial Surgery Self Assessment Tool (OMSSAT)

Rationale:
The question presents a patient with acute onset of hypomobility. T1 weighted MRI
images in closed and open mouth views are presented. The disc is displaced in the
closed mouth view, and does not reduce in the open mouth image. Muscle fibrosis
would not occur acutely. Joint effusion is best imaged with a T2 weighted MRI.

Reference:
Katzberg, RW. Temporomandibular joint imaging. Radiology 1989;170:297-307
Manzione, JV, Katzberg, RW, Tallents, RH. Magnetic resonance imaging of the
temporomandibular joint. J Am Dent Assoc 1986;113:398-402.

The American Board of Oral and Maxillofacial Surgery 116


2007 Oral and Maxillofacial Surgery Self Assessment Tool (OMSSAT)

112. Which statement is true regarding TMJ magnetic resonance imaging technique?

A. Sagittal images should be parallel to the long axis of the condyle

B. Coronal images should be perpendicular to the long axis of the condyle

C. Image should be acquired with surface coils

D. Open mouth images should be acquired at an opening that does not produce an opening
click

Answer: C

Rationale:
MRI of the TMJ image slice thickness should never be more than 3 mm. Sagittal images
should be perpendicular and coronal images parallel to the condylar long axis. Open
mouth images should be acquired at an opening that produces an opening click as this
click may represent a reducing disc.

Reference:
Gibbs SJ, Simmons HC: A protocol for magnetic resonance imaging of the
temporomandibular joint. J Craniomandib Prac 1998 16:236-241.

The American Board of Oral and Maxillofacial Surgery 117


2007 Oral and Maxillofacial Surgery Self Assessment Tool (OMSSAT)

113. A T2 weighted image of the temporomandibular joint best demonstrates which of the
following joint abnormalities?

A. Superior joint space adhesions

B. Perforation of the Temporomandibular Disc

C. Arthrosis of the bony condyle

D. Joint effusion

Answer: D

Rationale:
T2 weighted images best demonstrate structures with high water content. Joint effusion
is best demonstrated by this modality.

Reference:
Quinn, P: Color Atlas of Temporomandibular Joint Surgery, St. Louis, 1998, Mosby, p.
22.

The American Board of Oral and Maxillofacial Surgery 118


2007 Oral and Maxillofacial Surgery Self Assessment Tool (OMSSAT)

114. What statement best describes a Wilkes Stage III TMJ internal derangement?

A. Non-reducing disk displacement-chronic

B. Non-reducing disk displacement-subacute/acute

C. Reducing disk displacement-chronic

D. Reducing disk displacement-subacute/acute

Answer: B

Rationale:
In the Wilkes Classification of internal derangement stage III represents a non-reducing
disk displacement that is acute or subacute. Early stage III internal derangements may
be reducible with mandibular manipulation. Late stage III internal derangements
represent permanent disc displacement.

Reference:
Bays, R: Surgery for Internal Derangement. In Fonseca, R (ed): Oral and Maxillofacial
Surgery, Vol. 4. Philadelphia, WB Saunders, 2000, p 276.

The American Board of Oral and Maxillofacial Surgery 119


2007 Oral and Maxillofacial Surgery Self Assessment Tool (OMSSAT)

115. Which of the following is consistent with Wilkes Stage III internal derangement?

A. Painless reducing disc

B. Painless nonreducing perforated disc

C. Painful reducing disc

D. Acute or subacute painful nonreducing disc

Answer: D

Rationale:
Stage III refers to disc displacement without reduction often associated with pain
but prior to any hard tissue changes.

A. Describes Stage I
B. Describes Stage V
C. Describes Stage II

Reference:
Bays RA :Surgery for Internal Derangement. In Fonseca RJ, Bays RA, Quinn
PD (eds):Oral and Maxillofacial Surgery Vol 4. 2000, WB Saunders,
Philadelphia ,p. 276.

The American Board of Oral and Maxillofacial Surgery 120


2007 Oral and Maxillofacial Surgery Self Assessment Tool (OMSSAT)

116. What results are expected 6 months after TMJ arthrocentesis for pain and limitation of
motion from osteoarthritis?

A. Increased range of motion and decreased pain

B. Unchanged range of motion and decreased pain

C. Increased range of motion and unchanged pain

D. Decreased range of motion and decreased pain

Answer: A

Rationale:
Arthrocentesis is a safe procedure that in many instances results in the osteoarthritic
TMJs returning to a healthy functional state. Failure of arthrocentesis suggests that the
painful limitation is most probably caused by changes such as fibrous adhesions or
osteophytes that require surgical intervention for their removal. Of 38 TMJs treated
with arthrocentesis, 26 joints responded favorably; pain and dysfunction scores were
reduced from 9.86 ± 0.73 to 3.39 ± 0.76 and from 11.34 ± 0.66 to 3.4 ± 0.69,
respectively (P < .001). Maximal mouth opening increased from 24.40 ± 2.70 mm to
43.20 ± 3.10 mm (P < .001). Lateral and protrusive jaw movements also increased.

Reference:
Nitzan D, Price: Arthrocentesis for osteoarthritis TMJ. J Oral Maxillofac Surg 2001;
59:1154.

The American Board of Oral and Maxillofacial Surgery 121


2007 Oral and Maxillofacial Surgery Self Assessment Tool (OMSSAT)

117. Regarding TMJ discectomy and dermal graft placement, which of the following is most
correct?

A. With joint function, dermis assumes a biconcave shape.

B. Dermis grafting prevents condylar erosion after discectomy.

C. Dermis grafting has not been demonstrated superior to discectomy alone.

D. Dermis grafting limits intraarticular adhesion formation.

Answer: C

Rationale:
There is little evidence that dermal graft placement produces results superior to
discectomy, alone. Dermis does not assume a biconcave shape and does not prevent
condylar erosion after discectomy. Dermal grafting does no prevent intraarticular
adhesion formation.

Reference:
Bays RA :Surgery for Internal Derangement. In Fonseca RJ, Bays RA, Quinn PD
(eds):Oral and Maxillofacial Surgery Temporomandibular Disorders. Philadelphia, WB
Saunders, 2000 , Volume 4, p 292.

The American Board of Oral and Maxillofacial Surgery 122


2007 Oral and Maxillofacial Surgery Self Assessment Tool (OMSSAT)

118. The expected outcome after unilateral temporomandibular joint discectomy without disc
replacement is:

A. open bite.

B. loss of range of motion.

C. subcondylar cyst formation.

D. formation of a pseudo-disc.

Answer: D

Rationale:
MR imaging after discectomy without disc replacement has demonstrated formation of
new tissue between the condyle and fossa. This tissue has been referred to as pseudo-
disc because of its location. Loss of mandibular height, open bite, crepitation, loss of
range of motion, though described in isolated cases, are not the usual outcome of
discectomy. Accelerated condylar arthrosis has also been observed but with avoidance
of early joint loading is not expected. Subcondylar cyst formation is a manifestation of
osteoarthritis and not associated with discectomy.

Reference:
Susumu T, Tsuguo S, Masashi Y: Long-term magnetic resonance imaging after
temporomandibular joint discectomy without replacement. J Oral Maxillofac Surg
2000;58:739-745.

The American Board of Oral and Maxillofacial Surgery 123


2007 Oral and Maxillofacial Surgery Self Assessment Tool (OMSSAT)

119. To protect the facial nerve in the preauricular approach to the temporomandibular joint, the
incision through the fascia and periosteum in the area of the zygomatic arch should not be
more than ________ cm from the anterior border of the external auditory canal.

A. .8 cm

B. .5cm

C. 1.5cm

D. 2.2cm

Answer: A

Rationale:
The facial nerve crosses the zygomatic arch .8cm to 3.5cm anterior to the anterior
border of the external auditory canal. Therefore, the incision through temporoparietal
fascia and periosteum should not be more than be .8cm anterior to the anterior border of
the external auditory canal.

Reference:
Ellis, E, Zide, M: Surgical Approaches to the Facial Skeleton, Baltimore, 1995,
Williams and Wilkins, p. 165.

The American Board of Oral and Maxillofacial Surgery 124


2007 Oral and Maxillofacial Surgery Self Assessment Tool (OMSSAT)

120. Condylotomy for symptomatic internal derangement has the greatest positive clinical effect
with regard to improvement in:

A. pre-existing malocclusion.

B. position of a non-reducing disc.

C. symptoms associated with Wilkes stage II and III internal derangement.

D. symptoms associated with Wilkes stage IV and V internal derangement.

Answer: C

Rationale:
Pain and diet are significantly improved 3 years after modified condylotomy for internal
derangement. Disc position is improved at least 70% of the time with disc displacement
with reduction. Non-reducing disc displacement is unaffected by condylotomy. Late
stage internal derangement joints with degenerative joint disease (Wilkes Stage IV, V)
have less satisfactory pain relief after modified condylotomy. Although modified
condylotomy can be applied simultaneously to the patient with Class III malocclusion
and symptomatic internal derangement this is not well documented and a represents a
more novel application of condylotomy.

Reference:
Hall HD, Navarro ZE, Gibbs SJ: Prospective study of modified condylotomy for
treatment of nonreducing disc displacement. Oral Surg Oral Med Oral Pathol Oral
Radiol Endod 2000;89:147-158.
Hall HD, Navarro ZE, Gibbs SJ: One- and three-year prospective outcome study of
modified condylotomy for treatment of reducing disc displacement. J Oral Maxillofac
Surg 2000;58:7-17.

The American Board of Oral and Maxillofacial Surgery 125


2007 Oral and Maxillofacial Surgery Self Assessment Tool (OMSSAT)

121. Compared to no disc replacement, the main advantage of placing a dermis graft in
conjunction with TMJ discectomy is:

A. enhanced pain relief.

B. improved range of motion.

C. decreased joint noise.

D. decreased regressive condylar remodeling.

Answer: C

Rationale:
Interpositional dermis can decrease joint noise after discectomy. No clinical evidence
of joint sounds such as crepitus was found in 33 of 35 operated joints evaluated an
average of 2 years after discectomy with placement of a dermis interpositional graft.
Dermis grafting has no effect on pain response, range of motion or the incidence of
regressive condylar remodeling following discectomy.

Reference:
Dimitrioulis G: The use of dermis grafts after discectomy for internal derangement of
the temporomandibular joint JOMS. J. Oral Maxillofac Surg 2005;63:75

The American Board of Oral and Maxillofacial Surgery 126


2007 Oral and Maxillofacial Surgery Self Assessment Tool (OMSSAT)

122. Which branch of cranial nerve VII is at greatest risk with the pre-auricular approach to the
temporomandibular joint?

A. Buccal

B. Zygomatic

C. Cervical

D. Temporal

Answer: D

Rationale:
The most commonly injured branch of the facial nerve is the temporal branch with
preauricular approach to the TMJ. Less commonly, the zygomatic branch may be
injured.

Reference:
Vallerand WP, Dolwick MF. Complication of temporomandibular joint surgery. Oral
Maxillofac Clin North Am 1990;2:481-8.

The American Board of Oral and Maxillofacial Surgery 127


2007 Oral and Maxillofacial Surgery Self Assessment Tool (OMSSAT)

123. What statement describes a functional deficit associated with injury to the frontal branch of
the facial nerve?

A. Lagophthalmos

B. Exposure keratitis

C. Brow ptosis

D. Hypertropia

Answer: C

Rationale:
Injury to the frontal branch of the facial nerve causes frontalis weakness and brow
ptosis. Lagophthalmos and exposure keratitis are associated with injury to the
zygomatic branch of the facial nerve. Hypertropia refers to globe elevation from
impaired globe depressor function, as may be seen in injury to the inferior division of
the oculomotor nerve.

Reference:
Quinn P: Color atlas of temporomandibular joint surgery; 1998 Mosby St. Louis p.31

The American Board of Oral and Maxillofacial Surgery 128


2007 Oral and Maxillofacial Surgery Self Assessment Tool (OMSSAT)

124. During intra-articular TMJ surgery, preservation of the medial capsule, may avoid damage
to which vascular structure?

A. The internal maxillary artery

B. The masseteric artery

C. The deep temporal artery

D. The middle meningeal artery

Answer: D

Rationale:
The middle meningeal artery is immediately medial to the temporomandibular joint.
Preservation of the medial joint capsule will minimize injury to this vascular structure.
Options a,b,c, are incorrect because none of these vessels course immediately adjacent
to the medial portion of the TMJ.

Reference:
Mohamed,S:Developmental and Clinical Anatomy and Physiology of the
Temporomandibular Joint. In Fonseca RJ, Bays RA, Quinn PD (eds):Oral and
Maxillofacial Surgery Vol 4, 2000 WB Saunders, Philadelphia, p 4.

The American Board of Oral and Maxillofacial Surgery 129


2007 Oral and Maxillofacial Surgery Self Assessment Tool (OMSSAT)

125. Compared to discectomy, a more common complication of modified mandibular


condylotomy is?

A. Change in occlusion

B. Post operative joint noise

C. Intraarticular adhesions

D. Condylar arthrosis

Answer: A

Rationale:
Condylar “sag” and loss of ramus height can lead to a posterior occlusal prematurity on
the side of a modified condylotomy. Occlusion changes following discectomy, if any,
are mild and transient. Joint noise is more common after discectomy because of
intraarticular scarring/adhesions.

Reference:
Bays RA :Surgery for Internal Derangement. In Fonseca RJ, Bays RA, Quinn PD
(eds): Oral and Maxillofacial Surgery Vol 4, 2000, WB Saunders, Philadelphia, p. 297.

The American Board of Oral and Maxillofacial Surgery 130


2007 Oral and Maxillofacial Surgery Self Assessment Tool (OMSSAT)

126. During TMJ arthrotomy, what structure is most likely to be the source of bleeding during
entry into the superior joint space?

A. Superior head of the lateral pterygoid muscle

B. Inferior head of the lateral pterygoid muscle

C. Retrodiscal tissue

D. Posterior temporalis muscle

Answer: C

Rationale:
The retrodiscal tissue has a robust blood supply and can be encountered as the lateral
capsule is incised and the joint space entered.
a. and b. are incorrect because they are encountered at the very anterior and medial
region of the joint.
d. is incorrect because it is usually encountered before the joint capsule has been
opened.

Reference:
Mohamed,S:Developmental and Clinical Anatomy and Physiology of the
Temporomandibular Joint. In Fonseca RJ, Bays RA, Quinn PD (eds):Oral and
Maxillofacial Surgery Vol 4 2000 WB Saunders, Philadelphia, p 6.

The American Board of Oral and Maxillofacial Surgery 131


2007 Oral and Maxillofacial Surgery Self Assessment Tool (OMSSAT)

127. During modified mandibular condylotomy surgery what is the maximum distance that the
oscillating blade must extend medially beyond ramus at the level of the midsigmoid notch
to place the internal maxillary artery at least risk of injury?

A. 20 mm

B. 15 mm

C. 10 mm

D. <5 mm

Answer: D

Rationale:
The mean distance from the internal maxillary artery to the midsigmoid ramus is only
3.3 mm. In addition to the risk of injury to the internal maxillary artery, a branch of the
internal maxillary artery, the masseteric artery, passes through the sigmoid notch to
supply the masseter muscle. Both arteries are at risk during modified condylotomy.

Reference:
Fujimura K, Segami N, Kobayashi S. Anatomical study of the complications of
intraoral vertico-sagittal ramus osteotomy. J Oral Maxillofac Surg 64:384-389, 2006

The American Board of Oral and Maxillofacial Surgery 132


2007 Oral and Maxillofacial Surgery Self Assessment Tool (OMSSAT)

128. During intra-articular TMJ surgery, preservation of the medial capsule, may avoid damage
to which vascular structure?

A. The internal maxillary artery

B. The masseteric artery

C. The deep temporal artery

D. The middle meningeal artery

Answer: D

Rationale:
The middle meningeal artery is immediately medial to the temporomandibular joint.
Preservation of the medial joint capsule will minimize injury to this vascular structure.
Options a,b,c, are incorrect because none of these vessels course immediately adjacent
to the medial portion of the TMJ.

Reference:
Mohamed,S:Developmental and Clinical Anatomy and Physiology of the
Temporomandibular Joint. In Fonseca RJ, Bays RA, Quinn PD (eds):Oral and
Maxillofacial Surgery Vol 4, 2000 WB Saunders, Philadelphia, p 4.

The American Board of Oral and Maxillofacial Surgery 133


2007 Oral and Maxillofacial Surgery Self Assessment Tool (OMSSAT)

129. The surface of the mandibular condyle is covered with what material?

A. Hyaline cartilage

B. Synovium

C. Fibrous connective tissue

D. Fibrocartilage

Answer: D

Rationale:
The TMJ articular surface of the condyle covered with fibrocartilage. Fibrocartilage is
best suited to loading of the articular surface, while also allowing remodeling. A, B, and
C do not form the covering of the condyle.

Reference:
Mohamed, S: Developmental and Clinical Anatomy and Physiology of the
Temporomandibular Joint. In Fonseca RJ, Bays RA, Quinn PD (eds): Oral and
Maxillofacial Surgery. Vol 4, 2000, WB Saunders, Philadelphia, p 8.

The American Board of Oral and Maxillofacial Surgery 134


2007 Oral and Maxillofacial Surgery Self Assessment Tool (OMSSAT)

130. Marked degenerative TMJ disease has been associated with which condition?

A. Multiple sclerosis

B. Hemophilia

C. Cystic fibrosis

D. Grave’s disease

Answer: B

Rationale:
Hemarthrosis is a common consequence of moderate to severe coagulopathies.
Recurrent bleeding into the temporomandibular joint may occur in afflicted individuals
resulting in marked degradation of articular surfaces of the mandibular condyle and
temporal bone, and fibrosis. The mechanism responsible for these degenerative
changes involves the generation of free radicals from Fe++ released from degraded
hemoglobin.
Some patients with multiple sclerosis may complain of radiating, electric shock-like
pain limited to trigeminal dermatomes. Though the clinical presentation is
characteristic of trigeminal neuralgia, it is conceivable that this pain could be mistaken
as TMJ arthralgia, particularly if the pain was felt in the dermatome of CN V3.
However, multiple sclerosis is not associated with an increased incidence of
degenerative TMJ disease. Likewise, there is no evidence that the prevalence of
degenerative TMJ disease is higher in patients with either Grave's disease or cystic
fibrosis.

Reference:
Kaneda T. Nagayama M. Ohmori M. Minato F. Nakajima J. Shikimori M. Hemarthrosis
of the temporomandibular joint in a patient with hemophilia B: report of case. J Oral
Surg 1979;37:513-4.

Nishioka GJ. Van Sickels JE. Tilson HB. Hemophilic arthropathy of the
temporomandibular joint: review of the literature, a case report, and discussion. Oral
Surg, Oral Med, Oral Pathol 1988; 65:145-50.

The American Board of Oral and Maxillofacial Surgery 135


2007 Oral and Maxillofacial Surgery Self Assessment Tool (OMSSAT)

131. Which of the following molecules has been implicated in the pathogenesis of degenerative
TMJ diseases and can be inhibited by tetracyclines?

A. Tumor necrosis factor alpha (TNF)

B. Interleukin 1 beta (IL-1)

C. Matrix metalloproteinase 1 (MMP1)

D. Cathepsin D

Answer: C

Rationale:
Matrix metalloproteinases (MMPs) are enzymes which degrade molecules in the
extracellular matrices (e.g., collagens, proteoglycans) of articular tissues of the TMJ.
To date, four MMPs have been isolated from diseased human TMJs (MMP1, MMP2,
MMP3, MMP9). These matrix degrading enzymes require zinc as a co-factor for
activity. Tetracyclines inhibit these enzymes by chelation of zinc. Recent animal
studies and limited clinical trials indicate that tetracyclines may limit progression of
some degenerative TMJ diseases by inhibition of these matrix degrading enzymes.
TNF and IL-1 are potent cytokines that have also been isolated from symptomatic
human TMJs. These signaling molecules are believed to induce the synthesis of MMPs
and other molecules that are involved in tissue degradation and inflammation.
However, the activities of TNF and IL-1are not affected by tetracyclines.
Cathepsin D is an intracellular endopeptidase that is involved in the intracellular
degradation of molecules. Cathepsin D has been identified in synoviocytes of the TMJ.
The activity of this enzyme is not affected by tetracyclines.

Reference:
Milam SB. Pathogenesis of degenerative temporomandibular joint arthritides.
Odontology 2005;93):7-15.

Zardeneta G. Milam SB. Lee T. Schmitz JP. Detection and preliminary characterization
of matrix metalloproteinase activity in temporomandibular joint lavage fluid.
International Journal of Oral & Maxillofacial Surgery. 1998;27:397-403.

Haskin CL. Milam SB. Cameron IL. Pathogenesis of degenerative joint disease in the
human temporomandibular joint. Critical Reviews in Oral Biology & Medicine.
1995;6:248-77.

Kiyoshima T. Tsukuba T. Kido MA. Tashiro H. Yamamoto K. Tanaka T.


Immunohistochemical localization of cathepsins B and D in the synovial lining cells of
the normal rat temporomandibular joint. Archives of Oral Biology. 1993;38:357-9.

The American Board of Oral and Maxillofacial Surgery 136


2007 Oral and Maxillofacial Surgery Self Assessment Tool (OMSSAT)

132. A 46 year-old female presents with bilateral preauricular pain, restricted jaw movement
and slight apertognathia. CT and MR images reveal marked erosions of the articular
surfaces of both TMJs with mild joint effusions. Distal interphalangeal joint swelling and
pitting fingernails (i.e., nail dystrophy) are observed on general examination. The patient
has a negative rheumatoid factor, but is HLA-Bw38 and HLA-DR4 positive, anemic, and
has a mildly elevated erythrocyte sedimentation rate. What is most likely diagnosis is?

A. Psoriatic arthritis

B. Gout

C. Anklyosing spondylitis

D. Reiter syndrome

Answer: A

Rationale:
Psoriatic arthritis is an erosive polyarthritic disease occurring in patients with psoriasis
(psoriatic skin lesions). Psoriasis is a genetically determined disease, and HLA-Bw38,
HLA-DR4, and HLA-DR7 are genetic markers for psoriatic arthritis. The disease is
slightly more prevalent in women. Several forms of the arthropathy appear to exist.
Clinical findings suggestive of psoriatic arthritis include skin lesions, distal
interphalangeal joint swelling, and pitting fingernails (i.e., nail dystrophy). This
condition often results in an elevated ESR and anemia.
Crystal deposition diseases are believed to be genetic or acquired metabolic disorders
that result in the formation of crystals derived from uric acid (gout) or calcium
pyrophosphate dihydrate (pseudogout) in tissues of affected joints. Gout is typically a
slowly progressive monoarthritide (in initial phases) with restricted mandibular range of
motion. Gout predominantly affects older men (peak incidence in 5th decade of life).
Focal crystal deposits, termed tophi, are classically found in the helix or antihelix of the
ear in chronic cases. These deposits normally appear as small papular masses that
rarely ulcerate or become infected. Leukocytosis, elevated ESR, and elevated uric acid
are commonly observed on serology.
Apertognathia is also rare. CBC, ESR, uric acid and anti-nuclear antibody studies are
typically within normal limits.
Reiter syndrome is an asymmetrical inflammatory oligoarthritic condition consisting of
painless mucocutaneous lesions, conjunctivitis, urethritis, and arthritis affecting
predominantly young men (9:1-99:1 male to female ratio) who have recently
experienced an infection (Chlamydia, Campylobactor, Shigella, Salmonella, HIV).
Onset is typically 1-4 weeks after infection.

Reference:

The American Board of Oral and Maxillofacial Surgery 137


2007 Oral and Maxillofacial Surgery Self Assessment Tool (OMSSAT)

Espinoza LR. Cuellar ML. Silveira LH. Psoriatic arthritis. Current Opinion in
Rheumatology.1992;4:470-8.

Kononen M. Wolf J. Kilpinen E. Melartin E. Radiographic signs in the


temporomandibular and hand joints in patients with psoriatic arthritis. Acta
Odontologica Scandinavica. 1991;49):191-6.

Barthelemy I. Karanas Y. Sannajust JP. Emering C. Mondie JM. Gout of the


temporomandibular joint: pitfalls in diagnosis. Journal of Cranio-Maxillo-Facial
Surgery. 2001;2):307-10.

Bomalaski JS. Jimenez SA. Erosive arthritis of the temporomandibular joint in Reiter's
syndrome. Journal of Rheumatology. 1984;11:400-2.

The American Board of Oral and Maxillofacial Surgery 138


2007 Oral and Maxillofacial Surgery Self Assessment Tool (OMSSAT)

133. This 29 year-old male presented with slowly progressing malocclusion and chin deviation.
What is the most likely diagnosis based on CT images?

A. Osteochondroma

B. Giant cell lesion

C. Osteosarcoma

D. Condylar hyperplasia

Answer: A

Rationale:
This represents a slow growing well-circumscribed benign lesion. Osteosarcoma would
be poorly circumscribed and the margins of the native condyle would be indistinct.
Giant cell lesions typically would present as a destructive radiolucency rather than a
proliferative, radiopaque process. In condylar hyperplasia the native condyle is usually
dramatically enlarged in all dimensions.

Reference:
Stanton, D Stewart, J: Tumors of the Temporomandibular Joint. In Fonseca, R (ed):
Oral and Maxillofacial Surgery, Vol. 4. Philadelphia, WB Saunders, 2000, pp 369-373.

Waldron, C: Bone Pathology. In Neville, C (ed): Oral and Maxillofacial Pathology.


Philadelphia, WB Saunders, 1995, p 472.

The American Board of Oral and Maxillofacial Surgery 139


2007 Oral and Maxillofacial Surgery Self Assessment Tool (OMSSAT)

134. Select from the list the appropriate laboratory evaluations for a 12 year old girl with 8
weeks of intermittent daily fever, macular rash and swelling/pain involving the wrists,
knees, ankles and TM joints:

ANA (antinuclear antibodies)


CRP (C-Reactive protein)
CBC (complete blood count)
C-ANCA(cytoplasmic antineutrophil cytoplasmic antibody)
RF(rheumatoid factor)
ESR(erythrocyte sedimentation rate)
LFTs(liver function tests)

A. CBC, LFTs, CRP, ANA, RF, ESR

B. LFTs, CRP, ANA, RF, ESR

C. C-ANCA, CRP, ANA, RF, ESR

D. C-ANCA, LFTs, CRP, ANA, RF, ESR

Answer: A

Rationale:
The clinical scenario suggests systemic juvenile rheumatoid arthritis (Still's disease).
Systemic JRA is marked by fever, rash, hepato-splenomegally, anemia, leukocytosis,
and any number of joints may be affected, including the TMJ.
Laboratory studies should include complete blood count, liver function tests (e.g AST,
ALT), erythrocyte sedimentation rate and CRP. Minor AST and ALT enzyme
elevations are often present. The ESR is typically elevated in children with systemic
onset JRA and may exceed 100 mm/hr; normal sedimentation rates are rare when the
disease is active. The level of C-reactive protein, another acute phase reactant, is usually
elevated as well. Screening for rheumatic diseases should include rheumatoid factor and
antinuclear antibodies. Antinuclear antibodies and RF are rarely seen in systemic onset
JRA. Their presence suggests alternative diagnoses. RF, for example, may be associated
with Sjögren's syndrome or the early onset of adult type RA in teenagers. Both RF and
ANA may be present in children with mixed connective tissue disease. However, these
conditions rarely mimic systemic onset JRA.
Anti-neutrophil cytoplasmic antibodies with a cytoplasmic staining pattern (c-ANCA)
have been found to have a high degree of sensitivity and specificity for Wegener's
granulomatosis and is not an appropriate test to order in this child.

The American Board of Oral and Maxillofacial Surgery 140


2007 Oral and Maxillofacial Surgery Self Assessment Tool (OMSSAT)

Reference:
Rimon A, Zeharia A, Mimouni M, Mukamel M: Temporomandibular joint pain in
pediatrics: the clinical approach and differential diagnosis; Pediatric Rheumatology
Online, Volume 3, Number 3 2005.
(http://www.pedrheumonlinejournal.org/may-june05/TMJ-Pain.htm)

Cassidy, JT, Petty, RE. Juvenile rheumatoid arthritis. In: Cassidy, JT, Petty, RE (Eds)
Textbook of Pediatric Rheumatology, 4th ed, 2001 W.B. Saunders Company,
Philadelphia, p. 218.

The American Board of Oral and Maxillofacial Surgery 141


2007 Oral and Maxillofacial Surgery Self Assessment Tool (OMSSAT)

135. A 20 year-old female presented with a previous history of TMJ clicking, isolated left TM
joint pain, and limited opening. Physical examination now reveals left pre-auricular
tenderness and crepitus. You obtain the following radiograph. The most likely diagnosis
is:

A. psoriatic arthritis.

B. avascular necrosis.

C. osteoarthrosis.

D. rheumatoid arthritis.

Answer: C

Rationale:
The history of past TMJ clicking suggests a long-standing internal derangement. The
radiographic changes are that of arthrosis, consistent with a previously clicking joint.
Psoriatic and rheumatoid arthritis are both systemic arthropathies that involve multiple
joints. Avascular necrosis may possibly be diagnosed my MR imaging but has no
characteristic panoramic radiographic appearance.

Reference:
Silverstein K. Arthritis of the temporomandibular joint. In Fonseca, R (ed): Oral and
Maxillofacial Surgery Vol. 4. Philadelphia, WB Saunders, 2000 p. 73-92.

The American Board of Oral and Maxillofacial Surgery 142


2007 Oral and Maxillofacial Surgery Self Assessment Tool (OMSSAT)

136. A healthy 65 year-old male presents with a chief complaint of generalized intermittent
headache of 40 years duration. Over the past year his left TMJ is making a grinding noise
when he eats. He denies any TMJ pain or locking. What is your clinical diagnosis from this
patient’s history and image?

A. Avascular necrosis of the mandibular condyle

B. Degenerative joint disease

C. Synovial chondromatosis

D. Rheumatoid arthritis

Answer: B

Rationale:
a. The concept of “avascular necrosis” of the TMJ condyle is unproven.
b. This is a case of a patient with long standing “headache” unrelated to TMJ since he
has no TMJ pain or dysfunction. He does however exhibit primary, age related DJD of
the TMJ which if misdiagnosed can lead to unnecessary surgical intervention.
c. The patient has no TMJ pain and dysfunction which would be hallmarks of
chondromatosis along with joint noise.
d. The patient has no history of systemic arthritis, TMJ pain or dysfunction.
Involvement of the TMJ as the initial joint in systemic rheumatoid arthritis is rare.
Unilateral TMJ rheumatoid is also rare.

The American Board of Oral and Maxillofacial Surgery 143


2007 Oral and Maxillofacial Surgery Self Assessment Tool (OMSSAT)

Reference:
Dolwick, MF: Temporomandibular Joint Disc Displacement: A Clinical Perspective. In,
Sessle, BJ; Bryant, PS; Dionne, RA (Eds): Temporomandibular Disorders and Related
Pain Conditions. Progress in Pain Research and Management. Vol 4 1995,IASP Press.
Seattle. pp.79 – 87.
Milam, SB: Articular Disc Displacements and Degenerative Temporomandibular Joint
Disease. In: Sessle, BJ; Bryant, PS; Dionne, RA (Eds): Temporomandibular Disorders
and Related Pain Conditions. Progress in Pain Research and Management. Vol 4 1995
IASP Press. Seattle. Vol 4 pp.89 – 112.

The American Board of Oral and Maxillofacial Surgery 144


2007 Oral and Maxillofacial Surgery Self Assessment Tool (OMSSAT)

137. Which of the following is a prominent feature of type I condylar hyperplasia?

A. Ankylosis

B. Deviation of chin to contralateral side

C. open bite on ipsilateral side

D. bilateral crossbite

Answer: B

Rationale:
Ankylosis and crossbite are not usually features of type I condylar hyperplasia. Open
bite is a prominent feature of type II condylar hyperplasia. Deviation of the chin is a
common feature of type I but not type II condylar hyperplasia.

Reference:
Stanton D, Stewart, J: Tumors of the Temporomandibular Joint. In Fonseca, R (ed):
Oral and Maxillofacial Surgery Vol. 4. Philadelphia, WB Saunders, 2000, p 365.

The American Board of Oral and Maxillofacial Surgery 145


2007 Oral and Maxillofacial Surgery Self Assessment Tool (OMSSAT)

138. A 25 year-old female presents to the emergency department with a medial pole fracture of
the right TMJ condyle. Treatment should consist of which of the following?

A. Open reduction and internal fixation followed by maxillomandibular fixation for 14


days

B. Open reduction with internal fixation only

C. Closed reduction with maxillomandibular fixation for 4 weeks to maintain proper


posterior vertical height.

D. Medical management with soft diet only

Answer: D

Rationale:
The medial pole of the TMJ condyle by position does not function as a primary
determinant of mandibular vertical height. As such, open or closed surgical approaches
for such a fracture are unwarranted; and the injury is managed with soft diet and range
of motion exercises.

Reference:
Sicher, H; and DuBrul, EL: Temporomandibular articulation. In, Oral Anatomy (6th
Ed.) CV Mosby Co. St. Louis. pp. 160 - 191, 1975.
Quinn, PD: Color Atlas of Temporomandibular Joint Surgery. CV Mosby, St. Louis,
1998.

Assael, LA: Hard Tissue Trauma. In, Temporomandibular Joint Disorders - Diagnosis
and Treatment. Kaplan, AS and Assael, LA (Eds) WB Saunders, Philadelphia, 1992.
pp.224-237.

Mercuri, LG: Temporomandibular Joint Disorders. In: Kwon, PH; and Laskin, DM
(eds) Clinician' Manual of Oral and Maxillofacial Surgery 3rd ed. 2001 Quintessence.
Chicago.

Kobinson DA, Hohn FI: Traumatic injuries. In, Laskin DM, Greene CS, Hylander WL
(eds): Temporomandibular Disorders: An Evidence-Based Approach to Diagnosis and
Treatment. 2006 Quintessence. Chicago.

The American Board of Oral and Maxillofacial Surgery 146


2007 Oral and Maxillofacial Surgery Self Assessment Tool (OMSSAT)

139. A 25 year-old female presents to the emergency department with a medial pole fracture of
the right TMJ condyle. Treatment should consist of which of the following?

A. Open reduction and internal fixation followed by maxillomandibular fixation for 14


days

B. Open reduction with internal fixation only

C. Closed reduction with maxillomandibular fixation for 4 weeks to maintain proper


posterior vertical height.

D. Medical management with soft diet only

Answer: D

Rationale:
The medial pole of the TMJ condyle by position does not function as a primary
determinant of mandibular vertical height. As such, open or closed surgical approaches
for such a fracture are unwarranted; and the injury is managed with soft diet and range
of motion exercises.

Reference:
Sicher, H; and DuBrul, EL: Temporomandibular articulation. In, Oral Anatomy (6th
Ed.) CV Mosby Co. St. Louis. pp. 160 - 191, 1975.

Quinn, PD: Color Atlas of Temporomandibular Joint Surgery. CV Mosby, St. Louis,
1998.

Assael, LA: Hard Tissue Trauma. In, Temporomandibular Joint Disorders - Diagnosis
and Treatment. Kaplan, AS and Assael, LA (Eds) WB Saunders, Philadelphia, 1992.
pp.224-237.

Mercuri, LG: Temporomandibular Joint Disorders. In: Kwon, PH; and Laskin, DM
(eds) Clinician' Manual of Oral and Maxillofacial Surgery 3rd ed. 2001 Quintessence.
Chicago.

Kobinson DA, Hohn FI: Traumatic injuries. In, Laskin DM, Greene CS, Hylander WL
(eds): Temporomandibular Disorders: An Evidence-Based Approach to Diagnosis and
Treatment. 2006 Quintessence. Chicago.

The American Board of Oral and Maxillofacial Surgery 147


2007 Oral and Maxillofacial Surgery Self Assessment Tool (OMSSAT)

140. Which of the following ligaments restricts posterior movements of the mandibular
condyle?

A. Stylomandibular ligament

B. Sphenomandibular ligament

C. Capsular ligament

D. Lateral ligament

Answer: D

Rationale:
The lateral ligament, also known as the temporomandibular ligament, is a fan-shaped
structure that runs obliquely in a posterior and caudal direction from the lateral aspect of
the articular eminence to the posterior aspect of the mandibular condyle and lateral
margin of the articular disk. The lateral ligament is composed of two elements: an outer
oblique portion and an inner horizontal portion. The lateral ligament limits inferior and
posterior displacement of the mandibular condyle, and to a limited extent the posterior
displacement of the articular disk.

The capsular ligament is a relatively thin structure originating from the rim of the
glenoid fossa. This structure extends inferiorly to join the periosteum of the condylar
process below the condylar head. The capsular ligament encloses the joint cavity,
which is divided into a superior joint space and an inferior joint space by the articular
disk. The stylomandibular ligament extends from the styloid process to the angle of the
mandible. The sphenomandibular ligament, a remnant of the perichondrium of
Meckel's cartilage, runs from the spine of the sphenoid bone to the mandibular lingula.
The stylomandibular and sphenomandibular ligaments provide resistance to extreme
anterior, lateral, and caudal displacements of the mandible, offering indirect support of
the TMJ.

Reference:
Ten Cate, AR, Gross and micro anatomy. In: Zarb, GA, et al (eds) Temporomandibular
Joint and Masticatory Muscle Disorders. 1994 Mosby, Munksgaard, p. 58.

The American Board of Oral and Maxillofacial Surgery 148


2007 Oral and Maxillofacial Surgery Self Assessment Tool (OMSSAT)

141. A 32 year-old patient presents with a history of increasing frequency of open lock. He
now requires treatment in the local emergency room to be able to close his mouth. What
intervention is most likely to provide long-term relief of his condition?

A. Injection of the lateral pterygoid muscle with botulinum toxin

B. Eminectomy

C. Intra-articular injection of a sclerosing agent

D. Temporalis muscle scarification

Answer: B

Rationale:
Comprehensive literature review reveals that the of the interventions listed, only
eminectomy is likely to provide long-term control of recurrent dislocation.
Distractors: Botulinum toxin may be of benefit, but the literature is scanty and the
effect temporary. Sclerosing agents have a disproportionately low success rate.
Temporalis scarification does not have reported long-term follow-up.

Reference:
Shorey, CW and Campbell, JH. Temporomandibular joint dislocation. Oral Surg,
Oral Med Oral Pathol Oral Diag Endod 2000;89:662-8

The American Board of Oral and Maxillofacial Surgery 149


2007 Oral and Maxillofacial Surgery Self Assessment Tool (OMSSAT)

142. Agenesis of the mandibular condyle can be the result of which of the following?

A. Malformation of the third branchial arch

B. Damage to the stapedial artery during development in utero

C. Folic acid deficiency

D. Niacin deficiency

Answer: B

Rationale:
A. Agenesis of the mandibular condyle is also called “First Arch Syndrome” since the
first, not the third, branchial arch is involved.
B. This is the suspected etiology of hemifacial microsomia and related disorders, based
on the work of Poswillo.
C. and d. Vitamin deficiencies have nothing to do with this entity.

Reference:
Mercuri, LG: Temporomandibular Joint Disorders. In: Kwon, PH; and Laskin, DM
(Eds): Clinician Manual of Oral and Maxillofacial Surgery 3rd ed. 2001 Quintessence
Publishing Co. Chicago.

Poswillo, D: The pathogenesis of Treacher-Collins Syndrome (mandibular-facial


dysostosis). Br J Oral Surg 13:1, 1975.

The American Board of Oral and Maxillofacial Surgery 150


2007 Oral and Maxillofacial Surgery Self Assessment Tool (OMSSAT)

Troulis MJ, Kaban LB: Congenital and developmental anomalies. In, Laskin DM,
Greene CS, Hylander WL (eds): Temporomandibular Disorders: An Evidence-Based
Approach to Diagnosis and Treatment. 2006 Quintessence. Chicago.

The American Board of Oral and Maxillofacial Surgery 151


2007 Oral and Maxillofacial Surgery Self Assessment Tool (OMSSAT)

143. A 45 year-old male presents with recurrent left TMJ ankylosis and maximum interincisal
opening of 10mm. 5 years ago he underwent closed reduction of a mandibular condyle
fracture. One year after the closed reduction he underwent autogenous costochondral
reconstruction for TM joint ankylosis. Subsequently he underwent repeat costochondral
reconstruction for recurrent ankylosis. In this situation, which treatment demonstrates the
lowest rate of recurrent ankylosis?

A. Gap arthroplasty with the insertion of a permanent silicone rubber block

B. Gap arthroplasty with the insertion of a temporalis muscle rotation flap and autogenous
costochondral graft

C. Gap arthroplasty with the insertion of autogenous fat graft

D. Gap arthroplasty with the insertion of an alloplastic TMJ prosthesis

Answer: D

Rationale:
A. Silicone rubber implants are not a satisfactory material for use as a permanent
interpositional implant in the TMJ. Silicone rubber has poor wear properties in load-
bearing joints.
B. Autogenous bone grafting for re-ankylosis of the TMJ has poor outcome.
C. Autogenous fat grafting has been used successfully to prevent re-ankylosis of the
TMJ, but not as a sole entity.
D. Re-ankylosis of the TMJ is best managed by total TMJ alloplastic reconstruction and
autogenous fat grafting to minimize heterotopic bone formation around the prosthesis.

The American Board of Oral and Maxillofacial Surgery 152


2007 Oral and Maxillofacial Surgery Self Assessment Tool (OMSSAT)

Reference:
Mercuri, LG: Temporomandibular Joint Disorders. In: Kwon, PH; and Laskin, DM
(eds). Clinician Manual of Oral and Maxillofacial Surgery 3rd ed. 2001 Quintessence
Publishing Co. Chicago.

Anonymous.Recommendations for Management of Patients With Temporomandibular


joint Implants. J Oral Maxillofac Surg 1993; 51:1164-72.

Mercuri LG, Wolford LM, Sanders B, et al: Long-term follow-up of the CAD/CAM
patient fitted total temporomandibular joint reconstruction system. J Oral Maxillofac
Surg 2002;60:1440.

Mercuri LG: TMJ Concepts Patient Fitted Total TMJ Reconstruction Prostheses. Oral
Maxillofacial Surg Clinics North Am 2000;12:73.

The American Board of Oral and Maxillofacial Surgery 153


2007 Oral and Maxillofacial Surgery Self Assessment Tool (OMSSAT)

144. A 7 year-old child with a history of facial trauma at age 5 presents with oral opening
limited to 4 mm and left-sided mandibular deviation. The surgical intervention most
likely to provide functional and esthetic rehabilitation into adulthood is:

A. alloplastic total joint reconstruction.

B. alloplastic condylar replacement.

C. gap arthroplasty with auricular cartilage graft.

D. autogenous costochondral graft.

Answer: D

Rationale:
The history and image indicate bony ankylosis of the temporomandibular joint in
a growing child. When reconstructing the temporomandibular joint in children,
the surgeon must provide both functional anatomy and an environment in which
growth may occur. Donor sites with growth potential are preferred. Alloplastic
total or partial joint reconstruction or gap arthroplasty will not support growth.

Reference:
American Association of Oral and Maxillofacial Surgeons. Parameters and
Pathways: Clinical Practice Guidelines for Oral and Maxillofacial Surgery. J Oral
Maxillofac Surg, Version 3.0, TMJ-6, 2001

The American Board of Oral and Maxillofacial Surgery 154


2007 Oral and Maxillofacial Surgery Self Assessment Tool (OMSSAT)

145. During surgical removal of an ankylosed condyle brisk arterial bleeding is encountered and
localized to the region medial to the lateral pterygoid muscle. Which terminal branch of
the carotid artery is the most likely source of the bleeding?

A. Internal maxillary artery

B. Anterior deep temporal artery

C. Masseteric artery

D. Middle meningeal artery

Answer: D

Rationale:
The middle meningeal artery is a branch of the internal maxillary artery, coursing
medially from the internal maxillary artery ascending between the sphenomandibular
ligament and the lateral pterygoid muscle, passing through the foramen spinosum. The
average distance from the glenoid fossa to the middle meningeal artery is 2.4 mm. The
close proximity of the middle meningeal artery to the TMJ places it at risk during
complex procedures that require dissection medical to the mandibular condyle.

Reference:
Cillo JE, Sinn D, Truelson JM. Management of middle meningeal and superficial
temporoal artery hemorrhage from total temporomandibular joint replacement surgery
with a gelatin-based hemostatic agent. J Craniofac Surg 2005;16:309-12

The American Board of Oral and Maxillofacial Surgery 155


2007 Oral and Maxillofacial Surgery Self Assessment Tool (OMSSAT)

146. A 60 year-old post-menopausal female presents to the emergency department with the
complaint of transient but sudden, severe unilateral headache. Her past medical history is
significant for hypertension, tobacco and alcohol abuse. Her neurological examination is
normal. This clinical history is most compatible with which diagnosis.

A. variant migraine.

B. cerebral vasospasm.

C. giant cell arteritis.

D. subarachnoid hemorrhage.

Answer: D

Rationale:
A retrospective analysis of patients with subarachnoid hemorrhage (SAH) suggests that
minor episodes with sudden headache may precede rupture of an aneurysm. Up to 40%
of patient will experience a sentinel headache a week or more prior to rupture of a
cerebral aneurysm. This woman's history is compatible with sentinel headache for a
“leaking” aneurysm. Risk factors for the development of SAH are 1) smoking,
2)hypertension, and 3)alcohol, 4)estrogen depletion. Variant migraine includes such
findings as hemiplegia, brain stem symptoms (vertigo, etc.), transient blindness,
ophthalmoplegia, not present in this woman. Giant cell arteritis is associated with
unilateral temporal headache, jaw claudication, and possible symptoms of retinal
ischemia, not manifested in this woman. Cerebral vasospasm is a complication of SAH
leading to neurologic decline in the days following the SAH.

Reference:
Ingall, T, Asplund, K, Mahonen, M, Bonita, R. A multinational comparison of
subarachnoid hemorrhage epidemiology in the WHO MONICA stroke study. Stroke
2000; 31:1054

Edlow, JA, Caplan, LR. Avoiding pitfalls in the diagnosis of subarachnoid hemorrhage.
N Engl J Med 2000; 342:29

Lipton, RB, Bigal, ME, Steiner, TJ, et al. Classification of primary headaches.
Neurology 2004; 63:427

Hunder, GG. Giant cell arteritis and polymyalgia rheumatica. In: Textbook of
Rheumatology, 6th ed, Kelly, WN, Harris, ED, Ruddy, S, et al (Eds), WB Saunders
Company, Philadelphia, 2001

The American Board of Oral and Maxillofacial Surgery 156


2007 Oral and Maxillofacial Surgery Self Assessment Tool (OMSSAT)

147. The diagnosis of sympathetically maintained pain caused by injury to the inferior alveolar
nerve can be confirmed by which test?

A. Regional nerve block with local anesthesia

B. Somatosensory evoked potential recording

C. Magnetic resonance neurography of the mandible

D. Stellate ganglion blockade of the superior stellate ganglion

Answer: D

Rationale:
Sympathetic maintained pain (SMP) results from tonic activity in myelinated
mechanoreceptor afferents, whose activity is induced by sympathetic efferent actions
on sensory receptors, and this afferent input causes tonic firing in previously sensitized
wide-dynamic (WDR) or mulitreceptive neurons that are part of a central nociceptive
pathway. A burning painful sensation results from this chain of actions.
Insert

Reference:
Gregg JM: Studies of traumatic neuralgia in the maxillofacial region: symptom
complexes and response to microsurgery. J Oral Maxillofac Surg 48:135, 1990
Roberts W: A hypothesis on the physiological basis for causalgia and related pain. Pain
24:297, 1986

Raja SN. Turnquist JL. Meleka S. Campbell JN. Monitoring adequacy of alpha-
adrenoceptor blockade following systemic phentolamine administration. [Clinical Trial.
Controlled Clinical Trial. Journal Article] Pain. 1996;64):197-204, 1996

The American Board of Oral and Maxillofacial Surgery 157


2007 Oral and Maxillofacial Surgery Self Assessment Tool (OMSSAT)

148. A 38 year-old female with a long history of periodic migraine headache has, for the past 3
months, experienced an insidious increase in headache frequency to near daily headache
and frequent awakening with headache. For the past 4 months she has tried to manage her
headache with daily over-the-counter acetaminophen, aspirin, and caffeine containing
analgesics. Her present condition is best described by:

A. variant migraine.

B. rebound-type transformed migraine.

C. cluster headache.

D. hemiplegic migraine.

Answer: B

Rationale:
Variant migraine refers anyone of several sub-classifications of migraine such as retinal
migraine, hemiplegic migraine, etc. Cluster headache is a distinct clinical syndrome
affecting men more than woman, with attacks accompanied by ipsilateral lacrimation,
nasal discharge, ptosis, conjunctival injection, and pupillary change. Hemiplegic
migraine is a migraine variant characterized by motor and sensory symptoms that are
unilateral. Symptoms can last longer than the headache itself and complete recovery
may take weeks in some cases. Transformed headache is a transformation from
intermittent migraine headache attacks to daily or almost daily headache. The excessive
and frequent use of symptomatic medications including ergot derivatives, analgesics
and high-dose NSAID's can result in the insidious transformation to daily headaches.
This has been also referred to as medication-overuse headache where daily headache is
induced and maintained by the medications used to relieve such pain.

Reference:
Dodick DW. Chronic daily headache. N Engl J Med 2006;354:158-65
Saper JR. Headache disorders. Med Clin North Am 1999; 83:663-90

Corbett, JJ. Neuro-ophthalmic complications of migraine and cluster headaches. Neurol


Clin 1983; 1:973

The American Board of Oral and Maxillofacial Surgery 158


2007 Oral and Maxillofacial Surgery Self Assessment Tool (OMSSAT)

149. Excruciating unilateral periorbital burning or piercing pain episodes lasting 30 – 60


minutes, associated with ipsilateral lacrimation are characteristic of:

A. migraine headache.

B. trigeminal neuralgia.

C. tension headache.

D. cluster headache.

Answer: D

Rationale:
Cluster headache has a 6:1 male predilection, is most commonly periorbital and
excruciating, and is strictly unilateral. Episodes last from 15 – 180 minutes, but most
commonly from 30 – 60 minutes. Occurrence is from one every other day to eight per
day, and they tend to “cluster” over time; each cluster lasts from 1 – 2 months, with one
or two clusters per year. Pain is commonly described as burning, piercing, or neuralgic,
and ipsilateral autonomic symptoms such as lacrimation, rhinorrhea, miosis, and/or
ptosis occur. Migraine typically exhibits throbbing unilateral pain lasting from 4 – 72
hours, nausea, photophobia, and exacerbation by physical activity. Trigeminal neuralgia
can be differentiated from cluster headache because pain is transient and not associated
with autonomic signs/symptoms. Tension headache is commonly bandlike and
bilateral, lasts 30 minutes to seven days, of mild to moderate intensity that does not
prohibit activity, and not associated with nausea/vomiting or photophobia.

Reference:
Smetana, GW. The diagnostic value of historical features in primary headache
syndromes: a comprehensive review. Arch Int Med 200;160:2729-37.

The American Board of Oral and Maxillofacial Surgery 159


2007 Oral and Maxillofacial Surgery Self Assessment Tool (OMSSAT)

150. A 40 year-old male is referred for evaluation of facial pain and headache. He describes a
long history of temporal/periorbital region headaches. His headaches generally last 30
minutes, occur 3-4 times per day over a 2 month period, followed by several headache-free
months. Other symptoms characteristic of this man’s headache diagnosis are:

A. photophobia, phonophobia.

B. lacrimation and rhinorrhea.

C. meningismus.

D. nausea and vomiting.

Answer: B

Rationale:
The four categories of primary headache include migraine, tension-type, cluster
headache, and other trigeminal autonomic cephalgias and other primary headaches. This
man's history is most compatible with cluster headache. Cluster headache is a distinct
clinical syndrome affecting men more than woman, with attacks accompanied by
ipsilateral lacrimation, nasal discharge, ptosis, conjunctival injection, and pupillary
change. The features most predictive of migraine are nausea, photophobia,
phonophobia, exacerbation with physical activity. Meningismus is associated such
acute events as subarachnoid hemorrhage.

Reference:
Saper JR. Headache disorders. Med Clin North Am 1999; 83:663-90
Smetana, GW. The diagnostic value of historical features in primary headache
syndromes. A comprehensive review. Arch Intern Med 2000; 160:2729.

Bahra, A, May, A, Goadsby, PJ. Cluster headache: A prospective clinical study with
diagnostic implications. Neurology 2002; 58:354.

Lipton, RB, Bigal, ME, Steiner, TJ, et al. Classification of primary headaches.
Neurology 2004; 63:427.

The American Board of Oral and Maxillofacial Surgery 160


2007 Oral and Maxillofacial Surgery Self Assessment Tool (OMSSAT)

151. A healthy 65 year-old male is referred with a 3 week history of severe, “shock-like”,
paroxysmal pain in the right tonsillar and ear region associated with swallowing cold
liquids. He recently experienced a syncopal event where he was noted to be profoundly
bradycardic with one of the attacks. This man’s history is most compatible with which
diagnosis:

A. carotodynia.

B. Eagle’s syndrome.

C. glossopharyngeal neuralgia.

D. sphenopalatine neuralgia.

Answer: C

Rationale:
Glossopharyngeal neuralgia is defined as paroxysmal pain in areas innervated by cranial
nerves IX and X. Pain may be experienced in the ear, larynx, tonsillar region, and
tongue. It is almost always unilateral. Triggers include chewing, swallowing,
coughing, speaking, and yawning. Severe attacks have been associated with
bradycardia/syncope through the vagal motor nucleus. Carotodynia is pain that
originates in the carotid artery. The pain is usually unilateral, provoked by swallowing,
coughing, or neck movement, and typically lasts for several days to months. Eagle's
syndrome is loosely associated with ossification of the stylohyoid ligament. Dull
pharyngeal and neck pain possibly provoked with swallowing and head turning is
described. Palpation of the tonsillar fossa and/or hyoid may illicit pain. Sphenopalatine
(greater superficial petrosal) neuralgia presents as unilateral, episodic pain in the
perinasal region, associated with nasal congestion. Some case may be attributable to
cluster headache and may not represent a unique diagnostic entity.

Reference:
Rushton, JG, Stevens, JC, Miller, RH. Glossopharyngeal (vagoglossopharyngeal)
neuralgia: a study of 217 cases. Arch Neurol 1981; 38:201

Elias, J, Kuniyoshi, R, Valadao, W, et al. Glossopharyngeal neuralgia associated with


cardiac syncope. Arq Bras Cardiol 2002; 78:515

Fini, G, Gasparini, G, Filippini, F, et al. The long styloid process syndrome or Eagle's
syndrome. J Craniomaxillofac Surg 2000; 28:123

The American Board of Oral and Maxillofacial Surgery 161


2007 Oral and Maxillofacial Surgery Self Assessment Tool (OMSSAT)

Bruyn, G. Sphenopalatine neuralgia (Slyder). In: Vinken, PJ, Bruyn, GW, Klawanus,
HC, Rose, FC (eds.), Handbook of Clinical Neurology, Amsterdam, Elselvier, 1986, p
475-482

Raskin, NH. Headache, 2d ed, New York, Churchill Livingstone, 1988, p 353-357

The American Board of Oral and Maxillofacial Surgery 162


2007 Oral and Maxillofacial Surgery Self Assessment Tool (OMSSAT)

152. A healthy 75 year-old female presents with new temporal region headache and trismus. On
examination she is mildly febrile (38oC), with focal tenderness in the temporal and
occipital regions. Examination of her visual fields reveals a deficit in the temporal field of
her right eye. What is the most likely diagnosis?

A. Polymyalgia rheumatica

B. Polyarteritis nodosa

C. Giant cell arteritis

D. Variant (retinal) migraine

Answer: C

Rationale:
Giant cell arteritis(GCA) is a vasculitis of large and medium size vessels that most
prominently involves the cranial arteries. Symptoms in this patient compatible with
GCA are fever, temporal head pain and trismus. Polymyalgia rheumatica (PMR) is
closely linked to GCA but characterized by aching and morning stiffness in the
shoulder, hip girdles, neck and torso. Polyarteritis nodosa (PAN) is a systemic
necrotizing vasculitis effecting small and medium size vessels. Patients with PAN
present with fatigue, weakness, fever, arthralgias, and polyneuropathy. Systems
affected include neurologic, skin, renal, and gastrointestinal. Retinal or ocular migraine
is a rare variant migraine characterized by repeated attacks of monocular scotomata or
blindness lasting less than one hour, associated with or followed by headache.

Reference:
Saper JR. Headache disorders. Med Clin North Am 1999; 83:663-90
Corbett, JJ. Neuro-ophthalmic complications of migraine and cluster headaches. Neurol
Clin 1983; 1:973

Hunder, GG. Giant cell arteritis and polymyalgia rheumatica. In: Textbook of
Rheumatology, 6th ed, Kelly, WN, Harris, ED, Ruddy, S, et al (Eds), WB Saunders
Company, Philadelphia, 2001

Conn, DL. Polyarteritis. In: Rheumatology, Klippel, JH, Dieppe, PA (Eds), Mosby, St
Louis, 1994

The American Board of Oral and Maxillofacial Surgery 163


2007 Oral and Maxillofacial Surgery Self Assessment Tool (OMSSAT)

153. A neuropathic pain condition that is characterized by summation, delayed onset, overshoot
and aftersensation to a mechanical stimulus represents which condition?

A. allodynia

B. anesthesia dolorosa

C. hyperalgsia

D. hyperpathia

Answer: D

Rationale:
Neuropathic pain is described by the patient as pain that is severe and “out of
proportion” to the stimulus or pain that is often spontaneous, continuous, or lingering in
nature. Stimulus induced neuropathic pains are:
1. Allodynia: painful response to normally non-painful stimulus. For example, moving
a brush across the zone of injury causes a very painful stimulation of the area.
Normally a brush moving across the adjacent uninjured area is non-painful. Axons of
cutaneous afferent neurons become sensitive to mechanical stimulus. Local
inflammatory mediators at or around the injured nerve terminals or axons decrease the
threshold for activation and the response magnitude to constant stimuli may increase.
This sensitization, which is caused by endogenous bradykinin, prostaglandin and others
may enhance mechanosensitivity of afferent neurons.
2. Anesthesia Dolorosa: painful response in a zone of complete anesthesia. For
example, pain is still reported in a zone that has no response to tactile, thermal, painful
stimuli and not resolved with peripheral nerve blocks of the same zone. These findings
indicate a central origin of the pain response.
3. Hyperalgesia: enhanced pain response. For example, thermal pain tolerance over
the mental fold in a healthy adult is between 48o and 50o C. If the recorded pain
tolerance in the same area is now 45o C, then the patient has hyperalgesia. They have
pain response and sensitivity, but it is abnormally low in threshold and therefore
previously non-painful levels of a painful stimulus (cold, hot, pressure, etc.) is now
painful and normal just detectable painful stimulus is enhanced.
4. Hyperpathia: a painful syndrome characterized by an abnormally painful reaction to
a stimulus, especially a repetitive stimulus, as well as an increased threshold. For
example, in applying a neurosensory test protocol; a series of about ten tactile pulses
with Semmes Weinstein monofilaments slowly administered (1/sec) followed by a 1
minute rest period. This protocol is used to identify delayed onset in pain. The pain
may alternatively begin during the stimulation but continue for appreciable durations
exceeding 1minute after the stimulation is terminated and is an example of
aftersensation. If the pain begins during the stimulation, the approximate number of
pulses required for its elicitation should be recorded to describe the summation. If the

The American Board of Oral and Maxillofacial Surgery 164


2007 Oral and Maxillofacial Surgery Self Assessment Tool (OMSSAT)

intensity of the pain is reported to increase with continued stimulation, overshoot is


present. One or more observations of delayed onset, aftersensation, overshoot, or
summation are sufficient for a diagnosis of a hyperpathic pain condition.
5. Sympathetic Maintained Pain (SMP): Sympathetic maintained pain results from
tonic activity in myelinated mechanoreceptor afferents, whose activity is induced by
sympathetic efferent actions on sensory receptors, and this afferent input causes tonic
firing in previously sensitized wide-dynamic (WDR) or mulitreceptive neurons that are
part of a central nociceptive pathway. A burning painful sensation results from this
chain of actions. A regional nerve block with local anesthesia would result in the
reduction of the mechanical sensitivity but not the tonic firing of WDR neuron
generated impulses. Brush-stoke directional testing of the involved area would
demonstrated mechanical allodynia but would not be able to distinguish peripheral
mechanical allodynia from SMP pain.

Reference:
Dworkin RH: An overview of neuropathic pain: syndromes, symptoms, signs, and
several mechanisms. Clin J Pain 2002;18:343.

Coderre TJ, Melzack R: Cutaneous hyperalgesia: contributions of the peripheral and


central nervous systems to the increase in pain sensitivity after injury. Brain Res
1987;404:95.

Ochs G, Schenk M, Struppler A: Painful dysaesthesia following peripheral nerve injury:


a clinical and electrophysiological study. Brain Res 1987;496:228.

Michaelis M, Vogel C, Blenk K-H, Arnarson A, Janig W: Inflammatory mediators


sensitize acutely axotomized nerve fibers to mechanical stimulation in the rat. J
Neurosci 1998;18:7581.

Zuniga JR, Essick GK: A contemporary approach to the clinical evaluation of


trigeminal nerve injuries. Oral Maxillofac Surg Clinic North Am 1992;4:353.

The American Board of Oral and Maxillofacial Surgery 165


2007 Oral and Maxillofacial Surgery Self Assessment Tool (OMSSAT)

154. After injury to the mental nerve, a painful neuroma has formed. Surgical exploration,
reveals no viable nerve distal to the neuroma. The recommended surgical treatment of a
painful neuroma when there is no distal nerve element is resection of the neuroma and:

A. intraneural steroid application.

B. intraneural alcohol injection.

C. fasciculectomy with lateralization of the inferior alveolar nerve.

D. fasciculectomy with closure using an epineurial sleeve.

Answer: D

Rationale:
Painful neuromas form following injuries associated with third, fourth and fifth degree
nerve injuries and following inadequate surgical repair of severed nerves. Surgical
treatment of painful neuromas should include the exposure and separation of the
neuroma from adjacent tissues so that the proximal and distal nerves can be adequately
exposed for microscopic examination. After the neuroma is resected, the surgical
treatment chosen depends upon the presence or absence of distal nerves. In the absence
of adequate distal nerve tissue to repair, the treatment should be to reduce or eliminate
the recurrence of a painful neuroma.

Intraneural steroid applications may induce a chemical neuritis and increase the pain
associated with the neuroma and is largely contraindicated for treatment of neuroma
Resection of the neuroma will leave the proximal end intact so that regrowth of the
neuroma is highly likely. Neuromas form when amputated axons develop sprouting
axons (neurites) in such large numbers that they collectively form a mass of
unorganized unmyelinated axons that are sensitive to environmental conditions, both
physical and chemical.

Resection of the neuroma and transposition and implantation of the amputated end into
muscle tissue is one surgical option in this case. Muscle tissue reduces neurite
formation, in part, due to acetylcholine receptor or negative RATIONALE to neurite
growth. Resection of the neuroma and lateralization of the inferior alveolar nerve does
not accomplish the end goal of implantation into muscle tissue.

In a funiculectomy procedure of a peripheral nerve, the fascicles are exposed and


sectioned and allowed to retract within the epineurial sleeve. The epineurium is then
brought out to full length, and a double or triple suture ligature is placed around the
epineurial sleeve so that the cut nerve is within its epineurial space. The proposed
mechanism of action of funiculectomy and epineurial sleeve method is to reduce

The American Board of Oral and Maxillofacial Surgery 166


2007 Oral and Maxillofacial Surgery Self Assessment Tool (OMSSAT)

fibroproliferation of nerve and supporting tissue and reduce the size of a resulting
neuroma. (Figure 2)

Reference:
Williams HB: The painful stump neuroma and its treatment. In Microreconstruction of
Nerve Injuries. 1987 WB Saunders, Philadelphia, pp 165-171

Zuniga JR: Surgical management of trigeminal neuropathic pain. Atlas Oral Maxillofac
Surg Clinic North Am 2001;9:59.

The American Board of Oral and Maxillofacial Surgery 167


2007 Oral and Maxillofacial Surgery Self Assessment Tool (OMSSAT)

155. Therapy for chronic central neuropathic pain after injury to the inferior alveolar nerve
following third molar surgery includes:

A. external neurolysis with primary neurorrhaphy.

B. anti-convulsants.

C. non-steroidal antiinflammatory medication.

D. low energy laser application.

Answer: B

Rationale:
Pharmacological interventions for the management of neuropathic pain very widely in
terms of their safety, efficacy, tolerability, titration requirements and cost. Opioids
were previously considered ineffective but recent clinical trials demonstrate they offer
effective pain relief in many instances but can be subject to diversion and abuse.
Tricyclic antidepressants (amitriptyline) also have proven efficacy but tolerability
concerns and titration complexities can limit their practicality for use. Selected anti-
convulsants agents have proven effectiveness in neuropathic pain (Clonazepam and
gabapentin). Finally, nonsteroidal anti-inflammatory agents are inexpensive, well
tolerated, and easy to administer, but lack efficacy in neuropathic pain.

Peripheral nerve surgery has little role in the management of centrally mediated pain.

Reference:
Mendell JR, Sahenk Z: Painful sensory neuropathy. N Engl J Med 2003;348:1243.
Rowbotham MC, Twilling L, Davies PS, Reisner L, Taylor K, Mohr D: Oral opioid
therapy for chronic peripheral and central neuropathic pain. N Eng J Med 2003;l J Med
348:1223.

Gilron I, Bailey JM, Tu D, Holden RR, Weaver DF, Houlden RL: Morphine,
gabapentin, or their combination for neuropathic pain. N Eng J Med 2005;352:1324.
Gregg JM: Medical management of traumatic neurpathies. Oral Maxillofac Surg Clinic
North Am 2001;13:343.

The American Board of Oral and Maxillofacial Surgery 168


2007 Oral and Maxillofacial Surgery Self Assessment Tool (OMSSAT)

156. 3 months after third molar extraction, a patient is experiencing dysesthesia in the
distribution of the inferior alveolar nerve. Which neuropathic pain characteristics and
clinical findings would support microsurgical intervention?

A. Mechanical allodynia that is resolved with a proximal local anesthetic nerve block

B. Burning hyperpathia that is resolved with a stellate ganglion block

C. Mechanical allodynia that is not resolved with a proximal local anesthetic nerve block

D. Burning hyperpathia in which only the hyperpathia is resolved with a proximal local
anesthetic nerve block.

Answer: A

Rationale:
Diagnostic nerve blocks are employed to weigh the relative contributions of peripheral
and central factors to the neuropathic pain condition. Should peripheral nerve blocks
proximal to the site of injury significantly decrease the pain complaint, then peripheral
factors are accepted as responsible for the discomfort. Should the peripheral nerve
block fail to significantly decrease the pain, central factors are likely responsible for the
pain and discomfort. For sympathetic maintained pain conditions, blockade of
sympathetic afferent nerves results in elimination of the pain. When SMP involves the
cranial nerves, a stellate ganglion block is utilized to relieve SMP pain.
The literature supports the concept that invasive treatments (microsurgery of the inferior
alveolar nerve) are more frequently helpful (slightly better than chance improvement) in
peripheral neuropathic pain states than in central or sympathetic maintained pain states.

Reference:
Campbell JN, Raja SN, Meyer RA: Painful sequelae of nerve injury. In Dubner R,
Gebhart GF, Bond MR (eds): Proceedings of the Fifth World Congress on Pain. 1988
Elsevier, Amsterdam, p 135

Gregg JM: Studies of traumatic neuralgia in the maxillofacial region: symptom


complexes and response to microsurgery. J Oral Maxillofac Surg 1990;48:135.
Zuniga JR: Surgical management of trigeminal neuropathic pain. Atlas Oral Maxillofac
Surg Clinic North Am 2001;9:59.

The American Board of Oral and Maxillofacial Surgery 169


2007 Oral and Maxillofacial Surgery Self Assessment Tool (OMSSAT)

157. A neonatologist requests an evaluation of a week old baby who has an oral mass that was
present at birth. On examination the mass measures 2 x 1.5 cm, is pink in color and is
attached to the anterior maxillary gingiva by a narrow stalk. What is the most likely
diagnosis for this lesion?

A. Congenital granular cell tumor

B. Infantile hemangioma

C. Arteriovenous malformation

D. Melanotic neuroectodermal tumor of infancy

Answer: A

Rationale:
The primary clinical features that are consistent with a congenital granular cell tumor
and rule out the other possibilities are presence at birth and that the lesion is attached by
a stalk. A hemangioma and arteriovenous malformation can be present at birth but
would not be attached by a stalk. A melanotic neuroectodermal tumor of infancy is not
present at birth.

Reference:
Kaban LB, Troulis MJ: Pediatric Oral and Maxillofacial Surgery, 2004, p 260-262;
Marx RE, Stern D: Oral and Maxillofacial Pathology, 2003, p 427.

The American Board of Oral and Maxillofacial Surgery 170


2007 Oral and Maxillofacial Surgery Self Assessment Tool (OMSSAT)

158. Appropriate soft tissue margins for resection of squamous cell carcinoma of the oral cavity
are:

A. 0.25 – 0.75 cm.

B. 1.0 – 1.5 cm.

C. 2.0 – 2.5 cm.

D. 3.0 – 3.5 cm.

Answer: B

Rationale:
Resection of at least 1.0 cm beyond the clinical or microscopic disease is indicated in
the treatment of squamous cell carcinoma of the oral cavity. 0.25 cm is an inadequate
margin and while some authors have suggested resections of 1.0-1.5 cm, a 2.0 cm
margin or beyond for this disease process is not indicated.

Reference:
MacIntosh R. Oral Malignant Disease: Management and Investigational Directions.
Fonseca Oral and Maxillofacial Surgery. Williams Stewart, 152-235, 2000.

The American Board of Oral and Maxillofacial Surgery 171


2007 Oral and Maxillofacial Surgery Self Assessment Tool (OMSSAT)

159. Which of the following clinical findings is most consistent with a diagnosis of chronic,
diffuse sclerosing osteomyelitis of the mandible?

A. Pain

B. Lack of expansion of the mandible

C. The presence of an oro-cutaneous fistula

D. Sclerotic bone located at the alveolus of the anterior mandible

Answer: A

Rationale:
Chronic diffuse sclerosing osteomyelitis (CDSO) of the mandible is a poorly
understood clinical entity that is most often confused with florid osseous dysplasia.
Typically, CDSO is very painful, associated with expansion of the mandible, located
diffusely within the mandible from alveolus to the inferior border, and is unassociated
with an oro-cutaneous fistula.

Reference:
Marx R.E., Carlson E.R., Smith B.R. and Torayo N.: Isolation of Actinomyces Species
and Eikenella Corrodens from Patients with Chronic Diffuse Sclerosing Osteomyelitis.
J Oral Maxillofac Surg, 1994; 52:26-33.

The American Board of Oral and Maxillofacial Surgery 172


2007 Oral and Maxillofacial Surgery Self Assessment Tool (OMSSAT)

160. A 15 year-old female presents with an expansile mass of the anterior mandible. The CT
scan reveals calcifications and a well corticated lesion. The incisional biopsy demonstrates
the presence of the microcalcifications and shows epithelial-lined duct-like structures in a
background of swirling spindle-shaped epithelial cells. Which of the following diagnoses
is supported by this description?
161.

A. Ameloblastoma, multilocular

B. Ameloblastoma, unilocular

C. Adenomatoid odontogenic tumor

D. Odontogenic myxoma

Answer: C

Rationale:
The clinical history and histopathology is characteristic of adenomatoid odontogenic
tumor that is the correct answer. By definition ameloblastomas are odontogenic tumors
that do not produce calcifications and do not have duct-like structures (epithelial lined
structures with lumin). In addition, the age of patient, site of lesion and imaging is not
characteristic of both ameloblastomas. Therefore options A & B are incorrect. Option
D is also incorrect since odontogenic myxoma is a mesenchymal tumor.

The American Board of Oral and Maxillofacial Surgery 173


2007 Oral and Maxillofacial Surgery Self Assessment Tool (OMSSAT)

Reference:
Abaza, NA: Ultrastructural features and biologic aspects of odontogenic cysts and
tumors. In: Oral and Maxillofac Surg Clin of North Amer, L. Gold, (editor), 1994, P
523-577. W.B. Saunders, Philadelphia.

Rick CM: Adenomatoid odontogenic tumor. In: Oral & Maxillofac Surg Clin North
Amer., Odontogenic Tumors, DK White (editor), Saunders, Philadelphia; 2004, p333-
354.

The American Board of Oral and Maxillofacial Surgery 174


2007 Oral and Maxillofacial Surgery Self Assessment Tool (OMSSAT)

162. This photomicrograph is illustrative of which of the following diagnoses?

A. Calcifying odontogenic cyst.

B. Squamous cell carcinoma

C. Desmoplastic ameloblastoma

D. Parakeratotic odontogenic cyst

Answer: A

Rationale:
The photomicrograph reveals well illustrated “ghost cells” (sheets of epithelial cell that
only reveals shadow outlines of cell membranes and shadow of the cell nuclei). There
are also cuboidal epithelium and stellate-like cells. This is characteristic histology of
calcifying odontogenic cysts. Option B is incorrect since there is no sign of cellular
atypia and the “ghost cells” should not be confused with keratin pearls. Option C is
incorrect since “ghost cells” are not reported as components of the histopathology of
desmoplastic ameloblastoma. In addition, the glassy and dense stroma of desmoplastic
ameloblastoma is lacking from the micrograph. Option D is incorrect; there is not a 7-8
cell layer thick, corrugated and parakeratinized luminal epithelial lining illustrated.

The American Board of Oral and Maxillofacial Surgery 175


2007 Oral and Maxillofacial Surgery Self Assessment Tool (OMSSAT)

Reference:
Kramer IRH, Pindborg JJ, Shear M: Histological typing of odontogenic tumors, 2nd
edit, Berlin, 1992, Springer-Verlag.

Tomich CE: Calcifying odontogenic cyst and dentinogenic ghost cell tumor. In: Oral
Maxillofac Surg Clin North Amer; Odontogenic Tumors, DK White (editor) Saunders,
Philadelphia, 2004, p391-397

The American Board of Oral and Maxillofacial Surgery 176


2007 Oral and Maxillofacial Surgery Self Assessment Tool (OMSSAT)

163. This photomicrograph represents a lesion that is best described as which of the following?

A. Benign tumor with no recurrence after curettage

B. Benign tumor with high recurrence rate after curettage

C. Resection is not required for cure

D. Benign tumor with high frequency of malignant transformation

Answer: B

Rationale:
The photomicrograph is a classic picture of odontogenic myxoma with its stellate and
spindle myxoblasts with marked myxomatous stroma. The examinee is expected to
know that odontogenic myxoma is a benign tumor, that is aggressive with high
recurrence rates after curettage, and resection is required for a cure. The tumor is also
reported to have a very low incidence of malignant transformation as myxosarcoma.
Therefore, the best answer is option B. Options A, C & D are, therefore incorrect.

Reference:
Abaza, NA: Ultrastructural features and biologic aspects of odontogenic cysts and
tumors. In: Oral Maxillofac Surg Clin North Amer, L. Gold (editor), WB Saunders,
Philadelphia, 1994, p 523-577.

Marx RE, and Stern D: Oral and Maxillofac Pathol: A RATIONALE for diagnosis and
treatment. Quintessence Pub Comp, Chicago, 2003, p637-703.

The American Board of Oral and Maxillofacial Surgery 177


2007 Oral and Maxillofacial Surgery Self Assessment Tool (OMSSAT)

164. This photomicrograph is from a biopsy of a lesion in the angle of the mandible of a 17
year-old patient. Which of the following represents the correct diagnosis?

A. Follicular ameloblastoma

B. Desmoplastic ameloblastoma

C. Ameloblastic fibroma

D. Metastatic squamous cell carcinoma

Answer: C

Rationale:
The correct answer is option C because the photomicrograph shows two actively
proliferating tissue types: 1. cords and nests of hyperplastic odontogenic epithelium in
2: a hypercellular stroma. The ameloblastic fibroma is a mixed tumor. Option A is
incorrect: there are no well developed follicles of odontogenic epithelium with
peripheral palisading columnar cells and inner stellate-reticulum cells and the stroma is
usually hypocellular in follicular ameloblastoma. Option B is incorrect because in this
lesion, the stroma is characteristically acellular with the characteristic and marked wide
bands of glassy collagenous tissue. Option D is incorrect since there is no cellular
atypia of both the epithelium and connective tissue. In rare instances only the
connective component of the tumor may have malignant transformation and the lesion
is called ameloblastic fibrosarcoma.

Reference:

The American Board of Oral and Maxillofacial Surgery 178


2007 Oral and Maxillofacial Surgery Self Assessment Tool (OMSSAT)

Gardner DG: The mixed odontogenic tumors. Oral Surg Oral Med Pathol 58:166,
1984.
Cohen DH: Ameloblastic fibroma ameloblastic fibro-odontoma and odontoma. In: Oral
Maxillofac Surg North Amer: Odontogenic Tumors, DW White: Saunders,
Philadelphia, 2004, 16: 375-384.

The American Board of Oral and Maxillofacial Surgery 179


2007 Oral and Maxillofacial Surgery Self Assessment Tool (OMSSAT)

165. Which of the following is true regarding the clinical presentation of a majority of central
giant cell lesions?

A. Present in the maxilla in infants

B. Present in the mandible in young adults

C. Present in the mandible in elderly adults

D. Present as multiple maxillary and mandibular lesions in adults

Answer: B

Rationale:
Central giant cell lesions occur primarily in young adults. Occurrence in infants and the
elderly is rare. The most common site of involvement is the mandible (70%). Central
giant cell lesions are almost always solitary lesions.

Reference:
Oral and Maxillofacial Surgery Clinics of North America, November 1997, p 657.

The American Board of Oral and Maxillofacial Surgery 180


2007 Oral and Maxillofacial Surgery Self Assessment Tool (OMSSAT)

166. The incidence of which of the following neoplasms is increased in a patient with multiple
cysts of the jaws and skin cancer?

A. Liposarcoma

B. Renal cell carcinoma

C. Osteosarcoma

D. Medulloblastoma

Answer: D

Rationale:
The question deals with the basal cell nevus syndrome (Gorlin's) with its known
systemic manifestations as an autosomal dominant with 100% penetrance and located
on chromosome 9. Several structural anomalies and diseases have been associated with
the syndrome and their frequencies have been tabulated. Options A, B & C are not
recorded as part of the syndrome. Option D is correct, although the frequency has been
low; it is clinically significant and, therefore, should be recognized in working-up
patients with the syndrome.

Reference:
Rogerson KC: Gorlin's syndrome: an update on diagnosis and management, Oral
Maxillofac Surg Clin North Amer 3:155-164, 1991.

Marx RE, and Stern D: Oral and Maxillofac Pathol: A RATIONALE for diagnosis and
treatment. Quintessence Pub Comp, Chicago, 2003, p602-606.

The American Board of Oral and Maxillofacial Surgery 181


2007 Oral and Maxillofacial Surgery Self Assessment Tool (OMSSAT)

167. This photomicrograph is from a mass of the anterior maxilla of a 41 year old patient. The
CT scan revealed a mixed radiolucent/radiopaque lesion with ill-defined margins. Which of
the following is the correct diagnosis?

A. Desmoplastic ameloblastoma.

B. Ameloblastic fibroma

C. Odontogenic myxoma

D. Ameloblastic fibrosarcoma

Answer: A

Rationale:
The correct option is A. The most significant histologic feature of desmoplastic
ameloblastoma is the hypocellular and markedly hyalinized collagenous stroma that
compresses island of odontogenic epithelium. Option B is incorrect; the stroma should
be hypercellular and actively proliferating. Option C is incorrect because the
odontogenic myxoma is a connective tissue tumor where the cells are stellate and
spindle shaped with mucoid or mucinous stroma, occasional fibroblast may be present.
Occasional inert islands of odontogenic epithelium may be present. Option D is also
incorrect. Although malignant transformation of the actively proliferating hypercellular
stroma may occur, its incidence is very low. The photomicrograph does not show any
atypical changes of these cells suggesting any malignant transformation.

The American Board of Oral and Maxillofacial Surgery 182


2007 Oral and Maxillofacial Surgery Self Assessment Tool (OMSSAT)

Reference:
Yoshimura Y, Saito H: Desmoplastic variant of ameloblastoma: report of a case and
review of the literature, J Oral Maxillofac Surg 84:1231, 1990.

The American Board of Oral and Maxillofacial Surgery 183


2007 Oral and Maxillofacial Surgery Self Assessment Tool (OMSSAT)

168. Which of the following best describes the histopathologic features of the lining of the
glandular odontogenic cyst?

A. Non-keratinized stratified squamous epithelium with palisading basal cell layer and
parakeratotic luminal surface.

B. Non-keratinized stratified squamous epithelium with mucus cells and cilia.

C. Keratinized stratified squamous epithelium with ghost cells and stellate-like epithelial
cells.

D. Keratinized stratified squamous epithelium with keratohyaline granules and luminal


fibrillar keratin.

Answer: B

Rationale:
Option B is the correct answer. The stem of the question diagnoses the cyst as
glandular and the only option containing cells that can produce “glands” is option B
where epithelium contains mucus cells and cilia. Option A describes parakeratotic
odontogenic cyst; Option C describes calcifying odontogenic cyst; and option D is that
of an orthokeratinized odontogenic cyst.

Reference:
Hussain K, Edmondson HB, Browne RM: Glandular odontogenic cysts: Diagnosis and
treatment. Oral Surg Oral Med Oral Pathol, 1995: 79:593-602.

The American Board of Oral and Maxillofacial Surgery 184


2007 Oral and Maxillofacial Surgery Self Assessment Tool (OMSSAT)

169. A 13 year-old female is referred for evaluation of an expanding mass involving the
maxilla. The parents report that the mass has been slowly expanding over the last few
years and has now recently become more noticeable. Upon reviewing her medical history
it is discovered that the patient started menstruating at the age of 9. Physical examination
reveals an area of pigmentation with irregular borders involving the patient’s lower back
and buttock. A panoramic radiograph demonstrates a mixed radiolucent/radiopaque lesion
of the maxilla, as well as a smaller lesion involving the mandible. You obtain a chest
radiograph that demonstrates a lesion involving the left third and fourth ribs. Which of the
following is the most likely diagnosis?

A. Jaffe-Lichtenstein syndrome

B. Cleidocranial dysplasia

C. McCune-Albright syndrome

D. Fragile X syndrome

Answer: C

Rationale:
The patient's clinical and radiographic examinations demonstrate multiple skeletal
lesions suggestive of polyostotic fibrous dysplasia. The presence of café-au-lait
pigmentation is consistent with either Jaffe-Lichtenstein syndrome or McCune-Albright
syndrome. Endocrine abnormalities, such as precocious puberty as observed in this
patient, are only observed in McCune-Albright syndrome.

Reference:
Kaban LB, Troulis MJ: Pediatric Oral and Maxillofacial Surgery, 2004, p 222; Marx
RE, Stern D: Oral and Maxillofacial Pathology, 2003, p 749-750; Oral and
Maxillofacial Surgery Clinics of North America, November 1997, p 731.

The American Board of Oral and Maxillofacial Surgery 185


2007 Oral and Maxillofacial Surgery Self Assessment Tool (OMSSAT)

170. A 45 year-old female is referred for evaluation of multiple loose teeth. A review of her
history reveals recent onset depression. She also reports increased thirst and bone pain.
The panoramic radiograph demonstrates multiple mandibular radiolucencies. The most
likely diagnosis based on her history and radiographic findings is:

A. Langerhans cell disease.

B. multiple odontogenic keratocysts associated with nevoid basal cell carcinoma


syndrome.

C. multiple myeloma.

D. hyperparathyroidism.

Answer: D

Rationale:
The differential diagnosis for multiple mandibular radiolucencies should include all of
the above. However, the history of depression, thirst and bone pain is clearly consistent
with hyperparathyroidism.

Reference:
Marx RE, Stern D: Oral and Maxillofacial Pathology, 2003, p 739.

The American Board of Oral and Maxillofacial Surgery 186


2007 Oral and Maxillofacial Surgery Self Assessment Tool (OMSSAT)

171. Which of the following represents the most appropriate treatment of an ameloblastic
fibroma?

A. Resection with preservation of inferior border of mandible

B. En-bloc resection including the inferior border of mandible

C. Enucleation and curettage

D. Marsupialization

Answer: C

Rationale:
Option C is correct since the lesion is benign with very low or no recurrence after
adequate enucleation. Options A, B & D are incorrect. Option A may be selected for
follicular ameloblastoma; Option B may be selected for desmoplastic ameloblastoma;
and Option D is not appropriate for a diagnosis of ameloblastic fibroma which
represents a true odontogenic tumor that will not respond to such therapy.

Reference:
Gardner, DB: the mixed odontogenic tumors. Oral Surg Oral Med, Oral Pathol, 58:166-
1984.
Marx RE, and Stern D: Oral and Maxillofac Patho: A Rationale for diagnosis and
treatment. Quintessence Pub Comp, Chicago, 2003, p 637-703.

The American Board of Oral and Maxillofacial Surgery 187


2007 Oral and Maxillofacial Surgery Self Assessment Tool (OMSSAT)

172. A 40 year-old African American female presents for evaluation of mixed


radiolucent/radiopaque, round, non-corticated lesions involving the apices of the anterior
mandibular teeth. The patient is asymptomatic. All of the mandibular anterior teeth test
vital. Which of the following would be the recommended management?

A. Obtain a CT scan to further evaluate the extent of the lesion.

B. Obtain a tissue biopsy to confirm the diagnosis.

C. No treatment is necessary.

D. Perform curettage of the lesions and send tissue for pathologic diagnosis.

Answer: C

Rationale:
The clinical presentation is entirely consistent with periapical cementodysplasia. The
patient is asymptomatic. No treatment is necessary.

Reference::
Oral and Maxillofacial Surgery Clinics of North America, November 1997, p 644-646.

The American Board of Oral and Maxillofacial Surgery 188


2007 Oral and Maxillofacial Surgery Self Assessment Tool (OMSSAT)

173. This panoramic radiograph is that of an 18-year-old patient with an incisional biopsy
diagnosis of intraluminal unicystic ameloblastoma. Which of the following is the best
surgical treatment?

A. Hemimandibulectomy with disarticulation and reconstruction with titanium plate and


metal condyle

B. Hemimandibulectomy with disarticulation and reconstruction with iliac crest and rib
grafting

C. Enucleation of the lesion

D. Enucleation and curettage of the lesion

Answer: D

Rationale:
The panogram and the stem of the question emphasize the specific subtype of the
ameloblastoma in an 18 year old patient which clearly should render itself to option D
of the treatment which has the marked advantage of decreased morbidity with
predictable healing of the bony defect and the low recurrence rates. Options A and B
are incorrect and are reserved for solid or multicystic ameloblastomas. The intraluminal
subtype of the unicystic ameloblastoma is best treated with enucleation and curettage
(option D) rather than just an enucleation surgery (Option C).

The American Board of Oral and Maxillofacial Surgery 189


2007 Oral and Maxillofacial Surgery Self Assessment Tool (OMSSAT)

Reference:
Gold L.: Biologic Behavior of Ameloblastoma. Oral Maxillofac Surg Clin North Amer.
1991; 3:21-71

Nakamura N, Higuchi Y, Mitsuyasu T, et al, Oral Surg Oral Med Oral Pathol Oral Rad
Endod 2002; 93:13-20.

Kessler HP: Intraosseous ameloblastoma. In: Oral Maxillofac Surg Clinic North Amer,
Odontogenic Tumors DK, White L (editor), Saunders, Philadelphia, 2004, 16:309-322.

The American Board of Oral and Maxillofacial Surgery 190


2007 Oral and Maxillofacial Surgery Self Assessment Tool (OMSSAT)

174. Which of the following represents the most appropriate surgical management of an
adenomatoid odontogenic tumor?

A. En-bloc resection with delayed reconstruction

B. A two-stage surgery: decompression followed by enucleation

C. Enucleation and curettage

D. En-bloc resection and excision of surrounding involved soft tissue

Answer: C

Rationale:
Option A is incorrect since it is the recommended treatment for solid or multicystic
ameloblastoma that has not perforated the cortical bone. Option B is also incorrect as
this surgical modality is recommended for non-neoplastic entities. Option C is the
correct answer as the adenomatoid odontogenic tumor is benign and enucleation and
curettage is curative. Option D is incorrect and is the treatment of choice for
odontogenic myxoma and other aggressive odontogenic tumors.

Reference:
Philipsen JP, Reichart PA, Zhang KH, et al.: Adenomatoid odontogenic tumor: biologic
profile based on 499 cases. J. Oral Pathol Med 1991; 20:149-158.

Carlson ER: Odontogenic cysts and tumors. In: Peterson's Principles of Oral &
Maxillofacial Surgery, M Miloro, GE Ghali, PE Larson, and PD White (editors), Vol.
1, BC Decker, Inc., 2004, Hamilton, p575-596.

The American Board of Oral and Maxillofacial Surgery 191


2007 Oral and Maxillofacial Surgery Self Assessment Tool (OMSSAT)

175. An ameloblastoma exhibiting malignant cytologic features that is found in the lung is best
classified as which of the following diagnoses?

A. Malignant ameloblastoma

B. Ameloblastic carcinoma

C. Metastatic ameloblastoma

D. Clear cell odontogenic carcinoma

Answer: B

Rationale:
Ameloblastic carcinoma. The most important discriminating issue in the correct answer
is the fact that the tumor exhibits malignant cytologic features. As such, the most
accurate description of this process is ameloblastic carcinoma, and not malignant
ameloblastoma, which is a benign tumor by definition. While this tumor is a metastatic
ameloblastoma, this mere designation is incomplete. Also, a clear cell odontogenic
carcinoma is possible, however, there is not enough information to support this answer.

Reference:
Carlson ER: Odontogenic cysts and tumors. In: Miloro M (ed): Peterson's Principles of
Oral and Maxillofacial Surgery. Chapter 30, Hamilton, BC Decker, 575-596, 2004.

The American Board of Oral and Maxillofacial Surgery 192


2007 Oral and Maxillofacial Surgery Self Assessment Tool (OMSSAT)

176. A metastatic tumor of the mandible in a 65 year-old woman is most likely to originate in
which of the following anatomic sites?

A. Colon

B. Kidney

C. Lung

D. Breast

Answer: D

Rationale:
In adults, metastases to the jaws most commonly originate from primary carcinomas of
the breast in women and of the lung in men. Other common primary sites in decreasing
order of frequency are the prostate, gastrointestinal tract, kidney, colon, and rectum.

Reference:
Zarbo RJ, Carlson ER: Malignancies of the Jaws. In: Regezi J, Sciubba J, Jordan R
(eds), Oral Pathology, Clinical Pathologic Correlations, 4th edition, chapter 14, St.
Louis, Elsevier 336, 2003.

The American Board of Oral and Maxillofacial Surgery 193


2007 Oral and Maxillofacial Surgery Self Assessment Tool (OMSSAT)

177. Which of the following features is most consistent with the ameloblastic fibrosarcoma of
the jaws?

A. More commonly located in the maxilla than the mandible

B. More common in seventh decade adults than third decade adults

C. More common in females than males

D. Associated with the development of a painful mass.

Answer: D

Rationale:
The ameloblastic fibrosarcoma is a very rare odontogenic tumor that is most commonly
located in the mandible. It is found with peak incidence in third decade with a mean
age of 25 years. Males are affected more commonly than females with a ratio of about
2:1. The most common presenting symptoms are pain and a swelling.

Reference:
Reichart PA, Philipsen HP (eds): Odontogenic Tumors and Allied Lesions. Chapter 29,
Ameloblastic Fibrosarcoma, London, Quintessence Publishing Co. Ltd. 2004, 255-262.

The American Board of Oral and Maxillofacial Surgery 194


2007 Oral and Maxillofacial Surgery Self Assessment Tool (OMSSAT)

178. A metastatic tumor of the mandible in a 70-year-old man is most likely to originate in
which of the following anatomic sites?

A. Colon

B. Kidney

C. Lung

D. Breast

Answer: C

Rationale:
In adults, metastases to the jaws most commonly originate from primary carcinomas of
the breast in women and of the lung in men. Other common primary sites in decreasing
order of frequency are the prostate, gastrointestinal tract, kidney, colon, and rectum.

Reference:
Zarbo RJ, Carlson ER: Malignancies of the Jaws. In: Regezi J, Sciubba J, Jordan R
(eds), Oral Pathology, Clinical Pathologic Correlations, 4th edition, chapter 14, St.
Louis, Elsevier 336, 2003.

The American Board of Oral and Maxillofacial Surgery 195


2007 Oral and Maxillofacial Surgery Self Assessment Tool (OMSSAT)

179. Which of the following represents the most common site for oral squamous cell carcinoma
in the United States?

A. Tongue

B. Palate

C. Alveolar Gingiva

D. Buccal Mucosa

Answer: A

Rationale:
While exact site-specific incidence is variably reported, general consensus is that the
oral tongue is the most commonly affected site in the oral cavity. This seems to be
especially true of younger patients without significant risk factors where an increased
index of suspicion is warranted.

Reference:
Neville, Damm, Allen, Bouquot. Oral and Maxillofacial Pathology. W.B. Saunders.
1995

The American Board of Oral and Maxillofacial Surgery 196


2007 Oral and Maxillofacial Surgery Self Assessment Tool (OMSSAT)

180. Information that may guide the decision for elective neck dissection in patients with T1 and
T2 oral cavity squamous cell carcinoma includes which of the following?

A. Degree of differentiation

B. Margin STATUS at the time of diagnostic biopsy

C. Length of time that the cancer has been present

D. Tumor depth of invasion

Answer: D

Rationale:
Treatment of the N0 neck with elective neck dissection is indicated when an
approximate 20-30% chance of spread to lymph nodes exists. Length of time, degree of
differentiation, and margin STATUS at diagnostic biopsy are not reliable indicators of
spread to neck lymph nodes. Tumor depth of invasion has been correlated with risk of
positive nodes in a large number of reported studies and can be used as a guide in
making clinical decisions for elective neck dissection in T1 and T2 lesions.

Reference:
Holmes J, Dierks, E. Oral Cancer Treatment. Peterson's Principles of Oral and
Maxillofacial Surgery 2nd Ed. BC Decker, 631-657, 2004

The American Board of Oral and Maxillofacial Surgery 197


2007 Oral and Maxillofacial Surgery Self Assessment Tool (OMSSAT)

181. The presence of a single positive neck lymph node without extracapsular spread after
dissection for head and neck squamous cell carcinoma is equated to a decrease in 5-year
survival by what percent?

A. 10%

B. 25%

C. 50%

D. 75%

Answer: C

Rationale:
A single positive neck lymph node well contained within the capsule decreases 5-year
survival to 50%. If the patient had extracapsular spread associated with the node then
this would have decreased survival by an additional 50% to about 25% 5-year survival.

Reference:
Ghali G, Connor M. Oral Cancer: Classification, Staging, and Diagnosis. Peterson's
Principles of Oral and Maxillofacial Surgery 2nd Ed. BC Decker, 618-630, 2004

The American Board of Oral and Maxillofacial Surgery 198


2007 Oral and Maxillofacial Surgery Self Assessment Tool (OMSSAT)

182. Which of the following features distinguishes lentigo maligna from lentigo maligna
melanoma?

A. Lentigo maligna is a nodular melanoma while lentigo maligna melanoma is a superficial


spreading melanoma.

B. Lentigo maligna is a melanoma in situ while lentigo maligna melanoma develops from a
lentigo maligna.

C. Lentigo maligna is a superficial spreading melanoma while lentigo maligna melanoma


is a nodular melanoma.

D. Lentigo maligna is a primary melanoma while lentigo maligna melanoma is a


previously treated recurrent melanoma.

Answer: B

Rationale:
Lentigo maligna is a melanoma in situ. It is, by definition, a Clark's level I lesion that is
confined to the epithelium. There is no breach of the basement membrane, similar to a
carcinoma in situ. The former name of this entity was a melanotic freckle of Hutchison.
A lentigo maligna melanoma is an invasive melanoma that arose from a lentigo
maligna. As such, the lentigo maligna exhibits radial growth confined to the epithelium
while the lentigo maligna melanoma exhibits vertical growth within the lamina propria.

Reference:
Carlson ER: Melanoma of the Oral and Maxillofacial Region. Oral and Maxillofacial
Surgery Knowledge Update. Rosemont, American Association of Oral and
Maxillofacial Surgeons, Volume 4, PTH 57-69, 2006.

The American Board of Oral and Maxillofacial Surgery 199


2007 Oral and Maxillofacial Surgery Self Assessment Tool (OMSSAT)

183. Which of the following tumors is least likely to be found in the parotid gland?

A. Acinic cell carcinoma

B. Adenoid cystic carcinoma

C. Carcinoma ex-pleomorphic adenoma

D. Polymorphous low-grade adenocarcinoma

Answer: D

Rationale:
PLGA is almost exclusively found in the minor salivary glands. This is a characteristic
feature of this salivary gland tumor. All of the other options occur with reasonable
frequency in the parotid gland.

Reference:
Castle JT, Thompson LDR, from melt RA, Ewing BM, Kessler HP. Polymorphous low-
grade adenocarcinoma: a clinicopathologic study of 164 cases. Cancer 1999;86:207-19

The American Board of Oral and Maxillofacial Surgery 200


2007 Oral and Maxillofacial Surgery Self Assessment Tool (OMSSAT)

184. A 30 year-old female presents to your office with a large mass of her right parotid gland.
On examination you note that the buccal and frontal branches of her facial nerve are not
functioning. Her MRI reveals a large mass with ill-defined borders occupying the parotid
gland. You wish to procure tissue for histologic diagnosis. What is the most appropriate
next step?

A. Incisional biopsy

B. Fine Needle aspiration (FNA)

C. Nerve sparing superficial parotidectomy

D. Total parotidectomy with nerve sacrifice

Answer: B

Rationale:
A parotid incisional biopsy is the method of choice for the initial microscopic diagnosis
of a diffuse, rather than a discrete mass of the parotid gland. It is contraindicated to
perform an incisional biopsy when a neoplasm is suspected due to the seeding of the
overlying skin. The only exception to this general rule is when the parotid neoplasm
has already perforated through skin. The FNAB permits cytologic analysis of a parotid
neoplasm without seeding the skin, which otherwise would require sacrifice at the time
of parotidectomy.

Reference:
Marx RE: Incisional parotid biopsy for diagnosis of systemic disease. Oral and
Maxillofacial Surgery Clinics of North America 7: 505-517, 1995.

The American Board of Oral and Maxillofacial Surgery 201


2007 Oral and Maxillofacial Surgery Self Assessment Tool (OMSSAT)

185. Which of the salivary glands is more prone to development of sialolithiasis?

A. Parotid gland

B. Submandibular gland

C. Sublingual gland

D. Minor salivary glands

Answer: B

Rationale:
Due to the tortuosity of the Wharton's duct, as well as the character of the
submandibular saliva, the submandibular gland is the most common salivary gland to
develop stones.

Reference:
Levy BM, ReMine WH, Devine KD: Salivary gland calculi: Pain, swelling associated
with eating. JAMA 1962; 181:1115.

Berry RL: Sialadenitis and sialolithiasis. Diagnosis and management. Oral and
Maxillofacial Surgery Clinics of North America 7: 479-503, 1995.

The American Board of Oral and Maxillofacial Surgery 202


2007 Oral and Maxillofacial Surgery Self Assessment Tool (OMSSAT)

186. A 69 year-old patient presents to your office with a complaint of an enlarging mass of his
palate. He states that the mass has been present for the past year and has recently ulcerated
and became painful. On clinical examination you note a 2 cm raised ulcerated mass at the
junction of the right hard and soft palates. Which of the following diagnoses is most likely
in this case?

A. Acinic cell carcinoma

B. Mucoepidermoid Carcinoma

C. Polymorphous Low-grade Adenocarcinoma

D. Carcinoma ex-pleomorphic adenoma

Answer: B

Rationale:
The scenario points to a malignant diagnosis. Statistically, the most common minor
salivary gland malignancy of the palate is the mucoepidermoid carcinoma. Of the 692
malignant palatal tumors in the AFIP data, 305 were mucoepidermoid carcinoma. The
second most common malignancy in this location is the adenoid cystic carcinoma.

Reference:
Ellis GL, Auclair PL, Gnepp DR (eds): Surgical pathology of the salivary glands.
Philadelphia, WB Saunders, 144-145, 1991.

The American Board of Oral and Maxillofacial Surgery 203


2007 Oral and Maxillofacial Surgery Self Assessment Tool (OMSSAT)

187. What is the most common salivary gland tumor of the upper lip?

A. Canalicular Adenoma

B. Pleomorphic Adenoma

C. Acinic Cell Carcinoma

D. Mucoepidermoid Carcinoma

Answer: B

Rationale:
The largest database of salivary gland tumors (AFIP) showed the pleomorphic adenoma
to comprise 147 of 413 upper lip tumors, canalicular adenoma 89/413, acinic cell
carcinoma 24/413 and mucoepidermoid carcinoma 12 of 413 tumors.

Reference:
Ellis GL, Auclair PL, Gnepp DR (eds): Surgical pathology of the salivary glands.
Philadelphia, WB Saunders, 1991, pp 144-145.

The American Board of Oral and Maxillofacial Surgery 204


2007 Oral and Maxillofacial Surgery Self Assessment Tool (OMSSAT)

188. A 9 year-old patient presents with a mass on her left parotid gland. Your clinical
examination reveals a 2 cm lesion on the left parotid gland. The lesion is attached to the
overlying skin and the skin is erythematous. Her facial nerve is intact with exception of a
slight buccal branch weakness. Which of the following tumors is the most likely diagnosis?

A. Adenoid cystic carcinoma

B. Acinic cell carcinoma

C. Lymphoma

D. Mucoepidermoid carcinoma

Answer: D

Rationale:
The scenario depicts a malignant neoplasm in the parotid gland of this child.
Statistically, the most likely diagnosis is mucoepidermoid carcinoma.

Reference:
Bentz BG, Hughes A, Ludrmann JP, Maddalozzo J Masses of the salivary gland region
in children. Arch Otolaryngol Head Neck Surg 2000;126:1435-1439.

The American Board of Oral and Maxillofacial Surgery 205


2007 Oral and Maxillofacial Surgery Self Assessment Tool (OMSSAT)

189. What is the most common malignant submandibular gland tumor?

A. Mucoepidermoid carcinoma

B. Carcinoma ex-pleomorphic adenona

C. Adenoid Cystic Carcinoma

D. Acinic Cell Carcinoma

Answer: C

Rationale:
The AFIP data showed 144 adenoid cystic carcinomas of a total of 510 malignant
tumors. The second most common malignancy of the submandibular gland was the
mucoepidermoid carcinoma, accounting for 112 cases.

Reference:
Ellis GL, Auclair PL, Gnepp DR (eds): Surgical pathology of the salivary glands.
Philadelphia, WB Saunders, 1991, pp 144-145.

The American Board of Oral and Maxillofacial Surgery 206


2007 Oral and Maxillofacial Surgery Self Assessment Tool (OMSSAT)

190. A 12 year-old patient is brought to your office for evaluation of a swelling on the left
anterior floor of mouth. The examination reveals a dome-shaped, translucent, fluid filled
elevation on the left anterior floor of mouth. On further examination of the head and neck
you note that the swelling extends to the submental region on the same side. Based on your
clinical diagnosis, what is the best treatment to insure that the lesion does not recur?

A. Incision and drainage

B. Marsupialization

C. Partial/total extirpation of pseudocyst

D. Sublingual gland excision

Answer: D

Rationale:
The scientific literature points to the sublingual gland excision, option D, to be the most
reliably and effective form of treatment to ensure that a ranula does not recur. Incision
and drainage will not resolve a ranula as saliva will continue to collect in the
submucosal space. Marsupialization is a procedure reviewed in the literature; however,
insufficient evidence exists to suggest that this procedure is more effective in managing
the ranula than the sublingual gland excision. Since the gland is the culprit in this
pathologic entity, excision of the pseudocyst will not cure the problem.

Reference:
Catone GA: Sublingual gland mucous escape. Pseudocysts of the oro-cervical region.
Oral and Maxillofacial Surgery Clinics of North America 7: 431-477, 1995.

The American Board of Oral and Maxillofacial Surgery 207


2007 Oral and Maxillofacial Surgery Self Assessment Tool (OMSSAT)

191. A 40 year-old patient presents to the clinic one week after a left nerve sparing superficial
parotidectomy for a pleomorphic adenoma. On exam you note an area of fluctuant swelling
directly underneath the skin overlying the residual parotid bed. You explain to her that she
has a salivary fistula. Which of the following represents the next step?

A. Carryout a wound exploration.

B. Prescribe an anticholinergic agent.

C. Have her return to clinic in 2 weeks for reexamination.

D. Aspirate the area.

Answer: C

Rationale:
Most salivary fistulas resolve within 2 to 3 weeks without intervention. As such, a
wound exploration is not necessary. Aspiration of the area will not resolve the fistula,
as saliva will continue to be formed. Prescribing an anticholinergic agent such as
Robinol is not initially indicated, but may play a role in the management of a chronic
salivary fistula.

Reference:
Lee KJ (7th ed) Essential Otolaryngology, McGraw Hill 1998; pg 523.

The American Board of Oral and Maxillofacial Surgery 208


2007 Oral and Maxillofacial Surgery Self Assessment Tool (OMSSAT)

192. Which of the following represents an advantage of using platelet rich plasma?

A. Increased concentration of growth factors

B. Increased clotting potential

C. Decreased tissue viscosity

D. Decreased immune responses

Answer: A

Rationale:
PRP is an increase in platelets at a wound or recipient bone graft site. Platelets are
involved in hemostasis and contain multiple growth factors in their alpha granules. Yes,
the clotting potential is increased with the use of PRP, but this is not the significant
advantage since even the PPP can increase hemostasis. An increased concentration of
growth factors is the significant advantage with the use of PRP. Tissue viscosity and
immune responses are not changed by the use of PRP.

Reference:
Weibrich et al., Growth factor levels in PRP and correlations with donor age, sex, and
platelet count. Int J Oral Maxillofac Impl 16(5):693-9, 2001.

Weibrich et al. Growth factor levels in the platelet-rich plasma produced by 2 different
methods: curasan-type PRP kit versus PCCS PRP system. Int J Oral Maxillofac Impl
17(2):184-90, 2002.

The American Board of Oral and Maxillofacial Surgery 209


2007 Oral and Maxillofacial Surgery Self Assessment Tool (OMSSAT)

193. Which of the following statements is true regarding full thickness and split thickness skin
grafts?

A. A full thickness skin graft can be expected to contract approximately 80–90 % of its
initial size.

B. Split thickness skin graft donor sites heal primarily by epithelial migration from adnexal
structures.

C. Survival of split thickness skin grafts beyond 72 hours is dependent upon plasmatic
imbibition.

D. Survival of a split thickness or full thickness graft in the initial 24-48 hours is dependent
upon inosculation.

Answer: B

Rationale:
Full thickness skin grafts can be expected to contract 15 -20% and should be harvested
with that in mind. Healing of the donor site in split thickness skin grafting is similar to
burns, and occurs primarily from the contribution of adnexal skin structures. Therefore,
“b” is the best choice. Survival of skin grafts in the initial 48-72 hours is dependent
upon plasmatic circulation or imbibition. Beyond that, neovascularization and
inosculation take over.

Reference:
Ghali et al. Peterson's Principles of Oral and Maxillofacial Surgery. BC Decker 2004.
Baker et al. Local Flaps in Facial Reconstruction. Mosby 1994.

The American Board of Oral and Maxillofacial Surgery 210


2007 Oral and Maxillofacial Surgery Self Assessment Tool (OMSSAT)

194. Which of the following is true regarding the lateral arm vascularized flap?

A. One advantage over the radial forearm flap is the likelihood of direct closure of the
donor site of the lateral arm flap.

B. It is possible that harvest of the lateral arm flap can result in vascular compromise to the
atershed area of the lateral forearm and result in skin loss.

C. The arterial blood supply to the flap is provided by the anterior radial collateral artery.

D. Harvest of the flap should include preservation of the posterior cutaneous nerve of the
arm.

Answer: A

Rationale:
The lateral arm flap serves as a useful alternative to the radial forearm flap for
reconstruction of oral cavity defects. Direct linear closure of the donor site is almost
always possible, and results in a more esthetic site than the radial forearm. Sacrifice of
the posterior radial collateral artery dose not pose any risk to the vascular supply to the
distal arm. Harvest of the flap invariably results in sacrifice of the posterior cutaneous
nerve of the arm, and it can be used for re-innervation.

Reference:
Urken ML, et al. Atlas of regional and free flaps for head and neck reconstruction.
Raven Press 1995. Wolff KD, Holzle F. Raising of Microvascular flaps: A Systemic
Approach. Springer-Verlag. 2005.

The American Board of Oral and Maxillofacial Surgery 211


2007 Oral and Maxillofacial Surgery Self Assessment Tool (OMSSAT)

195. Which of the following statements is true regarding the radial forearm osseocutaneous free
flap?

A. The amount of bone thickness available is limited to 60 % of the radius’ circumference.

B. The amount of bone length available is limited to 5 cm.

C. Prophylactic plating of the donor site allows harvest of full thickness segments of the
radius.

D. It is primarily indicated for defects of the mandibular angle and ramus.

Answer: D

Rationale:
The radial forearm osseocutaneous flap has recently enjoyed a resurgence in popularity
based on Neal Futran’s work. He has found that prophylactic plating of the donor site has
basically eliminated pathologic fractures of the residual radius. Harvest is still limited to
40 % of the radius’ circumference and 10-12 cm of length. It is primarily indicated for
non-tooth bearing areas of the mandible, such as the angle and ramus.

Reference:
Villaret DB, Futran NA. The indications and outcomes in the use of osteocutaneous radial
forearm free flap. Head Neck 2003 Jun 25(6):475-481.

Farwell DG, Futran ND. Oromandibular Reconstruction. Facial Plastic Surgery Vol 16
2000.

The American Board of Oral and Maxillofacial Surgery 212


2007 Oral and Maxillofacial Surgery Self Assessment Tool (OMSSAT)

196. A superiorly based platysma flap receives its dominant blood supply from which of the
following vessels?

A. The occipital artery

B. Submental branch of the facial artery

C. Transverse cervical artery

D. Superior thyroid artery

Answer: B

Rationale:
Three different variations of the platysma flap are available based on the dominant
blood supply. The inferiorly based flap, with no real application in oral/facial
reconstruction, receives its arterial supply from the transverse cervical artery. The
posteriorly based platysma flap receives its blood supply primarily from branches of the
occipital artery. The superiorly based platysma flap receives its blood supply from the
submental branch of the facial artery at or near the inferior border of the mandible. The
superior thyroid artery does not provide blood supply to the flap.

Reference:
Baur DA: The Platysma Myocutaneous Flap. Oral and Maxillofacial Surgery Clinics of
North America. Nov, 2003, 559-564.

The American Board of Oral and Maxillofacial Surgery 213


2007 Oral and Maxillofacial Surgery Self Assessment Tool (OMSSAT)

197. Which of the following is the most important determinant in survival of a local random
pattern skin flap of the face?

A. Width of the flap base

B. Length of the flap

C. Perfusion pressure of nutrient vessels to the flap

D. Amount of reactive oxygen intermediates

Answer: C

Rationale:
It now recognized that the length to width ratio of facial skin flaps is only a rough
guideline and can vary with different flaps. Generally a 3-1 ratio is felt to be optimal.
However, the survival of a skin flap is primarily dependent on the intravascular
resistance and perfusion pressure of the nutrient vessels. If the perfusion pressure drops
below a critical closing pressure of the arterioles in the subdermal plexus, nutritional
blood flow will cease and necrosis will occur. A flap with a wider base will only
include additional subdermal vessels that have the same perfusion pressure since they
are all based upon the same feeding vessel.

Reference:
Baker SR, Swanson NA; Local Flaps in Facial Reconstruction. Mosby, 15-30, 1995

The American Board of Oral and Maxillofacial Surgery 214


2007 Oral and Maxillofacial Surgery Self Assessment Tool (OMSSAT)

198. Which of the following statements is true regarding the superiorly based
sternocleidomastoid myocutaneous flap?

A. The spinal accessory nerve can be sacrificed during elevation of the flap without
morbidity.

B. Survival of the skin paddle requires that the blood supply must traverse an intervening
muscular unit .

C. It can successfully and reliably reconstruct anterior floor of mouth defects.

D. The muscle is relatively resistant to fibrosis following radiation treatment to the neck.

Answer: B

Rationale:
The blood supply to the skin paddle of the sternocleidomastoid myocutaneous flap must
traverse through an intermediate muscular layer, the platysma muscle. The spinal
accessory nerve should not be sacrificed as it also innervates the trapezius muscle and to
do so would cause shoulder dysfunction. The arc of rotation of the SCM flap is
inadequate to reconstruct anterior floor of mouth defects. Radiation treatment to the
neck can cause fibrosis and reduction of the muscular blood supply, making dissection
more difficult and compromising flap survivability.

Reference:
Urkin ML, Cheney ML, Sullivan MJ, Biller HF; Atlas of Regional and Free Flaps for
Head and Neck Reconstruction. Raven Press, New York, NY. 1995, 49-64

Carlson ER; Regional Flaps in Maxillofacial Reconstruction. Oral and Maxillofacial


Surgery Clinics of North America, 5(4), 667-685, 1993

The American Board of Oral and Maxillofacial Surgery 215


2007 Oral and Maxillofacial Surgery Self Assessment Tool (OMSSAT)

199. The dominant blood supply of a trapezius myocutaneous flap is provided by which of the
following vessels?

A. Dorsal scapular artery

B. Transverse cervical artery

C. Paraspinous perforators

D. Occipital artery

Answer: B

Rationale:
While in situ, the trapezius muscle receives minor contributions from the dorsal
scapular artery, paraspinous perforators, and the occipital artery. However, once the
flap is elevated, the transverse cervical artery is the primary nutrient vessel of this axial
pattern flap. The transverse cervical artery is often sacrificed in patients who have
undergone a neck dissection. If a trapezius myocutaneous flap is considered in one of
these patients, the presence of the vessel must be confirmed by angiography.

Reference:
Urkin ML, Cheney ML, Sullivan MJ, Biller HF; Atlas of Regional and Free Flaps for
Head and Neck Reconstruction. Raven Press, New York, NY. 1995, 29-48.

Carlson ER; Regional Flaps in Maxillofacial Reconstruction. Oral and Maxillofacial


Surgery Clinics of North America, 5(4), 667-685, 1993

The American Board of Oral and Maxillofacial Surgery 216


2007 Oral and Maxillofacial Surgery Self Assessment Tool (OMSSAT)

200. Which of the following enzymes is most responsible for mediating a reperfusion injury in
an ischemic random pattern skin flap?

A. Acetylaldehyde dehydrogenase

B. Xanthine oxidase

C. Dopa decarboxylase

D. Cystathione synthetase

Answer: B

Rationale:
In ischemic tissue, purines are oxidized by xanthine oxidase to xanthine and uric acid.
The highly toxic superoxide free radical is formed as a byproduct. When oxygen is
reintroduced suddenly during reperfusion, large amounts of superoxide are formed
causing cellular damage. Acetylaldehyde dehydrogenase plays a role in the metabolism
of ethanol. Dopa decarboxylase metabolizes L-dopa to dopamine. Cystathione
synthetase is an enzyme involved in homocysteine metabolism.

Reference:
Baker SR, Swanson NA; Local Flaps in Facial Reconstruction. Mosby, 15-30, 1995

The American Board of Oral and Maxillofacial Surgery 217


2007 Oral and Maxillofacial Surgery Self Assessment Tool (OMSSAT)

201. Which of the following is true regarding the temporalis muscle flap in mid-face
reconstruction?

A. Cannot be used as a carrier of calvarial bone

B. Cannot transfer overlying temporal skin

C. The muscle flap cannot be skin grafted

D. Cannot be safely elevated without damaging the frontal branch of the facial nerve

Answer: B

Rationale:
The temporal muscle flap cannot be used to transfer overlying temporal skin due to a
lack of myocutaneous perforators over the muscle. The muscle flap can successfully be
grafted with a split thickness skin graft. The muscle flap can be used as a carrier of
vascularized outer table calvarial bone for orbital or palatal reconstruction. The frontal
branch of the facial nerve is located in the temporal parietal fascia, superficial to the
temporal muscle fascia. This location is predictable and the flap can be elevated
without damaging the frontal branch.

Reference:
Urkin ML, Cheney ML, Sullivan MJ, Biller HF; Atlas of Regional and Free Flaps for
Head and Neck Reconstruction. Raven Press, New York, NY. 1995, 65-76.

The American Board of Oral and Maxillofacial Surgery 218


2007 Oral and Maxillofacial Surgery Self Assessment Tool (OMSSAT)

202. Sixty percent of the lower lip is missing following tumor ablation. Frozen sections have
verified tumor free margins. Of the following choices, which is the best option for
obtaining an adequate functional and esthetic result in reconstructing the lower lip?

A. The pectoralis major flap

B. The Karapandzic flap

C. The Abbe-Estlander flap

D. The deltopectoral flap

Answer: B

Rationale:
In most instances of reconstructing one-half to two-thirds of the lower lip, the
Karapandzic flap provides a good functional and esthetic result. The Karapandzic flap
re-approximates the orbicularis oris muscle providing for adequate sphincter function of
the lips. In addition, since the sensory and motor nerves are preserved, the flap allows
for a sensate and functional flap. The pectoralis major flap or the deltopectoral flap
could be used to reconstruct the lower lip. However, these flaps would not provide
adequate sensation or function, and the patient would have poor oral competence. In
addition, the esthetic result would be less than optimal. The Abbe-Estlander flap does
not provide sufficient soft tissue for reconstruction of 60% of the lower lip, and its
indication is for reconstruction of the oral commissure.

Reference:
Baker SR, Swanson NA; Local Flaps in Facial Reconstruction. Mosby, 345-396, 1995

Carlson ER; Regional Flaps in Maxillofacial Reconstruction. Oral and Maxillofacial


Surgery Clinics of North America, 5(4), 667-685, 1993

The American Board of Oral and Maxillofacial Surgery 219


2007 Oral and Maxillofacial Surgery Self Assessment Tool (OMSSAT)

203. When medicinal leeches are used in salvaging flaps suffering from venous congestion,
antibiotic prophylaxis against which of the following organisms is indicated?

A. Pseudomonas aeruginosa

B. Pasteurella multocida

C. Aeromonas hydrophilia

D. Bacteroides species

Answer: C

Rationale:
The leech gut contains an endosymbiotic bacterium Aeromonas hydrophilia. The
patient is at risk of developing an infection from this organism. Therefore, prophylaxis
against A. hydrophilia is recommended in patients receiving leech therapy.
Trimethoprim/sulfa, fluoroquinolones, or third generation cephalosporins are the
antibiotics of choice. P. multocida is associated with cat bites. Bacteroides species are
associated with odontogenic infections. P. aeruginosa is responsible for a myriad of
infectious processes.

Reference:
Utley, DS: The Failing Flap in Facial Plastic and Reconstructive Surgery. Laryngoscope
108:1129-35. 1998

The American Board of Oral and Maxillofacial Surgery 220

Potrebbero piacerti anche